National Academies Press: OpenBook

Pile Design for Downdrag: Examples and Supporting Materials (2024)

Chapter: Appendix J: Design Example 8 - Embankment Over Clay Over Rock Using PileAXL

« Previous: Appendix I: Design Example 7 - Liquefaction in Gravel Using PileAXL and TZPILE
Page 269
Suggested Citation:"Appendix J: Design Example 8 - Embankment Over Clay Over Rock Using PileAXL." National Academies of Sciences, Engineering, and Medicine. 2024. Pile Design for Downdrag: Examples and Supporting Materials. Washington, DC: The National Academies Press. doi: 10.17226/27864.
×

APPENDIX J

Design Example 8 — Embankment Over Clay Over Rock Using PileAXL

Table J1. Budge and Dasenbrock (2016) MN 74551 data.

Pile Material Steel
Pile Shape H-Pile (HP12x53)
Pile Width [in] 12
Pile Height [in] 11.8
Pile Web and Flange Thickness [in] 0.435
Pile Area [in2] 15.19
Pile Embedded Length [ft] 52
Pile Modulus [ksi] 29000
Top Load on Pile [lb] 0
Number of Pile Increments 52
Soil Compression Ratio (Rc) 0.033
Soil Recompression Ratio (Rr) 0.005
Depositional Environment Normally Consolidated
Ground Water Table Depth [ft] 0
Fill Height [ft] 29
Fill Unit Weight [pcf] 120

The data for the design example that is contained herein were obtained from Budge and Dasenbrock (2016). Specific information related to the piles for the Minnesota 74551 bridge is included in Table J1. The MN 74551 bridge was constructed as a grade separation/overpass for an existing railway line. The soils at the site were normally consolidated and the settlement resulting from the overpass fill was of concern for drag load and downdrag development on the abutment piles. The piles that were driven at the site were HP12x53 H-piles (width=12in, web thickness=0.435in, pile area=15.19in2). The piles were driven to an embedded length of 52 feet with an additional 30 feet of stickup to protrude through the embankment fill. At 52 feet, the piles were expected to encounter a rock bearing layer (Figure J1). Downdrag and drag load prevention methods in the form of corrugated metal pile sleeves within the fill were used. Several of the piles were also instrumented to investigate the amount of reduction attributed to the prevention methods. Method A (flow chart provided on next page) as proposed by the NCHRP12-116A team was employed.

Soil properties for the MN bridge site
Figure J1. Soil properties for the MN bridge site.
Page 270
Suggested Citation:"Appendix J: Design Example 8 - Embankment Over Clay Over Rock Using PileAXL." National Academies of Sciences, Engineering, and Medicine. 2024. Pile Design for Downdrag: Examples and Supporting Materials. Washington, DC: The National Academies Press. doi: 10.17226/27864.
×

Step 1: Establish oil data

The undrained shear strength was correlated with the N60 corrected blow count values using the atmospheric pressure, pa=2116psf (Equation 1). The soil layering was developed based on the observed SPT blow count values and the obtained moisture content profile. An undrained shear strength value is presented for the rock layer below 52 feet instead of an unconfined compression strength because the Ensoft TZPILE(Ensoft 2021) program does contain pre-programmed t-z and q-w curves for rock.

c u p q = 0.6 N 60 Eqn. 1 (Kulhawy and Mayne, 1990)
Page 271
Suggested Citation:"Appendix J: Design Example 8 - Embankment Over Clay Over Rock Using PileAXL." National Academies of Sciences, Engineering, and Medicine. 2024. Pile Design for Downdrag: Examples and Supporting Materials. Washington, DC: The National Academies Press. doi: 10.17226/27864.
×

presentation

Page 272
Suggested Citation:"Appendix J: Design Example 8 - Embankment Over Clay Over Rock Using PileAXL." National Academies of Sciences, Engineering, and Medicine. 2024. Pile Design for Downdrag: Examples and Supporting Materials. Washington, DC: The National Academies Press. doi: 10.17226/27864.
×

Step 2: Determine soil settlement

Similar to Design Example 1, the amount of expected soil settlement is determined using consolidation theory. The soil settlement profile shown in Figure J2 was created by discretizing the soil into sublayers (52, 1ft thick layers) and then calculating the amount of settlement within each sublayer resulting from a non-symmetric 29ft thick, 28.7ft crest, 59ft base embankment fill (Figure J3), with a unit weight of 120pcf, being placed on top of the two-layer clay soil profile overlaying the weathered rock layer and parent rock. The irregular shape of the embankment was attributed to the existing railway right-of-way on the left side of the embankment shown in Figure J3. Equations similar to those used in Design Example 1 were used to determine the soil settlement. Two differences were observed between the soil settlement calculations contained herein and the soil settlement calculations in Design Example 1. These include: 1) the embankment is not symmetric about the center therefore influence factors for both the left and the right side of the embankment must be determined as a function of depth, and 2) the compression and recompression ratios were used for the settlement calculations instead of soil modulus values. The equations used to find the influence factor (I) are presented in Equations 2 through 4. The variables required in Equations 2 through 4 are presented in Figure J3. The amount of settlement was determined for each sublayer using Equations 5 and 6. The z, σzo′, α, I, ∆σ, σzf’, and δ are presented in Table J2.

Soil settlement profile for the MN bridge site
Figure J2. Soil settlement profile for the MN bridge site.
I = 1 π [ ( B 1 + B 2 B 2 ) ( α 1 + α 2 ) B 1 B 2 ( α 1 ) ] Eqn. 2
α 1 ( r a d i a n s ) = t a n 1 ( B 1 z ) Eqn. 3
α 2 ( r a d i a n s ) = t a n 1 ( B 1 + B 2 z ) t a n 1 ( B 1 z ) Eqn. 4
Δ σ = q I = ( γ f i l l H f i l l ) I Eqn. 5
S u l t , i n c r e m e n t = R c * t * log ( σ o ¯ + Δ σ σ o ) Eqn. 6
Embankment geometry for the MN bridge site
Figure J3. Embankment geometry for the MN bridge site.
Page 273
Suggested Citation:"Appendix J: Design Example 8 - Embankment Over Clay Over Rock Using PileAXL." National Academies of Sciences, Engineering, and Medicine. 2024. Pile Design for Downdrag: Examples and Supporting Materials. Washington, DC: The National Academies Press. doi: 10.17226/27864.
×

Table J2. Parameters used to find soil settlement.

Layer Depth z σzo α1,L α1,L IL α1,R α1,R IR Σ(I) ∆σ σ′zf δinc Σδ
0 - 1 0.5 28.8 0.025 1.446 0.500 0.014 1.551 0.500 1.000 3479 3508 0.069 0.069
1 - 2 1.5 86.4 0.067 1.212 0.493 0.043 1.510 0.500 0.993 3455 3542 0.054 0.123
2 - 3 2.5 144 0.095 1.012 0.473 0.071 1.470 0.500 0.973 3387 3531 0.046 0.169
3 - 4 3.5 201.6 0.108 0.852 0.443 0.099 1.430 0.500 0.943 3282 3484 0.041 0.210
4 - 5 4.5 259.2 0.111 0.727 0.408 0.126 1.390 0.500 0.908 3161 3420 0.037 0.247
5 - 6 5.5 316.8 0.109 0.629 0.374 0.153 1.351 0.500 0.873 3039 3356 0.034 0.281
6 - 7 6.5 374.4 0.104 0.552 0.341 0.179 1.313 0.499 0.840 2925 3299 0.031 0.313
7 - 8 7.5 432 0.098 0.490 0.312 0.205 1.275 0.499 0.811 2822 3254 0.029 0.342
8 - 9 8.5 489.6 0.092 0.440 0.286 0.230 1.239 0.498 0.785 2731 3220 0.027 0.369
9 - 10 9.5 547.2 0.086 0.399 0.264 0.253 1.203 0.498 0.762 2650 3197 0.025 0.395
10 - 11 10.5 604.8 0.080 0.364 0.244 0.276 1.168 0.497 0.741 2579 3184 0.024 0.419
11 - 12 11.5 662.4 0.075 0.335 0.227 0.298 1.134 0.496 0.723 2516 3178 0.023 0.441
12 - 13 12.5 720 0.071 0.310 0.211 0.319 1.101 0.495 0.707 2459 3179 0.021 0.463
13 - 14 13.5 777.6 0.067 0.288 0.198 0.339 1.070 0.494 0.692 2409 3186 0.020 0.483
14 - 15 14.5 835.2 0.063 0.269 0.186 0.358 1.039 0.493 0.679 2363 3198 0.019 0.502
15 - 16 15.5 892.8 0.059 0.253 0.175 0.376 1.009 0.492 0.667 2321 3214 0.018 0.521
16 - 17 16.5 950.4 0.056 0.238 0.165 0.393 0.981 0.490 0.656 2282 3233 0.018 0.538
17 - 18 17.5 1008 0.054 0.225 0.157 0.409 0.953 0.489 0.646 2247 3255 0.017 0.555
18 - 19 18.5 1065.6 0.051 0.213 0.149 0.424 0.927 0.487 0.636 2214 3280 0.016 0.572
19 - 20 19.5 1123.2 0.049 0.202 0.142 0.438 0.902 0.485 0.627 2183 3306 0.016 0.587
20 - 21 20.5 1180.8 0.047 0.193 0.135 0.451 0.877 0.484 0.619 2154 3335 0.015 0.602
21 - 22 21.5 1238.4 0.045 0.184 0.129 0.463 0.854 0.482 0.611 2126 3365 0.014 0.617
22 - 23 22.5 1296 0.043 0.176 0.124 0.474 0.831 0.480 0.604 2100 3396 0.014 0.630
23 - 24 23.5 1353.6 0.041 0.169 0.119 0.484 0.809 0.477 0.596 2075 3429 0.013 0.644
24 - 25 24.5 1411.2 0.039 0.162 0.114 0.494 0.788 0.475 0.589 2051 3463 0.013 0.657
25 - 26 25.5 1468.8 0.038 0.156 0.110 0.503 0.768 0.473 0.583 2028 3497 0.013 0.669
26 - 27 26.5 1526.4 0.037 0.150 0.106 0.511 0.749 0.470 0.576 2006 3532 0.012 0.681
27 - 28 27.5 1594 0.035 0.144 0.102 0.519 0.731 0.468 0.570 1985 3579 0.012 0.693
28 - 29 28.5 1661.6 0.034 0.139 0.099 0.526 0.713 0.465 0.564 1964 3625 0.011 0.704
29 - 30 29.5 1729.2 0.033 0.135 0.096 0.532 0.696 0.463 0.558 1943 3673 0.011 0.715
30 - 31 30.5 1796.8 0.032 0.130 0.093 0.538 0.680 0.460 0.553 1924 3720 0.011 0.726
31 - 32 31.5 1864.4 0.031 0.126 0.090 0.543 0.664 0.458 0.547 1904 3769 0.010 0.736
32 - 33 32.5 1932 0.030 0.122 0.087 0.547 0.649 0.455 0.542 1885 3817 0.010 0.746
33 - 34 33.5 1999.6 0.029 0.119 0.084 0.551 0.634 0.452 0.537 1867 3867 0.010 0.755
34 - 35 34.5 2067.2 0.028 0.115 0.082 0.555 0.620 0.449 0.531 1849 3916 0.009 0.764
35 - 36 35.5 2134.8 0.028 0.112 0.080 0.558 0.607 0.446 0.526 1831 3966 0.009 0.773
36 - 37 36.5 2202.4 0.027 0.109 0.078 0.561 0.594 0.444 0.521 1814 4016 0.009 0.782
37 - 38 37.5 2270 0.026 0.106 0.076 0.563 0.582 0.441 0.516 1797 4067 0.008 0.790
38 - 39 38.5 2337.6 0.026 0.104 0.074 0.565 0.569 0.438 0.511 1780 4117 0.008 0.799
39 - 40 39.5 2405.2 0.025 0.101 0.072 0.567 0.558 0.435 0.507 1763 4169 0.008 0.807
40 - 41 40.5 2472.8 0.024 0.098 0.070 0.568 0.547 0.432 0.502 1747 4220 0.008 0.814
41 - 42 41.5 2540.4 0.024 0.096 0.068 0.569 0.536 0.429 0.497 1731 4271 0.007 0.822
42 - 43 42.5 2608 0.023 0.094 0.067 0.570 0.526 0.426 0.493 1715 4323 0.007 0.829
43 - 44 43.5 2675.6 0.023 0.092 0.065 0.571 0.516 0.423 0.488 1700 4375 0.007 0.836
44 - 45 44.5 2743.2 0.022 0.090 0.064 0.571 0.506 0.420 0.484 1684 4427 0.007 0.843
45 - 46 45.5 2810.8 0.022 0.088 0.063 0.571 0.496 0.417 0.480 1669 4480 0.007 0.850
46 - 47 46.5 2878.4 0.021 0.086 0.061 0.571 0.487 0.414 0.475 1654 4533 0.007 0.856
47 - 48 47.5 2946 0.021 0.084 0.060 0.570 0.479 0.411 0.471 1639 4585 0.006 0.863
48 - 49 48.5 3018.6 0.020 0.082 0.059 0.570 0.470 0.408 0.467 1625 4643 0.006 0.869
49 - 50 49.5 3091.2 0.020 0.081 0.058 0.569 0.462 0.405 0.463 1610 4702 0.006 0.875
50 - 51 50.5 3163.8 0.020 0.079 0.056 0.568 0.454 0.402 0.459 1596 4760 0.006 0.881
51 - 52 51.5 3236.4 0.019 0.078 0.055 0.567 0.446 0.399 0.455 1582 4819 0.006 0.887
z=layer midpoint depth [ft], σ′z=vertical effective stress [psf], α1 and α2=angles [radians] from Figure J3, I=influence factor, ∆σ=change in stress [psf], δinc=settlement of sublayer [ft], Σδ (from bottom to top) = soil settlement profile (Figure J2). L=left, R=right, o=initial, f=final.
Page 274
Suggested Citation:"Appendix J: Design Example 8 - Embankment Over Clay Over Rock Using PileAXL." National Academies of Sciences, Engineering, and Medicine. 2024. Pile Design for Downdrag: Examples and Supporting Materials. Washington, DC: The National Academies Press. doi: 10.17226/27864.
×

Step 3: Establish pile data

The pile data was loaded into the Innovative Geotechnics (2023) PileAXL program (Version 2.5). As mentioned in the description of the previous design examples that used the Innovative Geotechnics programs, the programs only accept metric units. Therefore, many of the parameters were converted between imperial units and metric units. The input data, within the PileAXL program, are shown in Figures J4 through J12. The pile section properties and analysis options are shown in Figures J4 and J5, respectively. As recommended by Innovative Geotechnics, the User Defined option was selected to prevent the use of the gross area of the pile.

Establishment of pile section data within the PileAXL software program
Figure J4. Establishment of pile section data within the PileAXL software program.
Establishment of the pile length, number of increments, top load, and design code
Page 275
Suggested Citation:"Appendix J: Design Example 8 - Embankment Over Clay Over Rock Using PileAXL." National Academies of Sciences, Engineering, and Medicine. 2024. Pile Design for Downdrag: Examples and Supporting Materials. Washington, DC: The National Academies Press. doi: 10.17226/27864.
×
Figure J5. Establishment of the pile length, number of increments, top load, and design code.
Partial factors of safety used within the PileAXL software program
Figure J6. Partial factors of safety used within the PileAXL software program.

Step 4: Compute unit side resistance

The working load design approach that was utilized is presented in Figure J6. As with all of the other design examples, all of the used loads were unfactored loads and all of the determined resistances were unfactored resistances. Therefore, the factors of safety values in Figure J6 were set to unity. The different soil layer materials were created using the Material Sets, as shown in Figure J7. Each of the materials was created and then modified by selecting the Edit button to enable input of the various soil parameters (material type, material name, total unit weight, undrained shear strength, and undrained shear strength increase) as shown in Figure J8. Although the bearing layer was rock and there is a rock option in the PileAXL software, the PileAXL software required unconfined compressive strength that was not available. Therefore, clay parameters were used for the rock layer; the undrained shear strength (1276kPa) was selected from back-analyzing nominal unit tip resistance values for dolomite (240ksf) that were recommended by the Illinois Department of Transportation (2009). After assigning material properties to the respective layers (Figure J9), all of the maximum unit end bearing and maximum unit side resistance values remained at the default values (Figure J10). All of the required information was available within the program at the end of the definition of soil layers stage (Figure J11). The program was then executed to obtain the desired results that are presented in Figure J12 and Table J3. The discretized side resistance is represented as ∆Q in Table J3.

Define - Soil Materials within the PileAXL software program
Figure J7. Define - Soil Materials within the PileAXL software program.

Steps 5 and 6: Develop the depth-dependent load profile anddepth-dependent resistance profile

The depth-dependent load profile and resistance profile data are presented in Table J3. Specifically, the load data are represented by the variable Q in Table J3 and the resistance data are represented as R in Table J3.

Steps 7, 8, and 9: Develop the depth-dependent combined load-resistance curve, identify the location of the neutral plane, and calculate the drag load in the pile

The depth-dependent combined load-resistance curve is plotted in Figure J13. The combined curve was developed by plotting the minimum value of load and resistance at each depth, as a function of depth (Figure J13). The location of the neutral plane was identified as the depth at which the maximum value of the load-resistance value occurred. From the load-resistance curve, the neutral plane was identified

Page 276
Suggested Citation:"Appendix J: Design Example 8 - Embankment Over Clay Over Rock Using PileAXL." National Academies of Sciences, Engineering, and Medicine. 2024. Pile Design for Downdrag: Examples and Supporting Materials. Washington, DC: The National Academies Press. doi: 10.17226/27864.
×

to occur at a depth of 43ft below the existing ground surface. The drag load was determined to be 44.0tons.

Steps 10 and 11: Calculate the toe movement and elastic compression in the pile; determine geotechnical resistance of the pile

As shown in Figure J14, a calculated pile head movement of 0.441in and a geotechnical resistance of 88.8kN were obtained. For completeness, the data contained in Figure J14 are also tabulated in Table J4. According to the NCHRP12-116A Method A flowchart, the soil settlement-pile settlement data (Step 12) are not required because the pile is tipped into rock. Although toe movement and elastic compression were obtained, these values are not presented because Step 12 was not needed.

Step 13: Perform limit state checks

Limit state checks were performed to determine if the pile size was suitable for the design loads. For the structural strength limit state, the determined drag load (44tons) was multiplied by the drag load factor (γDR=1.1) to obtain a factored drag load of 48tons. The unfactored and factored top load deadload were 0tons. The combined total factored load was 48tons. The yield stress for the steel pile was assumed to be 45ksi resulting in a factored structural stress of 40.5ksi (0.9*45ksi) and a factored structural strength of 308tons when the stress was multiplied by the cross-sectional area of the pile wall (15.19in2). If Grade 3 A-252 HP12x53 H-piles were used then the piles are adequately sized; the factored structural strength was determined to be greater than the combined total factored load.

Define – Soil Materials – Edit within the PileAXL software program for the different materials
Page 277
Suggested Citation:"Appendix J: Design Example 8 - Embankment Over Clay Over Rock Using PileAXL." National Academies of Sciences, Engineering, and Medicine. 2024. Pile Design for Downdrag: Examples and Supporting Materials. Washington, DC: The National Academies Press. doi: 10.17226/27864.
×
Define – Soil Materials – Edit within the PileAXL software program for the different materials
Figure J8. Define – Soil Materials – Edit within the PileAXL software program for the different materials.
Define – Soil Layers within the PileAXL software program
Figure J9. Define – Soil Layers within the PileAXL software program.
Define – Soil Layers – Edit within the PileAXL software program for the different soil layers
Page 278
Suggested Citation:"Appendix J: Design Example 8 - Embankment Over Clay Over Rock Using PileAXL." National Academies of Sciences, Engineering, and Medicine. 2024. Pile Design for Downdrag: Examples and Supporting Materials. Washington, DC: The National Academies Press. doi: 10.17226/27864.
×
Define – Soil Layers – Edit within the PileAXL software program for the different soil layers
Figure J10. Define – Soil Layers – Edit within the PileAXL software program for the different soil layers.
Page 279
Suggested Citation:"Appendix J: Design Example 8 - Embankment Over Clay Over Rock Using PileAXL." National Academies of Sciences, Engineering, and Medicine. 2024. Pile Design for Downdrag: Examples and Supporting Materials. Washington, DC: The National Academies Press. doi: 10.17226/27864.
×
Main window of Pile AXL after inputting all of the required data
Figure J11. Main window of Pile AXL after inputting all of the required data.
PileAXL output data window after analyzing the data
Figure J12. PileAXL output data window after analyzing the data.
Page 280
Suggested Citation:"Appendix J: Design Example 8 - Embankment Over Clay Over Rock Using PileAXL." National Academies of Sciences, Engineering, and Medicine. 2024. Pile Design for Downdrag: Examples and Supporting Materials. Washington, DC: The National Academies Press. doi: 10.17226/27864.
×

Table J3. PileAXL output for the MN 74551 bridge abutment piles.

Depth ULS fs ULS Qs ULS Qb ∆Q Q R min(Q,R) δpile Comments:
ULS fs = Ultimate unit side resistance (PileAXL output), ULS Qs = Ultimate total side resistance (PileAXL output), ULS Qb = Ultimate bearing resistance (PileAXL output), ∆Q = discretized side resistance (Excel calculation), Q = cumulative load in the pile (Excel calculation), R= resistance from soil surrounding pile; resistance values are calculated at the top of each sublayer (Excel calculation), min(Q,R) = minimum of Q and R for each depth (Excel calculation), δpile=Settlement of pile obtained by determining the pile head movement from the PileAXL load-settlement curve (Figure J13) and the elastic compression.
[ft] [tsf] [tons] [tons] [tons] [tons] [tons] [tons] [in]
0.000 0.700 0.000 0.361 0.328 0.000 89.099 0.000 0.441
1.000 1.067 0.328 0.361 0.434 0.328 88.770 0.328 0.441
2.000 1.269 0.762 0.361 0.497 0.762 88.336 0.762 0.441
3.000 1.404 1.259 0.361 0.541 1.259 87.840 1.259 0.441
4.000 1.509 1.801 0.361 0.577 1.801 87.298 1.801 0.441
5.000 1.596 2.378 0.361 0.607 2.378 86.721 2.378 0.441
6.000 1.670 2.984 0.361 0.633 2.984 86.114 2.984 0.441
7.000 1.736 3.617 0.361 0.656 3.617 85.481 3.617 0.441
8.000 1.795 4.273 0.361 0.677 4.273 84.825 4.273 0.441
9.000 1.848 4.950 0.361 0.696 4.950 84.149 4.950 0.441
10.000 1.898 5.646 0.361 0.714 5.646 83.452 5.646 0.441
11.000 1.943 6.360 0.361 0.730 6.360 82.739 6.360 0.441
12.000 1.986 7.090 0.361 0.746 7.090 82.009 7.090 0.441
13.000 2.026 7.836 0.361 0.764 7.836 81.263 7.836 0.441
14.000 2.083 8.600 0.361 0.788 8.600 80.499 8.600 0.441
15.000 2.157 9.387 0.361 0.815 9.387 79.711 9.387 0.441
16.000 2.227 10.202 0.361 0.840 10.202 78.897 10.202 0.441
17.000 2.296 11.042 0.361 0.866 11.042 78.056 11.042 0.441
18.000 2.362 11.908 0.361 0.890 11.908 77.191 11.908 0.441
19.000 2.427 12.798 0.361 0.914 12.798 76.301 12.798 0.441
20.001 2.490 13.712 0.361 0.937 13.712 75.387 13.712 0.441
21.001 2.552 14.649 0.361 0.959 14.649 74.450 14.649 0.441
22.001 2.612 15.608 0.361 0.982 15.608 73.491 15.608 0.441
23.001 2.671 16.590 0.361 1.003 16.590 72.509 16.590 0.440
24.001 2.728 17.593 0.361 1.024 17.593 71.506 17.593 0.440
25.001 2.784 18.617 0.361 1.045 18.617 70.482 18.617 0.440
26.001 2.839 19.662 0.361 1.074 19.662 69.437 19.662 0.440
27.001 2.941 20.736 0.371 1.120 20.736 68.363 20.736 0.440
28.001 3.085 21.855 0.392 1.173 21.855 67.244 21.855 0.440
29.001 3.229 23.028 0.412 1.227 23.028 66.070 23.028 0.440
30.001 3.373 24.255 0.432 1.280 24.255 64.844 24.255 0.440
31.001 3.516 25.535 0.452 1.333 25.535 63.564 25.535 0.439
32.001 3.660 26.869 0.473 1.387 26.869 62.230 26.869 0.439
33.001 3.804 28.255 0.493 1.440 28.255 60.843 28.255 0.439
34.001 3.947 29.696 0.513 1.494 29.696 59.403 29.696 0.439
35.001 4.091 31.189 0.533 1.547 31.189 57.910 31.189 0.439
36.001 4.235 32.736 0.554 1.600 32.736 56.362 32.736 0.439
37.001 4.378 34.337 0.574 1.654 34.337 54.762 34.337 0.438
38.001 4.522 35.990 0.594 1.707 35.990 53.108 35.990 0.438
39.001 4.665 37.698 0.614 1.760 37.698 51.401 37.698 0.438
40.001 4.809 39.458 0.635 1.814 39.458 49.641 39.458 0.438
41.001 4.952 41.272 0.655 1.867 41.272 47.827 41.272 0.438
42.001 5.096 43.139 0.675 1.920 43.139 45.960 43.139 0.437
43.001 5.239 45.059 0.695 1.974 45.059 44.040 44.040 0.437
44.001 5.383 47.033 0.716 2.027 47.033 42.066 42.066 0.437
45.001 5.526 49.060 0.736 2.080 49.060 40.039 40.039 0.437
46.001 5.669 51.140 0.756 2.134 51.140 37.959 37.959 0.436
47.001 5.813 53.273 0.776 2.338 53.273 35.825 35.825 0.436
48.001 6.772 55.612 1.029 2.837 55.612 33.487 33.487 0.436
49.001 8.494 58.448 1.525 3.539 58.448 30.651 30.651 0.436
50.001 10.555 61.988 2.021 4.287 61.988 27.111 27.111 0.436
51.001 12.516 66.274 2.518 10.174 66.274 22.824 22.824 0.435
Page 281
Suggested Citation:"Appendix J: Design Example 8 - Embankment Over Clay Over Rock Using PileAXL." National Academies of Sciences, Engineering, and Medicine. 2024. Pile Design for Downdrag: Examples and Supporting Materials. Washington, DC: The National Academies Press. doi: 10.17226/27864.
×
52.001 42.239 76.448 12.650 76.448 12.650 12.650 0.435
PileAXL obtained neutral plane
Figure J13. PileAXL obtained neutral plane.
Load-settlement curve from PileAXL after being changed to imperial units. The unfactored top load applied to pile in PileAXL program to develop the cure was 600kN
Figure J14. Load-settlement curve from PileAXL after being changed to imperial units. The unfactored top load applied to pile in PileAXL program to develop the cure was 600kN.

Table J4. Load-settlement curve data.

δhead Qhead Comments:
δhead=Pile head movement, Qhead=Pile head load
[in] [tons]
0.000 0.0
0.018 7.9
0.036 15.7
0.056 23.6
0.077 31.5
0.100 39.3
0.126 47.2
0.155 55.1
0.187 62.9
0.227 70.8
0.288 78.7
0.441 88.8
Page 282
Suggested Citation:"Appendix J: Design Example 8 - Embankment Over Clay Over Rock Using PileAXL." National Academies of Sciences, Engineering, and Medicine. 2024. Pile Design for Downdrag: Examples and Supporting Materials. Washington, DC: The National Academies Press. doi: 10.17226/27864.
×

Conclusion:

The influence of the hard layer that the pile was tipped into caused the neutral plane to be located closer to the tip of the pile. Inputs for the maximum end bearing resistance and maximum unit side for dolomite obtained from the Illinois Department of Transportation (2009) were input for the analyses. From the PileAXL analyses, the neutral plane was determined to occur at a depth of 43 feet, a drag load of 44 tons.

References

Budge, A.S. Dasenbrock, D.D. (2016). “The Downdrag Response of a Driven Pile Bridge Foundation in Minnesota.” Proceedings of GeoVancouver 2016, Vancouver, British Columbia, Canada, October.

Ensoft (2021). “TZPILE v. 2021”. Software Program.

Illinois Department of Transportation. (2009). “Axial Geotechnical Resistance of Driven Piles.” 10 pgs.

Innovative Geotechnics Pty Ltd. (2023). “PileAXL 2.5: A Program for Single Piles Under Axial Loading”. Software Program.

Kulhawy, F.H. and Mayne, P.W. (1990). Manual on Estimating Soil Properties for Foundation Design. Electric Power Research Institute EL-6800, Project 1493-6, Electric Power Research Institute, Palo Alto, CA

Table B.1. Statistical output for Bohn et al. (2017) Cubic Root relationship.

Averages
Bohn et al. (2017) Cubic Root Mean Bias COV Bias NRMSE
A Steel Plastic 0.97 29.84 16.7%
Granular 1.02 33.50 23.8%
Concrete Plastic 0.99 34.86 20.2%
Granular 1.00 34.55 18.1%
B Steel Plastic 1.00 30.46 21.9%
Granular 1.03 33.34 21.1%
Concrete Plastic 1.06 27.05 20.3%
Granular 1.01 31.89 19.4%

Table B.2. Statistical output for Vijayvergiya (1977) relationship.

Averages
Vijayvergiya (1977) Mean Bias COV Bias NRMSE
A Steel Plastic 0.78 34.70 23.7%
Granular 0.80 35.62 27.6%
Concrete Plastic 0.74 38.88 25.8%
Granular 0.74 36.56 24.8%
B Steel Plastic 0.83 36.83 29.2%
Granular 0.80 38.69 28.9%
Concrete Plastic 0.77 30.96 23.0%
Granular 0.76 35.75 24.7%

Table B.3. Statistical output for Bohn et al. (2017) Hyperbolic relationship.

Averages
Bohn et al. (2017) Hyperbolic Mean Bias COV Bias NRMSE
A Steel Plastic 1.80 44.35 16.4%
Granular 1.31 45.80 24.0%
Concrete Plastic 1.00 38.70 19.1%
Granular 1.10 34.67 18.1%
B Steel Plastic 2.01 48.20 23.3%
Granular 1.56 48.56 23.5%
Concrete Plastic 1.07 25.38 16.6%
Granular 1.18 37.81 17.8%

Table B.4. Statistical output for Mosher (1984) relationship.

Averages
Mosher (1984) Mean Bias COV Bias NRMSE
A Steel Plastic
Granular 2.65 46.66 25.3%
Concrete Plastic
Granular 1.46 39.85 25.5%
B Steel Plastic
Granular 2.01 48.14 29.2%
Concrete Plastic
Granular 1.40 42.98 22.2%

*Plastic soils not considered for Mosher (1984), because it was developed for granular soils.

Table B.5. Statistical output for API (2002) relationship.

Averages
API (2002) Mean Bias COV Bias NRMSE
A Steel Plastic 2.14 52.33 19.5%
Granular 1.05 50.69 31.8%
Concrete Plastic 0.95 45.97 22.7%
Granular 0.81 41.46 28.0%
B Steel Plastic 2.45 56.16 26.4%
Granular 1.14 52.78 32.4%
Concrete Plastic 0.97 29.35 17.7%
Granular 0.85 46.38 28.1%

Table B.6. Statistical output for Coyle and Reese (1966) relationship.

Averages
Coyle and Reese (1966) Mean Bias COV Bias NRMSE
A Steel Plastic 0.99 43.56 27.8%
Granular
Concrete Plastic 0.82 50.80 31.6%
Granular
B Steel Plastic 1.95 56.18 27.8%
Granular
Concrete Plastic 0.93 41.78 31.3%
Granular

*Granular soils not considered for Coyle and Reese (1966), because it was developed for plastic soils.

Depth [m] Axial Force [kN] Axial Stress [kN/m2] Soil Settlement [m] Pile Settlement [m]
0.000E+00 2.225E+03 1.534E+04 3.200E-01 9.047E-02
8.352E-01 2.240E+03 1.545E+04 2.967E-01 8.994E-02
1.670E+00 2.257E+03 1.556E+04 2.734E-01 8.940E-02
2.506E+00 2.273E+03 1.568E+04 2.502E-01 8.886E-02
3.341E+00 2.291E+03 1.580E+04 2.269E-01 8.831E-02
4.176E+00 2.309E+03 1.593E+04 2.036E-01 8.776E-02
5.011E+00 2.329E+03 1.606E+04 1.803E-01 8.721E-02
5.846E+00 2.349E+03 1.620E+04 1.571E-01 8.665E-02
6.682E+00 2.369E+03 1.634E+04 1.412E-01 8.609E-02
7.517E+00 2.391E+03 1.649E+04 1.286E-01 8.552E-02
8.352E+00 2.413E+03 1.664E+04 1.159E-01 8.494E-02
9.187E+00 2.436E+03 1.680E+04 1.035E-01 8.436E-02
1.002E+01 2.460E+03 1.696E+04 9.503E-02 8.378E-02
1.086E+01 2.484E+03 1.713E+04 8.654E-02 8.319E-02
1.169E+01 2.509E+03 1.731E+04 7.805E-02 8.259E-02
1.253E+01 2.535E+03 1.748E+04 6.968E-02 8.199E-02
1.336E+01 2.527E+03 1.743E+04 6.146E-02 8.138E-02
1.420E+01 2.500E+03 1.724E+04 5.325E-02 8.078E-02
1.503E+01 2.472E+03 1.705E+04 4.503E-02 8.019E-02
1.587E+01 2.443E+03 1.685E+04 4.053E-02 7.960E-02
1.670E+01 2.413E+03 1.664E+04 3.724E-02 7.902E-02
1.754E+01 2.382E+03 1.643E+04 3.395E-02 7.845E-02
1.837E+01 2.351E+03 1.621E+04 3.067E-02 7.788E-02
1.921E+01 2.319E+03 1.599E+04 2.738E-02 7.732E-02
2.004E+01 2.286E+03 1.577E+04 2.410E-02 7.677E-02
2.088E+01 2.253E+03 1.553E+04 2.081E-02 7.623E-02
2.172E+01 2.218E+03 1.530E+04 1.865E-02 7.570E-02
2.255E+01 2.183E+03 1.506E+04 1.787E-02 7.517E-02
2.339E+01 2.146E+03 1.480E+04 1.710E-02 7.465E-02
2.422E+01 2.107E+03 1.453E+04 1.632E-02 7.414E-02
2.506E+01 2.064E+03 1.424E+04 1.554E-02 7.365E-02
2.589E+01 2.018E+03 1.392E+04 1.477E-02 7.316E-02
2.673E+01 1.969E+03 1.358E+04 1.399E-02 7.268E-02
2.756E+01 1.917E+03 1.322E+04 1.321E-02 7.222E-02
2.840E+01 1.861E+03 1.284E+04 1.243E-02 7.177E-02
2.923E+01 1.802E+03 1.243E+04 1.166E-02 7.133E-02
3.007E+01 1.740E+03 1.200E+04 1.088E-02 7.090E-02
3.090E+01 1.675E+03 1.155E+04 1.010E-02 7.050E-02
3.174E+01 1.606E+03 1.107E+04 9.325E-03 7.010E-02
3.257E+01 1.534E+03 1.058E+04 8.548E-03 6.973E-02
3.341E+01 1.459E+03 1.006E+04 7.771E-03 6.937E-02
3.424E+01 1.380E+03 9.519E+03 6.994E-03 6.903E-02
3.508E+01 1.299E+03 8.956E+03 6.217E-03 6.871E-02
3.591E+01 1.214E+03 8.370E+03 5.440E-03 6.841E-02
3.675E+01 1.125E+03 7.762E+03 4.663E-03 6.813E-02
3.758E+01 1.034E+03 7.131E+03 3.886E-03 6.787E-02
3.842E+01 9.393E+02 6.478E+03 3.108E-03 6.764E-02
3.925E+01 8.413E+02 5.802E+03 2.331E-03 6.743E-02
4.009E+01 7.401E+02 5.104E+03 1.554E-03 6.724E-02
4.092E+01 6.357E+02 4.384E+03 7.771E-04 6.707E-02
4.176E+01 5.279E+02 3.641E+03 0.000E+00 6.693E-02

Table C3. Calculated settlement parameters.

Layer Depth [m] z [m] Thickness [m] σzo′ [kPa] α1 [rad] α2 [rad] Σ(I) ∆σ [kPa] εz δ [m]
0 - 0.8352 0.4176 0.8352 4.0465 1.4668 0.0779 0.9999 116.9912 0.0040 0.0034
0.8352 - 1.6704 1.2528 0.8352 12.1396 1.2673 0.2254 0.9981 116.7756 0.0040 0.0034
1.6704 - 2.5056 2.088 0.8352 20.2327 1.0897 0.3513 0.9919 116.0572 0.0040 0.0033
2.5056 - 3.3408 2.9232 0.8352 28.3258 0.9397 0.4504 0.9805 114.7226 0.0040 0.0033
3.3408 - 4.176 3.7584 0.8352 36.4189 0.8165 0.5236 0.9642 112.8139 0.0039 0.0033
4.176 - 5.0112 4.5936 0.8352 44.5120 0.7164 0.5748 0.9440 110.4464 0.0038 0.0032
5.0112 - 5.8464 5.4288 0.8352 52.6051 0.6350 0.6087 0.9209 107.7477 0.0037 0.0031
5.8464 - 6.6816 6.264 0.8352 60.6982 0.5683 0.6293 0.8960 104.8309 0.0036 0.0030
6.6816 - 7.5168 7.0992 0.8352 68.7912 0.5131 0.6401 0.8700 101.7873 0.0035 0.0029
7.5168 - 8.352 7.9344 0.8352 76.8843 0.4669 0.6435 0.8435 98.6867 0.0034 0.0028
8.352 - 9.1872 8.7696 0.8352 84.9774 0.4279 0.6415 0.8169 95.5815 0.0033 0.0028
9.1872 - 10.0224 9.6048 0.8352 93.0705 0.3946 0.6355 0.7907 92.5100 0.0032 0.0027
10.0224 - 10.8576 10.44 0.8352 101.1636 0.3659 0.6268 0.7650 89.4999 0.0031 0.0026
10.8576 - 11.6928 11.2752 0.8352 109.2567 0.3409 0.6160 0.7399 86.5704 0.0030 0.0025
11.6928 - 12.528 12.1104 0.8352 117.3498 0.3190 0.6039 0.7157 83.7345 0.0029 0.0024
12.528 - 13.3632 12.9456 0.8352 125.4429 0.2997 0.5909 0.6923 81.0005 0.0028 0.0023
13.3632 - 14.1984 13.7808 0.8352 133.5360 0.2825 0.5773 0.6699 78.3730 0.0027 0.0023
14.1984 - 15.0336 14.616 0.8352 141.6290 0.2671 0.5634 0.6483 75.8540 0.0026 0.0022
15.0336 - 15.8688 15.4512 0.8352 149.7221 0.2533 0.5495 0.6277 73.4433 0.0025 0.0021
15.8688 - 16.704 16.2864 0.8352 157.8152 0.2408 0.5357 0.6080 71.1395 0.0025 0.0021
16.704 - 17.5392 17.1216 0.8352 165.9083 0.2295 0.5220 0.5892 68.9400 0.0024 0.0020
17.5392 - 18.3744 17.9568 0.8352 174.0014 0.2192 0.5087 0.5713 66.8415 0.0023 0.0019
18.3744 - 19.2096 18.792 0.8352 182.0945 0.2097 0.4956 0.5542 64.8402 0.0022 0.0019
19.2096 - 20.0448 19.6272 0.8352 190.1876 0.2010 0.4829 0.5379 62.9321 0.0022 0.0018
20.0448 - 20.88 20.4624 0.8352 198.2807 0.1930 0.4706 0.5223 61.1129 0.0021 0.0018
20.88 - 21.7152 21.2976 0.8352 206.3737 0.1857 0.4587 0.5075 59.3784 0.0020 0.0017
21.7152 - 22.5504 22.1328 0.8352 214.4668 0.1788 0.4471 0.4934 57.7242 0.0020 0.0017
22.5504 - 23.3856 22.968 0.8352 222.5599 0.1724 0.4360 0.4799 56.1463 0.0019 0.0016
23.3856 - 24.2208 23.8032 0.8352 230.6530 0.1665 0.4253 0.4670 54.6405 0.0019 0.0016
24.2208 - 25.056 24.6384 0.8352 238.7461 0.1609 0.4150 0.4547 53.2030 0.0018 0.0015
25.056 - 25.8912 25.4736 0.8352 246.8392 0.1558 0.4051 0.4430 51.8301 0.0018 0.0015
25.8912 - 26.7264 26.3088 0.8352 254.9323 0.1509 0.3955 0.4318 50.5182 0.0017 0.0015
26.7264 - 27.5616 27.144 0.8352 263.0254 0.1463 0.3863 0.4211 49.2638 0.0017 0.0014
27.5616 - 28.3968 27.9792 0.8352 271.1184 0.1420 0.3775 0.4108 48.0640 0.0017 0.0014
28.3968 - 29.232 28.8144 0.8352 279.2115 0.1379 0.3689 0.4010 46.9155 0.0016 0.0014
29.232 - 30.0672 29.6496 0.8352 287.3046 0.1341 0.3608 0.3916 45.8156 0.0016 0.0013
30.0672 - 30.9024 30.4848 0.8352 295.3977 0.1305 0.3529 0.3826 44.7616 0.0015 0.0013
30.9024 - 31.7376 31.32 0.8352 303.4908 0.1270 0.3453 0.3739 43.7510 0.0015 0.0013
31.7376 - 32.5728 32.1552 0.8352 311.5839 0.1238 0.3380 0.3657 42.7814 0.0015 0.0012
32.5728 - 33.408 32.9904 0.8352 319.6770 0.1207 0.3309 0.3577 41.8506 0.0014 0.0012
33.408 - 34.2432 33.8256 0.8352 327.7701 0.1177 0.3241 0.3501 40.9566 0.0014 0.0012
34.2432 - 35.0784 34.6608 0.8352 335.8632 0.1149 0.3176 0.3427 40.0973 0.0014 0.0012
35.0784 - 35.9136 35.496 0.8352 343.9562 0.1122 0.3113 0.3356 39.2710 0.0014 0.0011
35.9136 - 36.7488 36.3312 0.8352 352.0493 0.1097 0.3052 0.3289 38.4759 0.0013 0.0011
36.7488 - 37.584 37.1664 0.8352 360.1424 0.1072 0.2993 0.3223 37.7104 0.0013 0.0011
37.584 - 38.4192 38.0016 0.8352 368.2355 0.1049 0.2936 0.3160 36.9730 0.0013 0.0011
38.4192 - 39.2544 38.8368 0.8352 376.3286 0.1026 0.2882 0.3099 36.2624 0.0013 0.0010
39.2544 - 40.0896 39.672 0.8352 384.4217 0.1005 0.2829 0.3041 35.5770 0.0012 0.0010
40.0896 - 40.9248 40.5072 0.8352 392.5148 0.0984 0.2778 0.2984 34.9158 0.0012 0.0010
40.9248 - 41.76 41.3424 0.8352 400.6079 0.0965 0.2728 0.2930 34.2774 0.0012 0.0010
z = layer midpoint depth, presentationzo′ = vertical effective stress, presentation1 and presentation2 = angles (in radians) as shown in Figure C6, presentationz = vertical consolidation strain, presentation = vertical settlement of individual sublayer; presentation (from bottom to top) = settlement profile (Figure C5).

Table C4. Calculated pile side resistance parameters.

Layer Depth z Thickness σzo su α fn Fs
[m] [m] [m] [kPa] [kPa] [kPa] [kN]
0 - 0.8352 0.4176 0.8352 4.0465 13.29 0.35 0.00 0.00
0.8352 - 1.6704 1.2528 0.8352 12.1396 13.95 0.45 6.32 7.34
1.6704 - 2.5056 2.088 0.8352 20.2327 14.62 0.55 8.07 9.37
2.5056 - 3.3408 2.9232 0.8352 28.3258 15.29 0.64 9.76 11.33
3.3408 - 4.176 3.7584 0.8352 36.4189 15.95 0.71 11.31 13.13
4.176 - 5.0112 4.5936 0.8352 44.5120 16.62 0.77 12.76 14.81
5.0112 - 5.8464 5.4288 0.8352 52.6051 17.29 0.82 14.14 16.42
5.8464 - 6.6816 6.264 0.8352 60.6982 17.95 0.86 15.48 17.98
6.6816 - 7.5168 7.0992 0.8352 68.7912 18.62 0.90 16.79 19.49
7.5168 - 8.352 7.9344 0.8352 76.8843 19.29 0.94 18.06 20.97
8.352 - 9.1872 8.7696 0.8352 84.9774 19.95 0.97 19.31 22.42
9.1872 - 10.0224 9.6048 0.8352 93.0705 20.62 1.00 20.55 23.86
10.0224 - 10.8576 10.44 0.8352 101.1636 21.29 1.02 21.77 25.27
10.8576 - 11.6928 11.2752 0.8352 109.2567 21.95 1.05 22.97 26.67
11.6928 - 12.528 12.1104 0.8352 117.3498 22.62 1.07 24.17 28.06
12.528 - 13.3632 12.9456 0.8352 125.4429 23.29 1.09 25.35 29.43
13.3632 - 14.1984 13.7808 0.8352 133.5360 23.95 1.11 26.53 30.80
14.1984 - 15.0336 14.616 0.8352 141.6290 24.62 1.12 27.70 32.16
15.0336 - 15.8688 15.4512 0.8352 149.7221 25.29 1.14 28.86 33.51
15.8688 - 16.704 16.2864 0.8352 157.8152 25.95 1.16 30.02 34.85
16.704 - 17.5392 17.1216 0.8352 165.9083 26.62 1.17 31.17 36.19
17.5392 - 18.3744 17.9568 0.8352 174.0014 27.29 1.18 32.32 37.52
18.3744 - 19.2096 18.792 0.8352 182.0945 27.95 1.20 33.46 38.85
19.2096 - 20.0448 19.6272 0.8352 190.1876 28.62 1.21 34.61 40.17
20.0448 - 20.88 20.4624 0.8352 198.2807 29.29 1.22 35.74 41.50
20.88 - 21.7152 21.2976 0.8352 206.3737 29.95 1.23 36.88 42.81
21.7152 - 22.5504 22.1328 0.8352 214.4668 30.62 1.24 38.01 44.13
22.5504 - 23.3856 22.968 0.8352 222.5599 31.76 1.24 39.43 45.78
23.3856 - 24.2208 23.8032 0.8352 230.6530 36.05 1.19 42.77 49.66
24.2208 - 25.056 24.6384 0.8352 238.7461 40.35 1.14 46.04 53.45
25.056 - 25.8912 25.4736 0.8352 246.8392 44.65 1.10 49.24 57.16
25.8912 - 26.7264 26.3088 0.8352 254.9323 48.94 1.07 52.39 60.82
26.7264 - 27.5616 27.144 0.8352 263.0254 53.24 1.04 55.51 64.44
27.5616 - 28.3968 27.9792 0.8352 271.1184 57.54 1.02 58.58 68.01
28.3968 - 29.232 28.8144 0.8352 279.2115 61.84 1.00 61.63 71.55
29.232 - 30.0672 29.6496 0.8352 287.3046 66.13 0.98 64.65 75.06
30.0672 - 30.9024 30.4848 0.8352 295.3977 70.43 0.96 67.65 78.54
30.9024 - 31.7376 31.32 0.8352 303.4908 74.73 0.95 70.63 82.00
31.7376 - 32.5728 32.1552 0.8352 311.5839 79.02 0.93 73.60 85.44
32.5728 - 33.408 32.9904 0.8352 319.6770 83.32 0.92 76.55 88.87
33.408 - 34.2432 33.8256 0.8352 327.7701 87.62 0.91 79.49 92.28
34.2432 - 35.0784 34.6608 0.8352 335.8632 91.91 0.90 82.41 95.67
35.0784 - 35.9136 35.496 0.8352 343.9562 96.21 0.89 85.32 99.06
35.9136 - 36.7488 36.3312 0.8352 352.0493 100.51 0.88 88.23 102.43
36.7488 - 37.584 37.1664 0.8352 360.1424 104.80 0.87 91.12 105.79
37.584 - 38.4192 38.0016 0.8352 368.2355 109.10 0.86 94.01 109.14
38.4192 - 39.2544 38.8368 0.8352 376.3286 113.40 0.85 96.89 112.49
39.2544 - 40.0896 39.672 0.8352 384.4217 117.69 0.85 99.77 115.82
40.0896 - 40.9248 40.5072 0.8352 392.5148 121.99 0.84 102.64 119.15
40.9248 - 41.76 41.3424 0.8352 400.6079 126.29 0.84 105.50 122.48
z = layer midpoint depth, σzo′ = vertical effective stress, su = undrained shear strength, α = total stress side resistance value, fn=nominal unit side resistance, Fs=sublayer side resistance. Note: fn neglected for top 1.5m

Table C5. Calculated load as a function of depth.

Layer Depth z QAD QwUTL
[m] [m] [kN] [kN]
0 - 0.8352 0.4176 0.00 2225.00
0.8352 - 1.6704 1.2528 7.34 2232.34
1.6704 - 2.5056 2.088 16.70 2241.70
2.5056 - 3.3408 2.9232 28.04 2253.04
3.3408 - 4.176 3.7584 41.16 2266.16
4.176 - 5.0112 4.5936 55.97 2280.97
5.0112 - 5.8464 5.4288 72.39 2297.39
5.8464 - 6.6816 6.264 90.37 2315.37
6.6816 - 7.5168 7.0992 109.86 2334.86
7.5168 - 8.352 7.9344 130.83 2355.83
8.352 - 9.1872 8.7696 153.25 2378.25
9.1872 - 10.0224 9.6048 177.11 2402.11
10.0224 - 10.8576 10.44 202.38 2427.38
10.8576 - 11.6928 11.2752 229.04 2454.04
11.6928 - 12.528 12.1104 257.10 2482.10
12.528 - 13.3632 12.9456 286.53 2511.53
13.3632 - 14.1984 13.7808 317.33 2542.33
14.1984 - 15.0336 14.616 349.48 2574.48
15.0336 - 15.8688 15.4512 382.99 2607.99
15.8688 - 16.704 16.2864 417.84 2642.84
16.704 - 17.5392 17.1216 454.03 2679.03
17.5392 - 18.3744 17.9568 491.55 2716.55
18.3744 - 19.2096 18.792 530.40 2755.40
19.2096 - 20.0448 19.6272 570.57 2795.57
20.0448 - 20.88 20.4624 612.07 2837.07
20.88 - 21.7152 21.2976 654.88 2879.88
21.7152 - 22.5504 22.1328 699.01 2924.01
22.5504 - 23.3856 22.968 744.79 2969.79
23.3856 - 24.2208 23.8032 794.44 3019.44
24.2208 - 25.056 24.6384 847.89 3072.89
25.056 - 25.8912 25.4736 905.05 3130.05
25.8912 - 26.7264 26.3088 965.88 3190.88
26.7264 - 27.5616 27.144 1030.32 3255.32
27.5616 - 28.3968 27.9792 1098.33 3323.33
28.3968 - 29.232 28.8144 1169.88 3394.88
29.232 - 30.0672 29.6496 1244.93 3469.93
30.0672 - 30.9024 30.4848 1323.47 3548.47
30.9024 - 31.7376 31.32 1405.48 3630.48
31.7376 - 32.5728 32.1552 1490.92 3715.92
32.5728 - 33.408 32.9904 1579.79 3804.79
33.408 - 34.2432 33.8256 1672.06 3897.06
34.2432 - 35.0784 34.6608 1767.74 3992.74
35.0784 - 35.9136 35.496 1866.79 4091.79
35.9136 - 36.7488 36.3312 1969.22 4194.22
36.7488 - 37.584 37.1664 2075.01 4300.01
37.584 - 38.4192 38.0016 2184.15 4409.15
38.4192 - 39.2544 38.8368 2296.64 4521.64
39.2544 - 40.0896 39.672 2412.46 4637.46
40.0896 - 40.9248 40.5072 2531.61 4756.61
40.9248 - 41.76 41.3424 2654.09 4879.09
QAD= load in pile after driving,
QwUTL= load in pile with addition of unfactored top load

Table C6. Calculated resistance as a function of depth.

Layer Depth z R R= resistance from soil surrounding pile; resistance values are calculated at the top of each sublayer; the bottom sublayer includes the summation of the end bearing resistance and the side resistance in the bottom sublayer.
[m] [m] [kN]
0 - 0.8352 0.4176 2821.70
0.8352 - 1.6704 1.2528 2821.70
1.6704 - 2.5056 2.088 2814.36
2.5056 - 3.3408 2.9232 2805.00
3.3408 - 4.176 3.7584 2793.66
4.176 - 5.0112 4.5936 2780.54
5.0112 - 5.8464 5.4288 2765.73
5.8464 - 6.6816 6.264 2749.31
6.6816 - 7.5168 7.0992 2731.33
7.5168 - 8.352 7.9344 2711.84
8.352 - 9.1872 8.7696 2690.87
9.1872 - 10.0224 9.6048 2668.45
10.0224 - 10.8576 10.44 2644.59
10.8576 - 11.6928 11.2752 2619.32
11.6928 - 12.528 12.1104 2592.65
12.528 - 13.3632 12.9456 2564.60
13.3632 - 14.1984 13.7808 2535.17
14.1984 - 15.0336 14.616 2504.37
15.0336 - 15.8688 15.4512 2472.22
15.8688 - 16.704 16.2864 2438.71
16.704 - 17.5392 17.1216 2403.86
17.5392 - 18.3744 17.9568 2367.67
18.3744 - 19.2096 18.792 2330.15
19.2096 - 20.0448 19.6272 2291.30
20.0448 - 20.88 20.4624 2251.13
20.88 - 21.7152 21.2976 2209.63
21.7152 - 22.5504 22.1328 2166.82
22.5504 - 23.3856 22.968 2122.69
23.3856 - 24.2208 23.8032 2076.91
24.2208 - 25.056 24.6384 2027.26
25.056 - 25.8912 25.4736 1973.81
25.8912 - 26.7264 26.3088 1916.65
26.7264 - 27.5616 27.144 1855.82
27.5616 - 28.3968 27.9792 1791.38
28.3968 - 29.232 28.8144 1723.37
29.232 - 30.0672 29.6496 1651.82
30.0672 - 30.9024 30.4848 1576.77
30.9024 - 31.7376 31.32 1498.23
31.7376 - 32.5728 32.1552 1416.22
32.5728 - 33.408 32.9904 1330.78
33.408 - 34.2432 33.8256 1241.91
34.2432 - 35.0784 34.6608 1149.64
35.0784 - 35.9136 35.496 1053.96
35.9136 - 36.7488 36.3312 954.91
36.7488 - 37.584 37.1664 852.48
37.584 - 38.4192 38.0016 746.69
38.4192 - 39.2544 38.8368 637.55
39.2544 - 40.0896 39.672 525.06
40.0896 - 40.9248 40.5072 409.24
40.9248 - 41.76 41.3424 290.09

Table C7. Calculated elastic compression in the pile as a function of depth.

z M i n [ Q R ] δEC,s δEC Comments:
z=depth,
Q=load,
R=resistance,
δEC,s=segmental elastic compression,
δEC=cumulative elastic compression (from bottom to top),
A=pile cross-sectional area=0.145[m2]
Ep=pile elastic modulus=2.41E+07[kPa]
Ls=length of each pile segment=0.8532[m]
[m] [kN] [m] [m]
0.4176 2225.00 0.0005 0.0225
1.2528 2232.34 0.0005 0.0220
2.088 2241.70 0.0005 0.0215
2.9232 2253.04 0.0005 0.0209
3.7584 2266.16 0.0005 0.0204
4.5936 2280.97 0.0005 0.0198
5.4288 2297.39 0.0005 0.0193
6.264 2315.37 0.0006 0.0188
7.0992 2334.86 0.0006 0.0182
7.9344 2355.83 0.0006 0.0176
8.7696 2378.25 0.0006 0.0171
9.6048 2402.11 0.0006 0.0165
10.44 2427.38 0.0006 0.0159
11.2752 2454.04 0.0006 0.0154
12.1104 2482.10 0.0006 0.0148
12.9456 2511.53 0.0006 0.0142
13.7808 2535.17 0.0006 0.0136
14.616 2504.37 0.0006 0.0130
15.4512 2472.22 0.0006 0.0124
16.2864 2438.71 0.0006 0.0118
17.1216 2403.86 0.0006 0.0112
17.9568 2367.67 0.0006 0.0106
18.792 2330.15 0.0006 0.0101
19.6272 2291.30 0.0005 0.0095
20.4624 2251.13 0.0005 0.0090
21.2976 2209.63 0.0005 0.0084
22.1328 2166.82 0.0005 0.0079
22.968 2122.69 0.0005 0.0074
23.8032 2076.91 0.0005 0.0069
24.6384 2027.26 0.0005 0.0064
25.4736 1973.81 0.0005 0.0059
26.3088 1916.65 0.0005 0.0054
27.144 1855.82 0.0004 0.0049
27.9792 1791.38 0.0004 0.0045
28.8144 1723.37 0.0004 0.0041
29.6496 1651.82 0.0004 0.0037
30.4848 1576.77 0.0004 0.0033
31.32 1498.23 0.0004 0.0029
32.1552 1416.22 0.0003 0.0025
32.9904 1330.78 0.0003 0.0022
33.8256 1241.91 0.0003 0.0019
34.6608 1149.64 0.0003 0.0016
35.496 1053.96 0.0003 0.0013
36.3312 954.91 0.0002 0.0011
37.1664 852.48 0.0002 0.0008
38.0016 746.69 0.0002 0.0006
38.8368 637.55 0.0002 0.0004
39.672 525.06 0.0001 0.0003
40.5072 409.24 0.0001 0.0002
41.3424 290.09 0.0001 0.0001

Table C8. Calculated load-displacement values for the octagonal 419mm diameter by 41.76m long precast concrete pile.

δs, δb Rs Rt RT Comments:
δs=side displacement,
δt=toe displacement,
Rs=side resistance,
Rt=end bearing resistance,
RT=total resistance,
Ks=side flexibility factor = 1.58E-7[m/kN],
Kt=toe flexibility factor =1.00E-5[m/kN],
Ms=0.001,
Rsu=ultimate side resistance=2654[kN],
Rtu=ultimate toe resistance=174.6[kN],
Rt,10%=nominal net end bearing esistance=167.6[kN],
D=diameter=419[mm],
su=undrained shear strength=128.44[kPa],
Es,25=128440[kPa],
At=pile toe area=0.145[m2],
Ep=pile modulus= 2.41E+07[kPa],
L=pile length=41.76[m]
[m] [kN] [kN] [kN]
0 0.00 0.00 0.00
0.0005 1443.96 38.77 1482.73
0.001 1870.33 63.45 1933.78
0.0015 2074.52 80.54 2155.06
0.002 2194.30 93.08 2287.37
0.0025 2273.04 102.67 2375.70
0.003 2328.75 110.24 2438.99
0.0035 2370.25 116.37 2486.61
0.004 2402.35 121.43 2523.78
0.0045 2427.93 125.68 2553.61
0.005 2448.79 129.31 2578.10
0.0055 2466.13 132.43 2598.56
0.006 2480.76 135.15 2615.91
0.0065 2493.28 137.54 2630.82
0.007 2504.11 139.66 2643.77
0.0075 2513.57 141.55 2655.13
0.008 2521.91 143.25 2665.16
0.0085 2529.32 144.78 2674.10
0.009 2535.94 146.16 2682.10
0.0095 2541.89 147.43 2689.32
0.01 2547.27 148.58 2695.85
0.015 2581.88 156.36 2738.23
0.02 2599.54 160.55 2760.09
0.025 2610.25 163.18 2773.43
0.03 2617.44 164.98 2782.43
0.035 2622.60 166.29 2788.90
0.04 2626.49 167.29 2793.78
0.045 2629.52 168.07 2797.59
0.05 2631.94 168.71 2800.65

Table C9. Soil settlement and pile settlement as a function of depth.

z δp δs Comments:
z=depth,
Q=load,
R=resistance,
δp=pile settlement,
δs=soil settlement,
A=pile cross-sectional area=0.145[m2],
Ep=pile elastic modulus=2.41E+07[kPa]
Ls=length of each pile
segment=0.8532[m]
[m] [m] [m]
0.4176 0.0635 0.0972
1.2528 0.0630 0.0939
2.088 0.0625 0.0905
2.9232 0.0619 0.0872
3.7584 0.0614 0.0839
4.5936 0.0608 0.0806
5.4288 0.0603 0.0774
6.264 0.0598 0.0743
7.0992 0.0592 0.0713
7.9344 0.0586 0.0684
8.7696 0.0581 0.0655
9.6048 0.0575 0.0628
10.44 0.0569 0.0601
11.2752 0.0564 0.0575
12.1104 0.0558 0.0550
12.9456 0.0552 0.0526
13.7808 0.0546 0.0503
14.616 0.0540 0.0480
15.4512 0.0534 0.0458
16.2864 0.0528 0.0437
17.1216 0.0522 0.0417
17.9568 0.0516 0.0397
18.792 0.0511 0.0378
19.6272 0.0505 0.0359
20.4624 0.0500 0.0341
21.2976 0.0494 0.0323
22.1328 0.0489 0.0306
22.968 0.0484 0.0289
23.8032 0.0479 0.0273
24.6384 0.0474 0.0257
25.4736 0.0469 0.0242
26.3088 0.0464 0.0227
27.144 0.0459 0.0213
27.9792 0.0455 0.0198
28.8144 0.0451 0.0185
29.6496 0.0447 0.0171
30.4848 0.0443 0.0158
31.32 0.0439 0.0145
32.1552 0.0435 0.0132
32.9904 0.0432 0.0120
33.8256 0.0429 0.0108
34.6608 0.0426 0.0096
35.496 0.0423 0.0085
36.3312 0.0421 0.0073
37.1664 0.0418 0.0062
38.0016 0.0416 0.0051
38.8368 0.0414 0.0041
39.672 0.0413 0.0030
40.5072 0.0412 0.0020
41.3424 0.0411 0.0010

Table D2. Parameters used in Define – Soil Layers – Edit window.

z su σz,o ψ α fs fb
[m] [kPa] [kPa] [rad] [rad] [kPa] [kPa]
0.00 13 0.00 13415.89 0.05 1 117
22.86 31 221.51 0.14 1.34 42 279
41.76 128 404.65 0.32 0.89 114 1152

Table D3. Useful PileAXL output and corresponding calculated values using Microsoft Excel.

Depth ULS fs ULS Qs ULS Qb Qult ∆Q Q R min(Q,R) Comments:

ULS fs = Ultimate unit side resistance (PileAXL output),
ULS Qs = Ultimate total side resistance (PileAXL output),
ULS Qb = Ultimate bearing resistance (PileAXL output),
Qult = Ultimate total resistance (PileAXL output),
∆Q = incremental side resistance (Excel calculation),
Q = cumulative load in the pile (Excel calculation),
R= resistance from soil surrounding pile; resistance values are calculated at the top of each sublayer (Excel calculation),
min(Q,R) = minimum of Q and R for each depth (Excel calculation).

[m] [kPa] [kN] [kN] [kN] [kN] [kN] [kN] [kN]
0.000 2.879 0.000 17.057 17.057 5.132 2225.000 2800.163 2225.000
0.835 5.962 5.132 17.920 23.052 7.836 2230.132 2795.031 2230.132
1.670 7.536 12.968 18.783 31.750 9.854 2237.968 2787.196 2237.968
2.506 9.439 22.822 19.646 42.467 11.944 2247.822 2777.342 2247.822
3.341 11.136 34.766 20.509 55.275 13.843 2259.766 2765.397 2259.766
4.176 12.710 48.609 21.371 69.981 15.622 2273.609 2751.554 2273.609
5.011 14.202 64.231 22.234 86.466 17.320 2289.231 2735.932 2289.231
5.846 15.634 81.551 23.097 104.648 18.958 2306.551 2718.612 2306.551
6.682 17.023 100.509 23.960 124.469 20.550 2325.509 2699.654 2325.509
7.517 18.378 121.059 24.823 145.882 22.033 2346.059 2679.104 2346.059
8.352 19.576 143.092 25.686 168.778 23.110 2368.092 2657.071 2368.092
9.187 20.234 166.202 26.549 192.751 23.874 2391.202 2633.961 2391.202
10.022 20.892 190.076 27.412 217.487 24.637 2415.076 2610.088 2415.076
10.858 21.549 214.713 28.274 242.987 25.401 2439.713 2585.45 2439.713
11.693 22.207 240.114 29.137 269.251 26.164 2465.114 2560.05 2465.114
12.528 22.865 266.278 30.000 296.278 26.928 2491.278 2533.885 2491.278
13.363 23.522 293.206 30.863 324.069 27.691 2518.206 2506.958 2506.958
14.198 24.180 320.897 31.726 352.623 28.455 2545.897 2479.266 2479.266
15.034 24.838 349.352 32.589 381.941 29.218 2574.352 2450.812 2450.812
15.869 25.495 378.570 33.452 412.022 29.982 2603.570 2421.593 2421.593
16.704 26.153 408.552 34.315 442.867 30.745 2633.552 2391.611 2391.611
17.539 26.810 439.298 35.177 474.475 31.509 2664.298 2360.866 2360.866
18.374 27.468 470.806 36.040 506.847 32.272 2695.806 2329.357 2329.357
19.210 28.126 503.079 36.903 539.982 33.036 2728.079 2297.085 2297.085
20.045 28.783 536.115 37.766 573.881 33.800 2761.115 2264.049 2264.049
20.880 29.441 569.914 38.629 608.543 34.563 2794.914 2230.249 2230.249
21.715 30.099 604.477 39.492 643.969 35.327 2829.477 2195.686 2195.686
22.550 30.756 639.804 40.355 680.159 37.415 2864.804 2160.359 2160.359
23.386 33.696 677.219 44.212 721.431 41.609 2902.219 2122.944 2122.944
24.221 37.981 718.828 49.834 768.662 46.584 2943.828 2081.335 2081.335
25.056 42.266 765.412 55.456 820.867 51.558 2990.412 2034.752 2034.752
25.891 46.550 816.970 61.078 878.047 56.532 3041.970 1983.194 1983.194
26.726 50.835 873.502 66.699 940.201 61.507 3098.502 1926.661 1926.661
27.562 55.119 935.009 72.321 1007.330 66.481 3160.009 1865.155 1865.155
28.397 59.404 1001.490 77.943 1079.433 71.456 3226.490 1798.673 1798.673
29.232 63.688 1072.946 83.564 1156.510 76.430 3297.946 1727.218 1727.218
30.067 67.973 1149.376 89.186 1238.562 81.405 3374.376 1650.787 1650.787
30.902 72.258 1230.781 94.808 1325.588 86.039 3455.781 1569.383 1569.383
31.738 75.957 1316.820 100.429 1417.250 89.997 3541.820 1483.343 1483.343
32.573 79.075 1406.817 106.051 1512.868 93.608 3631.817 1393.346 1393.346
33.408 82.177 1500.425 111.673 1612.097 97.202 3725.425 1299.739 1299.739
34.243 85.266 1597.626 117.295 1714.921 100.782 3822.626 1202.537 1202.537
35.078 88.344 1698.408 122.916 1821.324 104.349 3923.408 1101.755 1101.755
35.914 91.411 1802.757 128.538 1931.295 107.904 4027.757 997.4067 997.4067
36.749 94.468 1910.660 134.160 2044.820 111.448 4135.660 889.5031 889.5031
37.584 97.516 2022.108 139.781 2161.890 114.983 4247.108 778.055 778.055
38.419 100.557 2137.091 145.403 2282.494 118.509 4362.091 663.0722 663.0722
39.254 103.591 2255.600 151.025 2406.625 122.027 4480.600 544.5633 544.5633
40.090 106.617 2377.627 156.647 2534.273 125.538 4602.627 422.5365 422.5365
40.925 109.638 2503.164 162.268 2665.433 129.053 4728.164 296.9989 296.9989
41.760 112.672 2632.217 167.946 2800.163 - 4857.217 167.9464 167.9464

Table D4. Calculated elastic compression values.

z M i n [ Q R ] δEC,s δEC Comments:
z=depth,
Q=load, R=resistance,
δEC,s=segmental elastic compression (from bottom to top),
δEC=cumulative elastic compression,
A=pile cross-sectional area=0.145[m2]
Ep=pile elastic modulus=2.41E+07[kPa]
Ls=length of each pile segment=0.8532[m]
[m] [kN] [m] [m]
0.000 2225.000 - -
0.835 2230.132 0.00053 0.02227
1.670 2237.968 0.00053 0.02174
2.506 2247.822 0.00054 0.02120
3.341 2259.766 0.00054 0.02066
4.176 2273.609 0.00054 0.02012
5.011 2289.231 0.00055 0.01958
5.846 2306.551 0.00055 0.01903
6.682 2325.509 0.00056 0.01848
7.517 2346.059 0.00056 0.01793
8.352 2368.092 0.00057 0.01737
9.187 2391.202 0.00057 0.01680
10.022 2415.076 0.00058 0.01623
10.858 2439.713 0.00058 0.01565
11.693 2465.114 0.00059 0.01507
12.528 2491.278 0.00060 0.01448
13.363 2506.958 0.00060 0.01388
14.198 2479.266 0.00059 0.01328
15.034 2450.812 0.00059 0.01269
15.869 2421.593 0.00058 0.01211
16.704 2391.611 0.00057 0.01153
17.539 2360.866 0.00056 0.01096
18.374 2329.357 0.00056 0.01039
19.210 2297.085 0.00055 0.00983
20.045 2264.049 0.00054 0.00929
20.880 2230.249 0.00053 0.00874
21.715 2195.686 0.00052 0.00821
22.550 2160.359 0.00052 0.00769
23.386 2122.944 0.00051 0.00717
24.221 2081.335 0.00050 0.00666
25.056 2034.752 0.00049 0.00617
25.891 1983.194 0.00047 0.00568
26.726 1926.661 0.00046 0.00521
27.562 1865.155 0.00045 0.00474
28.397 1798.673 0.00043 0.00430
29.232 1727.218 0.00041 0.00387
30.067 1650.787 0.00039 0.00346
30.902 1569.383 0.00038 0.00306
31.738 1483.343 0.00035 0.00269
32.573 1393.346 0.00033 0.00233
33.408 1299.739 0.00031 0.00200
34.243 1202.537 0.00029 0.00169
35.078 1101.755 0.00026 0.00140
35.914 997.4067 0.00024 0.00114
36.749 889.5031 0.00021 0.00090
37.584 778.055 0.00019 0.00069
38.419 663.0722 0.00016 0.00050
39.254 544.5633 0.00013 0.00034
40.090 422.5365 0.00010 0.00021
40.925 296.9989 0.00007 0.00011
41.760 167.9464 0.00004 0.00004

Table D5. Soil settlement and pile settlement as a function of depth.

zp δp Zs δs Comments:
zp=depth for pile calculations,
zs=depth for pile calculations,
δp=pile settlement,
δs=soil settlement.
[m] [m] [m] [m]
0.000 0.0632 0.4176 0.0972
0.835 0.0627 1.2528 0.0939
1.670 0.0622 2.088 0.0905
2.506 0.0616 2.9232 0.0872
3.341 0.0611 3.7584 0.0839
4.176 0.0605 4.5936 0.0806
5.011 0.0600 5.4288 0.0774
5.846 0.0594 6.264 0.0743
6.682 0.0589 7.0992 0.0713
7.517 0.0583 7.9344 0.0684
8.352 0.0578 8.7696 0.0655
9.187 0.0572 9.6048 0.0628
10.022 0.0566 10.44 0.0601
10.858 0.0560 11.2752 0.0575
11.693 0.0554 12.1104 0.0550
12.528 0.0548 12.9456 0.0526
13.363 0.0542 13.7808 0.0503
14.198 0.0536 14.616 0.0480
15.034 0.0531 15.4512 0.0458
15.869 0.0525 16.2864 0.0437
16.704 0.0519 17.1216 0.0417
17.539 0.0513 17.9568 0.0397
18.374 0.0508 18.792 0.0378
19.210 0.0502 19.6272 0.0359
20.045 0.0497 20.4624 0.0341
20.880 0.0492 21.2976 0.0323
21.715 0.0486 22.1328 0.0306
22.550 0.0481 22.968 0.0289
23.386 0.0476 23.8032 0.0273
24.221 0.0471 24.6384 0.0257
25.056 0.0466 25.4736 0.0242
25.891 0.0462 26.3088 0.0227
26.726 0.0457 27.144 0.0213
27.562 0.0453 27.9792 0.0198
28.397 0.0448 28.8144 0.0185
29.232 0.0444 29.6496 0.0171
30.067 0.0440 30.4848 0.0158
30.902 0.0436 31.32 0.0145
31.738 0.0433 32.1552 0.0132
32.573 0.0430 32.9904 0.0120
33.408 0.0426 33.8256 0.0108
34.243 0.0424 34.6608 0.0096
35.078 0.0421 35.496 0.0085
35.914 0.0419 36.3312 0.0073
36.749 0.0416 37.1664 0.0062
37.584 0.0415 38.0016 0.0051
38.419 0.0413 38.8368 0.0041
39.254 0.0412 39.672 0.0030
40.090 0.0411 40.5072 0.0020
40.925 0.0410 41.3424 0.0010
41.760 0.0410 - -

Table E2. Calculated soil settlement parameters.

Layer Depth [m] z [m] Thickness [m] σz,o′ [kPa] α1 [rad] α2 [rad] Σ(I) ∆σ [kPa] εz δ [m]
0 - 0.8352 0.4176 0.8352 4.0465 1.4668 0.0779 0.9999 116.9912 0.0040 0.0034
0.8352 - 1.6704 1.2528 0.8352 12.1396 1.2673 0.2254 0.9981 116.7756 0.0040 0.0034
1.6704 - 2.5056 2.088 0.8352 20.2327 1.0897 0.3513 0.9919 116.0572 0.0040 0.0033
2.5056 - 3.3408 2.9232 0.8352 28.3258 0.9397 0.4504 0.9805 114.7226 0.0040 0.0033
3.3408 - 4.176 3.7584 0.8352 36.4189 0.8165 0.5236 0.9642 112.8139 0.0039 0.0033
4.176 - 5.0112 4.5936 0.8352 44.5120 0.7164 0.5748 0.9440 110.4464 0.0038 0.0032
5.0112 - 5.8464 5.4288 0.8352 52.6051 0.6350 0.6087 0.9209 107.7477 0.0037 0.0031
5.8464 - 6.6816 6.264 0.8352 60.6982 0.5683 0.6293 0.8960 104.8309 0.0036 0.0030
6.6816 - 7.5168 7.0992 0.8352 68.7912 0.5131 0.6401 0.8700 101.7873 0.0035 0.0029
7.5168 - 8.352 7.9344 0.8352 76.8843 0.4669 0.6435 0.8435 98.6867 0.0034 0.0028
8.352 - 9.1872 8.7696 0.8352 84.9774 0.4279 0.6415 0.8169 95.5815 0.0033 0.0028
9.1872 - 10.0224 9.6048 0.8352 93.0705 0.3946 0.6355 0.7907 92.5100 0.0032 0.0027
10.0224 - 10.8576 10.44 0.8352 101.1636 0.3659 0.6268 0.7650 89.4999 0.0031 0.0026
10.8576 - 11.6928 11.2752 0.8352 109.2567 0.3409 0.6160 0.7399 86.5704 0.0030 0.0025
11.6928 - 12.528 12.1104 0.8352 117.3498 0.3190 0.6039 0.7157 83.7345 0.0029 0.0024
12.528 - 13.3632 12.9456 0.8352 125.4429 0.2997 0.5909 0.6923 81.0005 0.0028 0.0023
13.3632 - 14.1984 13.7808 0.8352 133.5360 0.2825 0.5773 0.6699 78.3730 0.0027 0.0023
14.1984 - 15.0336 14.616 0.8352 141.6290 0.2671 0.5634 0.6483 75.8540 0.0026 0.0022
15.0336 - 15.8688 15.4512 0.8352 149.7221 0.2533 0.5495 0.6277 73.4433 0.0025 0.0021
15.8688 - 16.704 16.2864 0.8352 157.8152 0.2408 0.5357 0.6080 71.1395 0.0025 0.0021
16.704 - 17.5392 17.1216 0.8352 165.9083 0.2295 0.5220 0.5892 68.9400 0.0024 0.0020
17.5392 - 18.3744 17.9568 0.8352 174.0014 0.2192 0.5087 0.5713 66.8415 0.0023 0.0019
18.3744 - 19.2096 18.792 0.8352 182.0945 0.2097 0.4956 0.5542 64.8402 0.0022 0.0019
19.2096 - 20.0448 19.6272 0.8352 190.1876 0.2010 0.4829 0.5379 62.9321 0.0022 0.0018
20.0448 - 20.88 20.4624 0.8352 198.2807 0.1930 0.4706 0.5223 61.1129 0.0021 0.0018
20.88 - 21.7152 21.2976 0.8352 206.3737 0.1857 0.4587 0.5075 59.3784 0.0020 0.0017
21.7152 - 22.5504 22.1328 0.8352 214.4668 0.1788 0.4471 0.4934 57.7242 0.0020 0.0017
22.5504 - 23.3856 22.968 0.8352 222.5599 0.1724 0.4360 0.4799 56.1463 0.0019 0.0016
23.3856 - 24.2208 23.8032 0.8352 230.6530 0.1665 0.4253 0.4670 54.6405 0.0019 0.0016
24.2208 - 25.056 24.6384 0.8352 238.7461 0.1609 0.4150 0.4547 53.2030 0.0018 0.0015
25.056 - 25.8912 25.4736 0.8352 246.8392 0.1558 0.4051 0.4430 51.8301 0.0018 0.0015
25.8912 - 26.7264 26.3088 0.8352 254.9323 0.1509 0.3955 0.4318 50.5182 0.0017 0.0015
26.7264 - 27.5616 27.144 0.8352 263.0254 0.1463 0.3863 0.4211 49.2638 0.0017 0.0014
27.5616 - 28.3968 27.9792 0.8352 271.1184 0.1420 0.3775 0.4108 48.0640 0.0017 0.0014
28.3968 - 29.232 28.8144 0.8352 279.2115 0.1379 0.3689 0.4010 46.9155 0.0016 0.0014
29.232 - 30.0672 29.6496 0.8352 287.3046 0.1341 0.3608 0.3916 45.8156 0.0016 0.0013
30.0672 - 30.9024 30.4848 0.8352 295.3977 0.1305 0.3529 0.3826 44.7616 0.0015 0.0013
30.9024 - 31.7376 31.32 0.8352 303.4908 0.1270 0.3453 0.3739 43.7510 0.0015 0.0013
31.7376 - 32.5728 32.1552 0.8352 311.5839 0.1238 0.3380 0.3657 42.7814 0.0015 0.0012
32.5728 - 33.408 32.9904 0.8352 319.6770 0.1207 0.3309 0.3577 41.8506 0.0014 0.0012
33.408 - 34.2432 33.8256 0.8352 327.7701 0.1177 0.3241 0.3501 40.9566 0.0014 0.0012
34.2432 - 35.0784 34.6608 0.8352 335.8632 0.1149 0.3176 0.3427 40.0973 0.0014 0.0012
35.0784 - 35.9136 35.496 0.8352 343.9562 0.1122 0.3113 0.3356 39.2710 0.0014 0.0011
35.9136 - 36.7488 36.3312 0.8352 352.0493 0.1097 0.3052 0.3289 38.4759 0.0013 0.0011
36.7488 - 37.584 37.1664 0.8352 360.1424 0.1072 0.2993 0.3223 37.7104 0.0013 0.0011
37.584 - 38.4192 38.0016 0.8352 368.2355 0.1049 0.2936 0.3160 36.9730 0.0013 0.0011
38.4192 - 39.2544 38.8368 0.8352 376.3286 0.1026 0.2882 0.3099 36.2624 0.0013 0.0010
39.2544 - 40.0896 39.672 0.8352 384.4217 0.1005 0.2829 0.3041 35.5770 0.0012 0.0010
40.0896 - 40.9248 40.5072 0.8352 392.5148 0.0984 0.2778 0.2984 34.9158 0.0012 0.0010
40.9248 - 41.76 41.3424 0.8352 400.6079 0.0965 0.2728 0.2930 34.2774 0.0012 0.0010
z = layer midpoint depth, presentationz,o′ = vertical effective stress, presentation1 and presentation2 = angles (in radians) as shown in Figure E4, presentationz = vertical consolidation strain, presentation = vertical settlement of individual sublayer; presentation (from bottom to top) = settlement profile (Figure E3).

Table E3. SHANSEP calculated parameters.

Layer Depth [m] σz,o′ [kPa] su,o [kPa] su,oz,o OCRpre σz,f′ [kPa] σ′max [kPa] OCRpost su,fz,f su,f [kPa]
0 - 0.8352 4.0465 13.2882 3.2838 90.6788 121.0378 366.9358 3.0316 0.3043 36.8313
0.8352 - 1.6704 12.1396 13.9549 1.1495 20.2430 128.9152 245.7423 1.9062 0.2199 28.3501
1.6704 - 2.5056 20.2327 14.6215 0.7227 10.4304 136.2899 211.0351 1.5484 0.1901 25.9129
2.5056 - 3.3408 28.3258 15.2882 0.5397 6.8740 143.0484 194.7108 1.3612 0.1737 24.8511
3.3408 - 4.176 36.4189 15.9548 0.4381 5.1023 149.2328 185.8210 1.2452 0.1632 24.3587
4.176 - 5.0112 44.5120 16.6215 0.3734 4.0613 154.9584 180.7763 1.1666 0.1559 24.1653
5.0112 - 5.8464 52.6051 17.2882 0.3286 3.3839 160.3528 178.0108 1.1101 0.1506 24.1526
5.8464 - 6.6816 60.6982 17.9548 0.2958 2.9115 165.5291 176.7201 1.0676 0.1466 24.2600
6.6816 - 7.5168 68.7912 18.6215 0.2707 2.5649 170.5785 176.4447 1.0344 0.1434 24.4530
7.5168 - 8.352 76.8843 19.2881 0.2509 2.3009 175.5710 176.9007 1.0076 0.1407 24.7101
8.352 - 9.1872 84.9774 19.9548 0.2348 2.0935 180.5589 180.5589 1.0000 0.1400 25.2782
9.1872 - 10.0224 93.0705 20.6214 0.2216 1.9268 185.5806 185.5806 1.0000 0.1400 25.9813
10.0224 - 10.8576 101.1636 21.2881 0.2104 1.7899 190.6635 190.6635 1.0000 0.1400 26.6929
10.8576 - 11.6928 109.2567 21.9548 0.2009 1.6758 195.8271 195.8271 1.0000 0.1400 27.4158
11.6928 - 12.528 117.3498 22.6214 0.1928 1.5792 201.0843 201.0843 1.0000 0.1400 28.1518
12.528 - 13.3632 125.4429 23.2881 0.1856 1.4965 206.4434 206.4434 1.0000 0.1400 28.9021
13.3632 - 14.1984 133.5360 23.9547 0.1794 1.4250 211.9090 211.9090 1.0000 0.1400 29.6673
14.1984 - 15.0336 141.6290 24.6214 0.1738 1.3625 217.4830 217.4830 1.0000 0.1400 30.4476
15.0336 - 15.8688 149.7221 25.2881 0.1689 1.3075 223.1655 223.1655 1.0000 0.1400 31.2432
15.8688 - 16.704 157.8152 25.9547 0.1645 1.2587 228.9547 228.9547 1.0000 0.1400 32.0537
16.704 - 17.5392 165.9083 26.6214 0.1605 1.2151 234.8483 234.8483 1.0000 0.1400 32.8788
17.5392 - 18.3744 174.0014 27.2880 0.1568 1.1760 240.8429 240.8429 1.0000 0.1400 33.7180
18.3744 - 19.2096 182.0945 27.9547 0.1535 1.1407 246.9347 246.9347 1.0000 0.1400 34.5709
19.2096 - 20.0448 190.1876 28.6213 0.1505 1.1087 253.1197 253.1197 1.0000 0.1400 35.4368
20.0448 - 20.88 198.2807 29.2880 0.1477 1.0796 259.3936 259.3936 1.0000 0.1400 36.3151
20.88 - 21.7152 206.3737 29.9547 0.1451 1.0529 265.7521 265.7521 1.0000 0.1400 37.2053
21.7152 - 22.5504 214.4668 30.6213 0.1428 1.0285 272.1910 272.1910 1.0000 0.1400 38.1067
22.5504 - 23.3856 222.5599 31.7574 0.1427 1.0276 278.7062 278.7062 1.0000 0.1400 39.0189
23.3856 - 24.2208 230.6530 36.0542 0.1563 1.1705 285.2935 285.2935 1.0000 0.1400 39.9411
24.2208 - 25.056 238.7461 40.3510 0.1690 1.3087 291.9491 312.4493 1.0702 0.1468 42.8614
25.056 - 25.8912 246.8392 44.6478 0.1809 1.4419 298.6693 355.9218 1.1917 0.1583 47.2752
25.8912 - 26.7264 254.9323 48.9447 0.1920 1.5701 305.4504 400.2735 1.3104 0.1692 51.6726
26.7264 - 27.5616 263.0254 53.2415 0.2024 1.6934 312.2892 445.3964 1.4262 0.1795 56.0554
27.5616 - 28.3968 271.1184 57.5383 0.2122 1.8118 319.1824 491.2005 1.5389 0.1893 60.4256
28.3968 - 29.232 279.2115 61.8351 0.2215 1.9255 326.1270 537.6098 1.6485 0.1986 64.7845
29.232 - 30.0672 287.3046 66.1319 0.2302 2.0346 333.1202 584.5598 1.7548 0.2075 69.1335
30.0672 - 30.9024 295.3977 70.4287 0.2384 2.1395 340.1593 631.9952 1.8579 0.2160 73.4738
30.9024 - 31.7376 303.4908 74.7255 0.2462 2.2402 347.2418 679.8683 1.9579 0.2241 77.8063
31.7376 - 32.5728 311.5839 79.0224 0.2536 2.3369 354.3653 728.1376 2.0548 0.2318 82.1321
32.5728 - 33.408 319.6770 83.3192 0.2606 2.4298 361.5276 776.7668 2.1486 0.2391 86.4518
33.408 - 34.2432 327.7701 87.6160 0.2673 2.5192 368.7267 825.7242 2.2394 0.2462 90.7661
34.2432 - 35.0784 335.8632 91.9128 0.2737 2.6052 375.9605 874.9815 2.3273 0.2529 95.0758
35.0784 - 35.9136 343.9562 96.2096 0.2797 2.6879 383.2272 924.5140 2.4124 0.2593 99.3812
35.9136 - 36.7488 352.0493 100.5064 0.2855 2.7675 390.5252 974.2995 2.4948 0.2655 103.6830
36.7488 - 37.584 360.1424 104.8032 0.2910 2.8442 397.8528 1024.3180 2.5746 0.2714 107.9814
37.584 - 38.4192 368.2355 109.1000 0.2963 2.9181 405.2085 1074.5520 2.6518 0.2771 112.2770
38.4192 - 39.2544 376.3286 113.3969 0.3013 2.9894 412.5909 1124.9855 2.7266 0.2825 116.5700
39.2544 - 40.0896 384.4217 117.6937 0.3062 3.0581 419.9987 1175.6040 2.7991 0.2878 120.8607
40.0896 - 40.9248 392.5148 121.9905 0.3108 3.1245 427.4305 1226.3945 2.8692 0.2928 125.1494
40.9248 - 41.76 400.6079 126.2873 0.3152 3.1885 434.8853 1277.3453 2.9372 0.2976 129.4364
presentationz,o′ = initial vertical effective stress, su,o=initial undrained shear strength, presentationsu,o/presentationz,o′=initial c/p ratio, OCRpre= pre-embankment over-consolidation ratio, presentationz,f′ = final vertical effective stress, presentation′max=maximum past pressure, OCRpost= post-embankment over-consolidation ratio, su,f/presentationz,f′= final c/p ratio, su,f=final undrained shear strength

Table E4. Calculated pile side resistance parameters.

Layer Depth [m] z [m] Thickness [m] σz,f′ [kPa] su,f [kPa] α fn [kPa] Fs [kN]
0 - 0.8352 0.4176 0.8352 121.0378 36.83 0.85 0.00 0.00
0.8352 - 1.6704 1.2528 0.8352 128.9152 28.35 1.00 28.36 32.92
1.6704 - 2.5056 2.088 0.8352 136.2899 25.91 1.08 27.87 32.36
2.5056 - 3.3408 2.9232 0.8352 143.0484 24.85 1.13 27.97 32.47
3.3408 - 4.176 3.7584 0.8352 149.2328 24.36 1.16 28.28 32.83
4.176 - 5.0112 4.5936 0.8352 154.9584 24.17 1.19 28.70 33.32
5.0112 - 5.8464 5.4288 0.8352 160.3528 24.15 1.21 29.19 33.89
5.8464 - 6.6816 6.264 0.8352 165.5291 24.26 1.23 29.72 34.51
6.6816 - 7.5168 7.0992 0.8352 170.5785 24.45 1.24 30.29 35.17
7.5168 - 8.352 7.9344 0.8352 175.5710 24.71 1.25 30.89 35.87
8.352 - 9.1872 8.7696 0.8352 180.5589 25.28 1.25 31.69 36.79
9.1872 - 10.0224 9.6048 0.8352 185.5806 25.98 1.25 32.57 37.81
10.0224 - 10.8576 10.44 0.8352 190.6635 26.69 1.25 33.46 38.85
10.8576 - 11.6928 11.2752 0.8352 195.8271 27.42 1.25 34.37 39.90
11.6928 - 12.528 12.1104 0.8352 201.0843 28.15 1.25 35.29 40.97
12.528 - 13.3632 12.9456 0.8352 206.4434 28.90 1.25 36.23 42.06
13.3632 - 14.1984 13.7808 0.8352 211.9090 29.67 1.25 37.19 43.17
14.1984 - 15.0336 14.616 0.8352 217.4830 30.45 1.25 38.17 44.31
15.0336 - 15.8688 15.4512 0.8352 223.1655 31.24 1.25 39.17 45.47
15.8688 - 16.704 16.2864 0.8352 228.9547 32.05 1.25 40.18 46.65
16.704 - 17.5392 17.1216 0.8352 234.8483 32.88 1.25 41.22 47.85
17.5392 - 18.3744 17.9568 0.8352 240.8429 33.72 1.25 42.27 49.07
18.3744 - 19.2096 18.792 0.8352 246.9347 34.57 1.25 43.34 50.31
19.2096 - 20.0448 19.6272 0.8352 253.1197 35.44 1.25 44.42 51.57
20.0448 - 20.88 20.4624 0.8352 259.3936 36.32 1.25 45.52 52.85
20.88 - 21.7152 21.2976 0.8352 265.7521 37.21 1.25 46.64 54.14
21.7152 - 22.5504 22.1328 0.8352 272.1910 38.11 1.25 47.77 55.46
22.5504 - 23.3856 22.968 0.8352 278.7062 39.02 1.25 48.91 56.78
23.3856 - 24.2208 23.8032 0.8352 285.2935 39.94 1.25 50.07 58.13
24.2208 - 25.056 24.6384 0.8352 291.9491 42.86 1.22 52.47 60.91
25.056 - 25.8912 25.4736 0.8352 298.6693 47.28 1.18 55.73 64.70
25.8912 - 26.7264 26.3088 0.8352 305.4504 51.67 1.14 58.93 68.41
26.7264 - 27.5616 27.144 0.8352 312.2892 56.06 1.11 62.06 72.05
27.5616 - 28.3968 27.9792 0.8352 319.1824 60.43 1.08 65.14 75.62
28.3968 - 29.232 28.8144 0.8352 326.1270 64.78 1.05 68.18 79.15
29.232 - 30.0672 29.6496 0.8352 333.1202 69.13 1.03 71.18 82.63
30.0672 - 30.9024 30.4848 0.8352 340.1593 73.47 1.01 74.15 86.08
30.9024 - 31.7376 31.32 0.8352 347.2418 77.81 0.99 77.10 89.50
31.7376 - 32.5728 32.1552 0.8352 354.3653 82.13 0.97 80.02 92.90
32.5728 - 33.408 32.9904 0.8352 361.5276 86.45 0.96 82.92 96.27
33.408 - 34.2432 33.8256 0.8352 368.7267 90.77 0.95 85.81 99.62
34.2432 - 35.0784 34.6608 0.8352 375.9605 95.08 0.93 88.68 102.95
35.0784 - 35.9136 35.496 0.8352 383.2272 99.38 0.92 91.54 106.27
35.9136 - 36.7488 36.3312 0.8352 390.5252 103.68 0.91 94.38 109.57
36.7488 - 37.584 37.1664 0.8352 397.8528 107.98 0.90 97.22 112.86
37.584 - 38.4192 38.0016 0.8352 405.2085 112.28 0.89 100.05 116.15
38.4192 - 39.2544 38.8368 0.8352 412.5909 116.57 0.88 102.86 119.42
39.2544 - 40.0896 39.672 0.8352 419.9987 120.86 0.87 105.68 122.68
40.0896 - 40.9248 40.5072 0.8352 427.4305 125.15 0.87 108.48 125.94
40.9248 - 41.76 41.3424 0.8352 434.8853 129.44 0.86 111.28 129.19
z = layer midpoint depth, presentationz,f′ = vertical effective stress, su,f = undrained shear strength, presentation = total stress side resistance parameter, fn=nominal unit side resistance, Fs=sublayer side resistance. Note: fn neglected for top 1.5m

Table E5. Calculated load as a function of depth.

Layer Depth z Q R M i n [ Q R ] δEC,s δEC Comments:
z=depth,
Q=load,
R=resistance,
δEC,s=segmental elastic compression,
δEC=cumulative elastic compression (from bottom to top),
A=pile cross-sectional area=0.145[m2]
Ep=pile elastic modulus=2.41E+07[kPa]
Ls=length of each pile segment=0.8532[m]
[m] [m] [kN] [kN] [kN] [m] [m]
0 - 0.8352 0.4176 2225.00 3377.24 2225.00 0.0005 0.0242
0.8352 - 1.6704 1.2528 2257.92 3377.24 2257.92 0.0005 0.0236
1.6704 - 2.5056 2.088 2290.28 3344.32 2290.28 0.0005 0.0231
2.5056 - 3.3408 2.9232 2322.74 3311.96 2322.74 0.0006 0.0226
3.3408 - 4.176 3.7584 2355.58 3279.50 2355.58 0.0006 0.0220
4.176 - 5.0112 4.5936 2388.90 3246.67 2388.90 0.0006 0.0214
5.0112 - 5.8464 5.4288 2422.78 3213.35 2422.78 0.0006 0.0209
5.8464 - 6.6816 6.264 2457.29 3179.46 2457.29 0.0006 0.0203
6.6816 - 7.5168 7.0992 2492.46 3144.95 2492.46 0.0006 0.0197
7.5168 - 8.352 7.9344 2528.32 3109.79 2528.32 0.0006 0.0191
8.352 - 9.1872 8.7696 2565.11 3073.92 2565.11 0.0006 0.0185
9.1872 - 10.0224 9.6048 2602.92 3037.13 2602.92 0.0006 0.0179
10.0224 - 10.8576 10.44 2641.77 2999.32 2641.77 0.0006 0.0173
10.8576 - 11.6928 11.2752 2681.67 2960.48 2681.67 0.0006 0.0166
11.6928 - 12.528 12.1104 2722.64 2920.58 2722.64 0.0007 0.0160
12.528 - 13.3632 12.9456 2764.70 2879.61 2764.70 0.0007 0.0153
13.3632 - 14.1984 13.7808 2807.87 2837.55 2807.87 0.0007 0.0147
14.1984 - 15.0336 14.616 2852.18 2794.37 2794.37 0.0007 0.0140
15.0336 - 15.8688 15.4512 2897.65 2750.06 2750.06 0.0007 0.0133
15.8688 - 16.704 16.2864 2944.30 2704.59 2704.59 0.0006 0.0127
16.704 - 17.5392 17.1216 2992.15 2657.95 2657.95 0.0006 0.0120
17.5392 - 18.3744 17.9568 3041.22 2610.10 2610.10 0.0006 0.0114
18.3744 - 19.2096 18.792 3091.53 2561.03 2561.03 0.0006 0.0108
19.2096 - 20.0448 19.6272 3143.10 2510.72 2510.72 0.0006 0.0102
20.0448 - 20.88 20.4624 3195.95 2459.15 2459.15 0.0006 0.0096
20.88 - 21.7152 21.2976 3250.09 2406.30 2406.30 0.0006 0.0090
21.7152 - 22.5504 22.1328 3305.55 2352.15 2352.15 0.0006 0.0084
22.5504 - 23.3856 22.968 3362.33 2296.69 2296.69 0.0005 0.0078
23.3856 - 24.2208 23.8032 3420.46 2239.91 2239.91 0.0005 0.0073
24.2208 - 25.056 24.6384 3481.37 2181.78 2181.78 0.0005 0.0068
25.056 - 25.8912 25.4736 3546.08 2120.87 2120.87 0.0005 0.0062
25.8912 - 26.7264 26.3088 3614.48 2056.17 2056.17 0.0005 0.0057
26.7264 - 27.5616 27.144 3686.53 1987.76 1987.76 0.0005 0.0052
27.5616 - 28.3968 27.9792 3762.15 1915.71 1915.71 0.0005 0.0048
28.3968 - 29.232 28.8144 3841.30 1840.09 1840.09 0.0004 0.0043
29.232 - 30.0672 29.6496 3923.94 1760.94 1760.94 0.0004 0.0039
30.0672 - 30.9024 30.4848 4010.02 1678.31 1678.31 0.0004 0.0035
30.9024 - 31.7376 31.32 4099.52 1592.22 1592.22 0.0004 0.0030
31.7376 - 32.5728 32.1552 4192.42 1502.72 1502.72 0.0004 0.0027
32.5728 - 33.408 32.9904 4288.69 1409.82 1409.82 0.0003 0.0023
33.408 - 34.2432 33.8256 4388.30 1313.56 1313.56 0.0003 0.0020
34.2432 - 35.0784 34.6608 4491.25 1213.94 1213.94 0.0003 0.0017
35.0784 - 35.9136 35.496 4597.52 1110.99 1110.99 0.0003 0.0014
35.9136 - 36.7488 36.3312 4707.09 1004.73 1004.73 0.0002 0.0011
36.7488 - 37.584 37.1664 4819.95 895.15 895.15 0.0002 0.0009
37.584 - 38.4192 38.0016 4936.10 782.29 782.29 0.0002 0.0006
38.4192 - 39.2544 38.8368 5055.51 666.15 666.15 0.0002 0.0005
39.2544 - 40.0896 39.672 5178.20 546.73 546.73 0.0001 0.0003
40.0896 - 40.9248 40.5072 5304.14 424.05 424.05 0.0001 0.0002
40.9248 - 41.76 41.3424 5433.33 298.11 298.11 0.0001 0.0001

Table F2. Calculated soil settlements resulting from drawdown event.

Layer Depth z Thickness σzo σzf ∆σ εz δincrement Σδreversed
[m] [m] [m] [kPa] [kPa] [kPa] [m] [m]
0 - 0.8352 0.4176 0.8352 8.1432 8.1432 4.0967 0.0001 0.0001 0.0787
0.8352 - 1.6704 1.2528 0.8352 24.4296 24.4296 12.2900 0.0004 0.0004 0.0786
1.6704 - 2.5056 2.088 0.8352 40.7160 40.7160 20.4833 0.0007 0.0006 0.0782
2.5056 - 3.3408 2.9232 0.8352 57.0024 57.0024 28.6766 0.0010 0.0008 0.0777
3.3408 - 4.176 3.7584 0.8352 73.2888 73.2888 36.8699 0.0013 0.0011 0.0768
4.176 - 5.0112 4.5936 0.8352 89.5752 89.5752 45.0632 0.0016 0.0013 0.0758
5.0112 - 5.8464 5.4288 0.8352 105.8616 105.8616 53.2565 0.0018 0.0015 0.0745
5.8464 - 6.6816 6.264 0.8352 119.5582 119.5582 58.8600 0.0020 0.0017 0.0729
6.6816 - 7.5168 7.0992 0.8352 127.6512 127.6512 58.8600 0.0020 0.0017 0.0712
7.5168 - 8.352 7.9344 0.8352 135.7443 135.7443 58.8600 0.0020 0.0017 0.0695
8.352 - 9.1872 8.7696 0.8352 143.8374 143.8374 58.8600 0.0020 0.0017 0.0678
9.1872 - 10.0224 9.6048 0.8352 151.9305 151.9305 58.8600 0.0020 0.0017 0.0662
10.0224 - 10.8576 10.44 0.8352 160.0236 160.0236 58.8600 0.0020 0.0017 0.0645
10.8576 - 11.6928 11.2752 0.8352 168.1167 168.1167 58.8600 0.0020 0.0017 0.0628
11.6928 - 12.528 12.1104 0.8352 176.2098 176.2098 58.8600 0.0020 0.0017 0.0611
12.528 - 13.3632 12.9456 0.8352 184.3029 184.3029 58.8600 0.0020 0.0017 0.0594
13.3632 - 14.1984 13.7808 0.8352 192.3960 192.3960 58.8600 0.0020 0.0017 0.0577
14.1984 - 15.0336 14.616 0.8352 200.4890 200.4890 58.8600 0.0020 0.0017 0.0560
15.0336 - 15.8688 15.4512 0.8352 208.5821 208.5821 58.8600 0.0020 0.0017 0.0543
15.8688 - 16.704 16.2864 0.8352 216.6752 216.6752 58.8600 0.0020 0.0017 0.0526
16.704 - 17.5392 17.1216 0.8352 224.7683 224.7683 58.8600 0.0020 0.0017 0.0509
17.5392 - 18.3744 17.9568 0.8352 232.8614 232.8614 58.8600 0.0020 0.0017 0.0492
18.3744 - 19.2096 18.792 0.8352 240.9545 240.9545 58.8600 0.0020 0.0017 0.0475
19.2096 - 20.0448 19.6272 0.8352 249.0476 249.0476 58.8600 0.0020 0.0017 0.0458
20.0448 - 20.88 20.4624 0.8352 257.1407 257.1407 58.8600 0.0020 0.0017 0.0441
20.88 - 21.7152 21.2976 0.8352 265.2337 265.2337 58.8600 0.0020 0.0017 0.0424
21.7152 - 22.5504 22.1328 0.8352 273.3268 273.3268 58.8600 0.0020 0.0017 0.0407
22.5504 - 23.3856 22.968 0.8352 281.4199 281.4199 58.8600 0.0020 0.0017 0.0390
23.3856 - 24.2208 23.8032 0.8352 289.5130 289.5130 58.8600 0.0020 0.0017 0.0373
24.2208 - 25.056 24.6384 0.8352 297.6061 297.6061 58.8600 0.0020 0.0017 0.0356
25.056 - 25.8912 25.4736 0.8352 305.6992 305.6992 58.8600 0.0020 0.0017 0.0339
25.8912 - 26.7264 26.3088 0.8352 313.7923 313.7923 58.8600 0.0020 0.0017 0.0322
26.7264 - 27.5616 27.144 0.8352 321.8854 321.8854 58.8600 0.0020 0.0017 0.0305
27.5616 - 28.3968 27.9792 0.8352 329.9784 329.9784 58.8600 0.0020 0.0017 0.0288
28.3968 - 29.232 28.8144 0.8352 338.0715 338.0715 58.8600 0.0020 0.0017 0.0271
29.232 - 30.0672 29.6496 0.8352 346.1646 346.1646 58.8600 0.0020 0.0017 0.0254
30.0672 - 30.9024 30.4848 0.8352 354.2577 354.2577 58.8600 0.0020 0.0017 0.0237
30.9024 - 31.7376 31.32 0.8352 362.3508 362.3508 58.8600 0.0020 0.0017 0.0221
31.7376 - 32.5728 32.1552 0.8352 370.4439 370.4439 58.8600 0.0020 0.0017 0.0204
32.5728 - 33.408 32.9904 0.8352 378.5370 378.5370 58.8600 0.0020 0.0017 0.0187
33.408 - 34.2432 33.8256 0.8352 386.6301 386.6301 58.8600 0.0020 0.0017 0.0170
34.2432 - 35.0784 34.6608 0.8352 394.7232 394.7232 58.8600 0.0020 0.0017 0.0153
35.0784 - 35.9136 35.496 0.8352 402.8162 402.8162 58.8600 0.0020 0.0017 0.0136
35.9136 - 36.7488 36.3312 0.8352 410.9093 410.9093 58.8600 0.0020 0.0017 0.0119
36.7488 - 37.584 37.1664 0.8352 419.0024 419.0024 58.8600 0.0020 0.0017 0.0102
37.584 - 38.4192 38.0016 0.8352 427.0955 427.0955 58.8600 0.0020 0.0017 0.0085
38.4192 - 39.2544 38.8368 0.8352 435.1886 435.1886 58.8600 0.0020 0.0017 0.0068
39.2544 - 40.0896 39.672 0.8352 443.2817 443.2817 58.8600 0.0020 0.0017 0.0051
40.0896 - 40.9248 40.5072 0.8352 451.3748 451.3748 58.8600 0.0020 0.0017 0.0034
40.9248 - 41.76 41.3424 0.8352 459.4679 459.4679 58.8600 0.0020 0.0017 0.0017
z = layer midpoint depth, σzo′ = vertical effective stress, σzf′ = final effective stress, ∆σ = change in effective stress, εz = incremental soil strain, δincrement = incremental soil settlement, Σδreversed = soil settlement from tip of pile to top of pile.

Table F3. Calculated pile side resistance parameters for the pre-drawdown condition.

Layer Depth z Thickness γ σzo su α fn Fs
[m] [m] [m] [kN/m3] [kPa] [kPa] [kPa] [kN]
0 - 0.8352 0.4176 0.8352 19.5 4.0465 13.29 0.35 0.00 0.00
0.8352 - 1.6704 1.2528 0.8352 19.5 12.1396 13.95 0.45 6.32 7.34
1.6704 - 2.5056 2.088 0.8352 19.5 20.2327 14.62 0.55 8.07 9.37
2.5056 - 3.3408 2.9232 0.8352 19.5 28.3258 15.29 0.64 9.76 11.33
3.3408 - 4.176 3.7584 0.8352 19.5 36.4189 15.95 0.71 11.31 13.13
4.176 - 5.0112 4.5936 0.8352 19.5 44.5120 16.62 0.77 12.76 14.81
5.0112 - 5.8464 5.4288 0.8352 19.5 52.6051 17.29 0.82 14.14 16.42
5.8464 - 6.6816 6.264 0.8352 19.5 60.6982 17.95 0.86 15.48 17.98
6.6816 - 7.5168 7.0992 0.8352 19.5 68.7912 18.62 0.90 16.79 19.49
7.5168 - 8.352 7.9344 0.8352 19.5 76.8843 19.29 0.94 18.06 20.97
8.352 - 9.1872 8.7696 0.8352 19.5 84.9774 19.95 0.97 19.31 22.42
9.1872 - 10.0224 9.6048 0.8352 19.5 93.0705 20.62 1.00 20.55 23.86
10.0224 - 10.8576 10.44 0.8352 19.5 101.1636 21.29 1.02 21.77 25.27
10.8576 - 11.6928 11.2752 0.8352 19.5 109.2567 21.95 1.05 22.97 26.67
11.6928 - 12.528 12.1104 0.8352 19.5 117.3498 22.62 1.07 24.17 28.06
12.528 - 13.3632 12.9456 0.8352 19.5 125.4429 23.29 1.09 25.35 29.43
13.3632 - 14.1984 13.7808 0.8352 19.5 133.5360 23.95 1.11 26.53 30.80
14.1984 - 15.0336 14.616 0.8352 19.5 141.6290 24.62 1.12 27.70 32.16
15.0336 - 15.8688 15.4512 0.8352 19.5 149.7221 25.29 1.14 28.86 33.51
15.8688 - 16.704 16.2864 0.8352 19.5 157.8152 25.95 1.16 30.02 34.85
16.704 - 17.5392 17.1216 0.8352 19.5 165.9083 26.62 1.17 31.17 36.19
17.5392 - 18.3744 17.9568 0.8352 19.5 174.0014 27.29 1.18 32.32 37.52
18.3744 - 19.2096 18.792 0.8352 19.5 182.0945 27.95 1.20 33.46 38.85
19.2096 - 20.0448 19.6272 0.8352 19.5 190.1876 28.62 1.21 34.61 40.17
20.0448 - 20.88 20.4624 0.8352 19.5 198.2807 29.29 1.22 35.74 41.50
20.88 - 21.7152 21.2976 0.8352 19.5 206.3737 29.95 1.23 36.88 42.81
21.7152 - 22.5504 22.1328 0.8352 19.5 214.4668 30.62 1.24 38.01 44.13
22.5504 - 23.3856 22.968 0.8352 19.5 222.5599 31.76 1.24 39.43 45.78
23.3856 - 24.2208 23.8032 0.8352 19.5 230.6530 36.05 1.19 42.77 49.66
24.2208 - 25.056 24.6384 0.8352 19.5 238.7461 40.35 1.14 46.04 53.45
25.056 - 25.8912 25.4736 0.8352 19.5 246.8392 44.65 1.10 49.24 57.16
25.8912 - 26.7264 26.3088 0.8352 19.5 254.9323 48.94 1.07 52.39 60.82
26.7264 - 27.5616 27.144 0.8352 19.5 263.0254 53.24 1.04 55.51 64.44
27.5616 - 28.3968 27.9792 0.8352 19.5 271.1184 57.54 1.02 58.58 68.01
28.3968 - 29.232 28.8144 0.8352 19.5 279.2115 61.84 1.00 61.63 71.55
29.232 - 30.0672 29.6496 0.8352 19.5 287.3046 66.13 0.98 64.65 75.06
30.0672 - 30.9024 30.4848 0.8352 19.5 295.3977 70.43 0.96 67.65 78.54
30.9024 - 31.7376 31.32 0.8352 19.5 303.4908 74.73 0.95 70.63 82.00
31.7376 - 32.5728 32.1552 0.8352 19.5 311.5839 79.02 0.93 73.60 85.44
32.5728 - 33.408 32.9904 0.8352 19.5 319.6770 83.32 0.92 76.55 88.87
33.408 - 34.2432 33.8256 0.8352 19.5 327.7701 87.62 0.91 79.49 92.28
34.2432 - 35.0784 34.6608 0.8352 19.5 335.8632 91.91 0.90 82.41 95.67
35.0784 - 35.9136 35.496 0.8352 19.5 343.9562 96.21 0.89 85.32 99.06
35.9136 - 36.7488 36.3312 0.8352 19.5 352.0493 100.51 0.88 88.23 102.43
36.7488 - 37.584 37.1664 0.8352 19.5 360.1424 104.80 0.87 91.12 105.79
37.584 - 38.4192 38.0016 0.8352 19.5 368.2355 109.10 0.86 94.01 109.14
38.4192 - 39.2544 38.8368 0.8352 19.5 376.3286 113.40 0.85 96.89 112.49
39.2544 - 40.0896 39.672 0.8352 19.5 384.4217 117.69 0.85 99.77 115.82
40.0896 - 40.9248 40.5072 0.8352 19.5 392.5148 121.99 0.84 102.64 119.15
40.9248 - 41.76 41.3424 0.8352 19.5 400.6079 126.29 0.84 105.50 122.48
z = layer midpoint depth, σzo′ = vertical effective stress, γ=unit weight, su = undrained shear strength, α = total stress side resistance parameter, fn=nominal unit side resistance, Fs=sublayer side resistance. Note: fn neglected for top 1.5m

Table F4. Calculated pile side resistance parameters for the post-drawdown condition.

Layer Depth z Thickness γ σzo su α fn Fs
[m] [m] [m] [kN/m3] [kPa] [kPa] [kPa] [kN]
0 - 0.8352 0.4176 0.8352 19.5 8.1432 13.29 0.41 0.00 0.00
0.8352 - 1.6704 1.2528 0.8352 19.5 24.4296 13.95 0.62 8.66 10.05
1.6704 - 2.5056 2.088 0.8352 19.5 40.7160 14.62 0.78 11.44 13.29
2.5056 - 3.3408 2.9232 0.8352 19.5 57.0024 15.29 0.91 13.85 16.07
3.3408 - 4.176 3.7584 0.8352 19.5 73.2888 15.95 1.01 16.04 18.62
4.176 - 5.0112 4.5936 0.8352 19.5 89.5752 16.62 1.09 18.10 21.01
5.0112 - 5.8464 5.4288 0.8352 19.5 105.8616 17.29 1.16 20.07 23.29
5.8464 - 6.6816 6.264 0.8352 19.5 119.5582 17.95 1.21 21.73 25.23
6.6816 - 7.5168 7.0992 0.8352 19.5 127.6512 18.62 1.23 22.87 26.55
7.5168 - 8.352 7.9344 0.8352 19.5 135.7443 19.29 1.24 24.00 27.86
8.352 - 9.1872 8.7696 0.8352 19.5 143.8374 19.95 1.26 25.13 29.17
9.1872 - 10.0224 9.6048 0.8352 19.5 151.9305 20.62 1.27 26.25 30.48
10.0224 - 10.8576 10.44 0.8352 19.5 160.0236 21.29 1.29 27.38 31.78
10.8576 - 11.6928 11.2752 0.8352 19.5 168.1167 21.95 1.30 28.50 33.08
11.6928 - 12.528 12.1104 0.8352 19.5 176.2098 22.62 1.31 29.61 34.38
12.528 - 13.3632 12.9456 0.8352 19.5 184.3029 23.29 1.32 30.73 35.67
13.3632 - 14.1984 13.7808 0.8352 19.5 192.3960 23.95 1.33 31.84 36.97
14.1984 - 15.0336 14.616 0.8352 19.5 200.4890 24.62 1.34 32.95 38.26
15.0336 - 15.8688 15.4512 0.8352 19.5 208.5821 25.29 1.35 34.06 39.55
15.8688 - 16.704 16.2864 0.8352 19.5 216.6752 25.95 1.36 35.17 40.83
16.704 - 17.5392 17.1216 0.8352 19.5 224.7683 26.62 1.36 36.28 42.12
17.5392 - 18.3744 17.9568 0.8352 19.5 232.8614 27.29 1.37 37.39 43.41
18.3744 - 19.2096 18.792 0.8352 19.5 240.9545 27.95 1.38 38.50 44.69
19.2096 - 20.0448 19.6272 0.8352 19.5 249.0476 28.62 1.38 39.60 45.97
20.0448 - 20.88 20.4624 0.8352 19.5 257.1407 29.29 1.39 40.70 47.25
20.88 - 21.7152 21.2976 0.8352 19.5 265.2337 29.95 1.40 41.81 48.54
21.7152 - 22.5504 22.1328 0.8352 19.5 273.3268 30.62 1.40 42.91 49.82
22.5504 - 23.3856 22.968 0.8352 19.5 281.4199 31.76 1.40 44.34 51.48
23.3856 - 24.2208 23.8032 0.8352 19.5 289.5130 36.05 1.33 47.92 55.63
24.2208 - 25.056 24.6384 0.8352 19.5 297.6061 40.35 1.27 51.40 59.67
25.056 - 25.8912 25.4736 0.8352 19.5 305.6992 44.65 1.23 54.80 63.62
25.8912 - 26.7264 26.3088 0.8352 19.5 313.7923 48.94 1.19 58.13 67.48
26.7264 - 27.5616 27.144 0.8352 19.5 321.8854 53.24 1.15 61.40 71.28
27.5616 - 28.3968 27.9792 0.8352 19.5 329.9784 57.54 1.12 64.63 75.03
28.3968 - 29.232 28.8144 0.8352 19.5 338.0715 61.84 1.10 67.82 78.73
29.232 - 30.0672 29.6496 0.8352 19.5 346.1646 66.13 1.07 70.97 82.39
30.0672 - 30.9024 30.4848 0.8352 19.5 354.2577 70.43 1.05 74.09 86.01
30.9024 - 31.7376 31.32 0.8352 19.5 362.3508 74.73 1.03 77.18 89.60
31.7376 - 32.5728 32.1552 0.8352 19.5 370.4439 79.02 1.02 80.25 93.17
32.5728 - 33.408 32.9904 0.8352 19.5 378.5370 83.32 1.00 83.30 96.70
33.408 - 34.2432 33.8256 0.8352 19.5 386.6301 87.62 0.99 86.33 100.22
34.2432 - 35.0784 34.6608 0.8352 19.5 394.7232 91.91 0.97 89.34 103.72
35.0784 - 35.9136 35.496 0.8352 19.5 402.8162 96.21 0.96 92.34 107.20
35.9136 - 36.7488 36.3312 0.8352 19.5 410.9093 100.51 0.95 95.32 110.66
36.7488 - 37.584 37.1664 0.8352 19.5 419.0024 104.80 0.94 98.29 114.11
37.584 - 38.4192 38.0016 0.8352 19.5 427.0955 109.10 0.93 101.25 117.54
38.4192 - 39.2544 38.8368 0.8352 19.5 435.1886 113.40 0.92 104.20 120.96
39.2544 - 40.0896 39.672 0.8352 19.5 443.2817 117.69 0.91 107.13 124.37
40.0896 - 40.9248 40.5072 0.8352 19.5 451.3748 121.99 0.90 110.06 127.78
40.9248 - 41.76 41.3424 0.8352 19.5 459.4679 126.29 0.89 112.98 131.17
z = layer midpoint depth, σzo′ = vertical effective stress, γ=unit weight, su = undrained shear strength, α = total stress side resistance parameter, fn=nominal unit side resistance, Fs=sublayer side resistance. Note: fn neglected for top 1.5m

Table F5. Ultimate Unit Tip Resistance, [kN/m2] values for the four layers used in the TZPILE software.

TZPILE Depth Layer Depth Midpoint Depth su At qn
[m] [m] [m] [kPa] [m2] [kPa]
0 0 - 0.8352 0.4176 13.29 0.145 119.59
6 5.8464 - 6.6816 6.264 17.95 0.145 161.59
22.86 21.7152 - 22.504 22.1328 30.62 0.145 275.59
41.76 40.9248 - 41.76 41.3424 126.29 0.145 1136.58
100 NA - NA NA NA NA NA

Table F6. Design parameters used to develop t-z and Q-w curves.

Top of Layer Bottom of Layer
Layer 1a Depth, z, [m] 0 6
Unit Weight, γ, [kN/m3] 19.5 19.5
Undrained Shear Strength, su, [kPa] 12.95 17.74
Ultimate Skin Resistance, fs,ult, [kPa] 8.66 21.73
Ultimate Tip Resistance, qp,ult, [kPa] 116.55 159.66
Layer 1b Depth, z, [m] 6 22.86
Unit Weight, γ, [kN/m3] 9.69 9.69
Undrained Shear Strength, su, [kPa] 17.74 31.2
Ultimate Skin Resistance, fs,ult, [kPa] 21.73 42.91
Ultimate Tip Resistance, qp,ult, [kPa] 159.66 280.8
Layer 2 Depth, z, [m] 22.86 41.76
Unit Weight, γ, [kN/m3] 9.69 9.69
Undrained Shear Strength, su, [kPa] 31.2 128.44
Ultimate Skin Resistance, fs,ult, [kPa] 42.91 112.98
Ultimate Tip Resistance, qp,ult, [kPa] 280.8 1155.96

Table F7. Develop t-z curves using Vijayvergiya (1977).

Displacement, z, [m]
(used for all t-z curves)
Layer 1a
Top of Layer
Layer 1a
Bottom of Layer
Layer 1b
Bottom of Layer
Layer 2
Bottom of Layer
Load Transfer, t, [kPa]
0.0000 0.0000 0.0000 0.0000 0.0000
0.0010 5.1697 12.9720 25.6157 67.4449
0.0020 6.6347 16.6480 32.8746 86.5572
0.0030 7.4901 18.7945 37.1133 97.7177
0.0040 8.0301 20.1494 39.7887 104.7619
0.0050 8.3684 20.9983 41.4651 109.1756
0.0060 8.5635 21.4878 42.4317 111.7208
0.0070 8.6500 21.7050 42.8607 112.8501
0.0075 8.6600 21.7300 42.9100 112.9800
0.0080 8.6507 21.7066 42.8638 112.8585

Table F9. Soil settlement, pile settlement, and combined load and resistance values for Design Example 4.

z Combined Load and Resistance Pile Settlement Soil Settlement
V HC V HC
[m] [kN] [m] [m]
41.34 306 299 0.0438 0.0449 0.0017
40.51 427 427 0.0439 0.0450 0.0034
39.67 544 551 0.0440 0.0451 0.0051
38.84 658 672 0.0442 0.0453 0.0068
38.00 768 789 0.0444 0.0455 0.0085
37.17 875 904 0.0446 0.0457 0.0102
36.33 979 1014 0.0448 0.0459 0.0119
35.50 1079 1121 0.0450 0.0462 0.0136
34.66 1176 1225 0.0453 0.0465 0.0153
33.83 1270 1325 0.0456 0.0468 0.0170
32.99 1360 1422 0.0459 0.0471 0.0187
32.16 1446 1515 0.0462 0.0475 0.0204
31.32 1530 1605 0.0466 0.0479 0.0221
30.48 1609 1691 0.0470 0.0483 0.0237
29.65 1686 1773 0.0474 0.0487 0.0254
28.81 1759 1852 0.0478 0.0491 0.0271
27.98 1829 1927 0.0482 0.0496 0.0288
27.14 1895 1998 0.0486 0.0501 0.0305
26.31 1958 2066 0.0491 0.0506 0.0322
25.47 2017 2129 0.0496 0.0511 0.0339
24.64 2073 2189 0.0501 0.0516 0.0356
23.80 2126 2245 0.0506 0.0521 0.0373
22.97 2175 2296 0.0511 0.0527 0.0390
22.13 2222 2346 0.0516 0.0533 0.0407
21.30 2268 2394 0.0522 0.0538 0.0424
20.46 2312 2442 0.0527 0.0544 0.0441
19.63 2355 2488 0.0533 0.0550 0.0458
18.79 2397 2532 0.0538 0.0556 0.0475
17.96 2438 2576 0.0544 0.0562 0.0492
17.12 2475 2618 0.0550 0.0569 0.0509
16.29 2510 2659 0.0556 0.0575 0.0526
15.45 2541 2698 0.0562 0.0581 0.0543
14.62 2563 2677 0.0568 0.0588 0.0560
13.78 2570 2639 0.0574 0.0594 0.0577
12.95 2556 2602 0.0580 0.0600 0.0594
12.11 2532 2566 0.0586 0.0606 0.0611
11.28 2505 2531 0.0593 0.0612 0.0628
10.44 2477 2498 0.0598 0.0618 0.0645
9.61 2448 2467 0.0604 0.0624 0.0662
8.77 2420 2436 0.0610 0.0630 0.0678
7.93 2393 2407 0.0616 0.0636 0.0695
7.10 2367 2379 0.0622 0.0642 0.0712
6.26 2342 2353 0.0627 0.0647 0.0729
5.43 2319 2327 0.0633 0.0653 0.0745
4.59 2298 2304 0.0638 0.0658 0.0758
3.76 2278 2283 0.0644 0.0664 0.0768
2.92 2261 2264 0.0649 0.0669 0.0777
2.09 2245 2248 0.0655 0.0675 0.0782
1.25 2232 2235 0.0660 0.0680 0.0786
0.42 2231 2225 0.0665 0.0685 0.0787
z=Depth, V=Vijayvergiya, HC=Hand Calculations

Table G1. Bearing capacity factor (kc) for various soil types, pile groups, and cone penetration tip resistance (after Bustamante and Gianeselli, 1982). Units of tons per square foot are also included.

kc Factor
Soil Type CPT Tip Resistance, qc Group I Group II
Soft clay and mud <10 MPa (<9.3 tsf) 0.40 0.5
Moderately compact clay 10 to 50 MPa (9.3 to 46.6 tsf) 0.35 0.45
Silt and loose sand <50 MPa (<46.6 tsf) 0.40 0.5
Compact to stiff clay and compact silt >50 MPa (>46.6 tsf) 0.45 0.55
Soft chalk <50 MPa (<46.6 tsf) 0.20 0.3
Moderately compact sand and gravel 50 to 120 MPa (46.6 to 111.9 tsf) 0.4 0.5
Weathered to fragmented chalk >50 MPa (>46.6 tsf) 0.2 0.4
Compact to very compact sand and gravel >120 MPa (>111.9 tsf) 0.3 0.4

Table G2. Side resistance factors for the LCPC method (after Bustamante and Gianeselli, 1982).

Soil Type qc [MPa] aLCPC Limiting value of fs,max [MPa]
Category
I II I II III
IA IB IIA IIB IA IB IIA IIB IIIA IIIB
Soft clay and mud <1 30 30 30 30 0.015 0.015 0.015 0.015 0.015
Moderately compact clay 1 to 5 40 80 40 80 0.08 to 0.035 0.08 to 0.035 0.08 to 0.035 0.035 0.08 >0.120
Silt and loose sand <5 60 150 60 120 0.035 0.035 0.035 0.035 0.08
Compact to stiff clay and compact silt >5 60 120 60 120 0.08 to 0.035 0.08 to 0.035 0.08 to 0.035 0.035 0.08 >0.2
Soft chalk <5 100 120 100 120 0.035 0.035 0.035 0.035 0.08
Moderately compact sand and gravel 5 to 12 100 200 100 200 0.12 to 0.08 0.08 to 0.035 0.12 to 0.08 0.08 0.12 >0.2
Weathered to fragmented chalk >5 60 80 60 80 0.15 to 0.12 0.12 to 0.08 0.15 to 0.12 0.12 0.15 >0.2
Compact to very compact sand and gravel >12 150 300 150 200 0.15 to 0.12 0.12 to 0.08 0.15 to 0.12 0.12 0.15 >0.2

Table G3. Average CPT sounding record for the Blytheville, AR Test Site.

z fs qt u2 Comments: z=Depth [ft],
fs=Sleeve friction [tsf],
qt=Tip resistance [tsf],
u2=Pore pressure [psi]

Values collected every presentationz=0.164ft but reported herein every presentationz=0.984ft (except for first and last values).
0.163 0.163 5.990 -0.080
1.148 0.343 6.644 -4.346
2.133 0.321 5.024 -3.507
3.117 0.232 30.509 -0.474
4.101 0.283 8.452 0.052
5.085 0.259 3.602 1.817
6.070 0.279 4.583 2.308
7.054 0.321 5.619 3.310
8.038 0.314 7.221 3.799
9.022 0.178 4.880 4.400
10.007 0.153 26.554 4.783
10.991 0.283 55.585 4.052
11.975 0.380 57.579 3.349
12.959 0.299 41.164 3.934
13.944 0.332 44.718 1.888
14.928 0.412 80.277 0.862
15.912 0.485 99.291 1.548
16.896 0.549 102.243 3.270
17.881 0.435 82.030 3.717
18.865 0.413 81.470 3.345
19.849 0.259 50.410 3.707
20.833 0.233 30.238 3.041
21.818 0.294 56.976 4.014
22.802 0.274 48.270 7.523
23.786 0.306 58.057 7.823
24.770 0.316 59.929 8.438
25.755 0.303 59.692 9.553
26.739 0.271 53.975 11.429
27.723 0.290 60.599 11.954
28.707 0.307 68.630 12.150
29.692 0.328 76.193 13.576
30.676 0.350 86.086 13.470
31.660 0.336 89.231 14.176
32.644 0.419 98.042 15.180
33.629 0.396 96.314 15.467
34.613 0.372 107.343 15.953
35.597 0.410 113.077 16.297
36.581 0.508 143.899 16.744
37.566 0.666 185.609 16.821
38.550 0.608 179.809 17.287
39.561 0.656 185.487 17.292
40.546 0.840 206.957 17.771
41.530 0.866 205.632 16.569
42.569 0.925 210.830 18.586
43.553 0.995 232.122 18.903
44.537 1.028 245.359 19.529
45.549 1.077 259.473 19.777
46.533 1.035 227.953 20.100
47.517 1.000 211.495 20.318
48.529 0.967 220.913 20.881
49.513 1.070 253.789 20.833
50.498 0.895 268.331 21.315
51.482 0.757 256.276 21.870
52.466 0.680 247.751 22.581
53.450 0.717 240.933 22.914
54.435 0.653 219.256 23.600
55.474 0.648 245.683 24.005
56.458 0.688 250.659 24.232
57.442 0.767 245.872 24.346
58.481 0.847 268.145 25.168
59.465 1.130 329.023 25.588
60.449 1.317 379.164 25.735
61.488 1.457 392.789 24.920
62.473 1.366 378.497 26.585
63.457 1.462 393.080 26.816
64.469 1.218 375.347 27.283
65.518 1.017 343.502 27.677
66.503 1.156 355.590 28.325
67.626 1.355 342.052 29.154
68.611 0.844 311.256 29.417
69.595 0.687 298.517 29.799
70.579 0.727 290.580 30.460
71.563 0.842 289.159 30.683
72.548 0.729 249.830 31.720
73.710 0.863 273.813 31.332
74.694 0.901 348.687 31.865
75.678 1.635 376.848 31.968
77.018 0.980 322.823 32.031
78.002 0.885 260.040 28.425
78.986 1.091 375.017 32.978
79.970 0.986 345.087 32.455
80.955 0.788 374.134 33.192
81.939 1.274 374.530 34.805
82.923 1.856 404.210 33.297
83.661 1.557 351.657 29.414

Table G5. Results from calculations to find corrected tip resistance.

z presentation presentationvo Ic n Qtn qc1N CN m presentationqc1N qc1Ncs Comments: For Event #1 – amax=0.1,
Mw=6.5,
z=Depth [ft],
presentation=Unit weight [pcf],
presentationvo–=Vertical effective stress [psf],
Ic=Soil behavior type index,
n=stress normalization exponent,
Qtn=Normalized corrected net cone tip resistance,
qc1N=Overburden stress-corrected cone tip resistance [tsf],
CN=Overburden correction factor,
m=Overburden correction factor exponent,
presentationqc1N=Increment in cone tip resistance [tsf],
qc1Ncs
=Clean sand and overburden stress-corrected cone tip resistance [tsf].

Values collected every presentationz=0.164ft but reported herein every presentationz=0.984ft (except for first and last values).

See definition of variables on previous page.

Values collected every ∆z=0.164ft but reported herein every ∆z=0.984ft (except for first and last values).
0.163 120.822 19.738 2.190 0.683 137.351 9.624 1.700 0.628 43.214 52.837
1.148 107.801 125.255 2.543 0.819 67.872 10.674 1.700 0.592 53.081 63.755
2.133 105.991 229.496 2.720 0.887 46.606 8.071 1.700 0.596 54.513 62.584
3.117 113.182 338.402 1.837 0.553 104.314 49.018 1.700 0.620 6.153 55.171
4.101 110.548 453.651 2.539 0.821 43.669 13.580 1.700 0.581 53.853 67.433
5.085 103.417 555.312 2.996 0.996 22.063 5.786 1.700 0.599 55.730 61.516
6.070 105.630 658.756 2.924 0.969 23.599 7.363 1.700 0.594 55.970 63.333
7.054 106.426 762.746 2.882 0.954 25.261 9.027 1.700 0.588 56.185 65.212
8.038 107.520 867.675 2.769 0.912 27.638 11.602 1.700 0.580 56.046 67.648
9.022 101.243 969.769 2.880 0.955 17.579 7.841 1.700 0.593 55.809 63.649
10.007 114.235 1074.550 1.971 0.611 58.582 42.662 1.700 0.565 30.473 73.136
10.991 113.878 1188.052 1.729 0.520 103.531 89.305 1.700 0.523 0.000 89.305
11.975 113.888 1299.143 1.785 0.543 105.468 92.510 1.700 0.514 0.488 92.998
12.959 112.948 1411.400 1.925 0.597 76.425 66.136 1.700 0.518 25.361 91.497
13.944 115.292 1522.129 1.917 0.595 79.401 71.846 1.700 0.507 24.349 96.195
14.928 114.023 1635.068 1.662 0.500 124.908 120.641 1.590 0.456 0.000 120.641
15.912 116.185 1748.957 1.593 0.475 146.144 139.982 1.492 0.420 0.000 139.982
16.896 117.133 1864.281 1.614 0.484 147.175 140.264 1.452 0.420 0.000 140.264
17.881 115.266 1978.696 1.693 0.515 117.381 114.269 1.474 0.468 0.000 114.269
18.865 114.686 2091.573 1.694 0.517 113.434 111.183 1.444 0.474 0.000 111.183
19.849 110.400 2201.933 1.871 0.585 71.261 70.255 1.475 0.535 14.012 84.267
20.833 107.996 2306.491 2.144 0.690 44.084 40.910 1.432 0.524 47.842 88.752
21.818 111.073 2414.187 1.845 0.578 76.371 75.895 1.409 0.535 8.632 84.527
22.802 109.488 2522.566 1.933 0.612 64.164 62.479 1.370 0.524 26.522 89.002
23.786 110.799 2630.505 1.860 0.585 74.492 73.952 1.348 0.532 11.743 85.695
24.770 111.341 2740.067 1.857 0.585 75.134 74.755 1.320 0.531 11.049 85.804
25.755 110.547 2849.221 1.858 0.587 73.204 73.119 1.296 0.535 11.296 84.415
26.739 110.086 2957.878 1.901 0.604 65.096 64.819 1.271 0.533 20.286 85.105
27.723 110.588 3067.169 1.856 0.588 71.248 71.631 1.251 0.540 10.844 82.475
28.707 111.897 3176.704 1.804 0.570 78.705 79.730 1.229 0.542 1.790 81.520
29.692 112.653 3286.978 1.764 0.556 85.465 86.551 1.202 0.530 0.026 86.577
30.676 113.638 3397.761 1.713 0.538 94.472 95.483 1.174 0.508 0.000 95.483
31.660 113.003 3509.623 1.691 0.530 96.059 97.256 1.153 0.504 0.000 97.256
32.644 115.062 3621.893 1.688 0.530 103.928 104.746 1.130 0.488 0.000 104.746
33.629 114.899 3734.786 1.693 0.533 100.416 101.522 1.115 0.495 0.000 101.522
34.613 114.591 3847.326 1.622 0.508 109.784 111.068 1.095 0.475 0.000 111.068
35.597 115.863 3961.458 1.618 0.507 113.953 115.207 1.078 0.466 0.000 115.207
36.581 117.757 4076.590 1.526 0.474 142.725 143.622 1.056 0.414 0.000 143.622
37.566 120.242 4194.474 1.440 0.443 181.446 181.864 1.037 0.355 0.000 181.864
38.550 119.499 4312.741 1.444 0.445 173.422 174.495 1.027 0.365 0.000 174.495
39.561 120.083 4434.628 1.447 0.448 176.598 178.041 1.016 0.360 0.000 178.041
40.546 121.871 4553.866 1.445 0.448 194.761 196.572 1.005 0.334 0.000 196.572
41.530 122.199 4674.384 1.461 0.456 191.015 193.487 0.996 0.338 0.000 193.487
42.569 123.445 4802.070 1.468 0.460 193.243 196.480 0.986 0.334 0.000 196.480
43.553 123.941 4923.348 1.432 0.448 210.501 214.749 0.979 0.310 0.000 214.749
44.537 123.910 5045.827 1.410 0.441 220.087 225.413 0.972 0.297 0.000 225.413
45.549 124.236 5171.284 1.392 0.435 230.278 236.911 0.966 0.283 0.000 236.911
46.533 123.506 5293.349 1.467 0.465 198.903 205.483 0.954 0.322 0.000 205.483
47.517 124.343 5415.452 1.509 0.483 181.689 188.407 0.943 0.345 0.000 188.407
48.529 123.000 5540.488 1.477 0.472 188.003 195.521 0.936 0.336 0.000 195.521
49.513 123.547 5662.041 1.420 0.452 214.886 224.721 0.937 0.298 0.000 224.721
50.498 122.997 5783.748 1.342 0.424 226.501 236.827 0.934 0.283 0.000 236.827
51.482 121.750 5903.794 1.335 0.422 214.291 223.896 0.924 0.299 0.000 223.896
52.466 121.018 6022.677 1.335 0.424 205.151 214.395 0.916 0.311 0.000 214.395
53.450 121.375 6141.791 1.368 0.438 196.831 206.549 0.907 0.321 0.000 206.549
54.435 120.474 6260.247 1.410 0.455 176.321 185.030 0.893 0.350 0.000 185.030
55.474 120.556 6385.614 1.341 0.430 197.683 208.063 0.896 0.319 0.000 208.063
56.458 120.924 6504.888 1.345 0.433 199.820 211.300 0.892 0.315 0.000 211.300
57.442 121.561 6624.140 1.386 0.450 193.053 205.445 0.884 0.322 0.000 205.445
58.481 122.479 6750.393 1.358 0.441 209.641 224.845 0.887 0.298 0.000 224.845
59.465 124.887 6872.280 1.304 0.421 257.745 278.294 0.895 0.264 0.000 254.000
60.449 126.740 6996.671 1.257 0.405 297.100 319.071 0.890 0.264 0.000 254.000
61.488 127.600 7128.689 1.266 0.410 304.597 328.776 0.886 0.264 0.000 254.000
62.473 126.950 7254.346 1.275 0.415 290.399 315.231 0.881 0.264 0.000 254.000
63.457 127.479 7379.282 1.272 0.416 299.257 325.789 0.877 0.264 0.000 254.000
64.469 125.938 7507.147 1.255 0.411 284.171 309.583 0.873 0.264 0.000 254.000
65.518 124.651 7640.161 1.269 0.417 256.807 281.911 0.868 0.264 0.000 254.000
66.503 125.502 7762.766 1.283 0.424 263.053 290.541 0.865 0.264 0.000 254.000
67.626 126.212 7902.300 1.356 0.454 246.692 278.103 0.860 0.264 0.000 254.000
68.611 122.403 8024.086 1.297 0.433 225.304 251.602 0.855 0.267 0.000 251.602
69.595 121.499 8144.146 1.280 0.428 215.142 238.091 0.844 0.282 0.000 238.091
70.579 121.793 8264.421 1.314 0.442 206.152 229.293 0.835 0.292 0.000 229.293
71.563 122.231 8385.033 1.355 0.459 201.555 226.729 0.830 0.296 0.000 226.729
72.548 122.618 8505.017 1.421 0.485 169.611 188.973 0.800 0.344 0.000 188.973
73.710 123.440 8649.010 1.404 0.481 185.100 209.681 0.810 0.317 0.000 209.681
74.694 123.754 8770.369 1.257 0.426 243.712 275.531 0.836 0.264 0.000 254.000
75.678 128.338 8895.132 1.371 0.471 253.916 296.602 0.833 0.264 0.000 254.000
77.018 124.661 9061.838 1.337 0.460 216.689 252.473 0.828 0.266 0.000 252.473
78.002 124.108 9185.051 1.461 0.509 166.798 191.854 0.781 0.341 0.000 191.854
78.986 125.660 9307.667 1.270 0.438 253.122 291.482 0.822 0.264 0.000 254.000
79.970 124.876 9432.607 1.304 0.452 228.713 267.216 0.819 0.264 0.000 254.000
80.955 124.062 9556.417 1.198 0.413 254.053 288.658 0.816 0.264 0.000 254.000
81.939 126.667 9681.199 1.322 0.462 243.412 287.915 0.813 0.264 0.000 254.000
82.923 130.206 9805.708 1.384 0.487 256.159 309.623 0.811 0.264 0.000 254.000
83.661 127.812 10859.659 1.430 0.505 217.688 268.635 0.808 0.264 0.000 254.000

Table G6. Results from CRR, cyclic stress ratio, and factor of safety calculations.

z rd presentation(z) presentation(z) Kpresentation Cpresentation MSFmax MSF CRR7.5,1atm CRRM,presentation′v CSRM,presentation′v Comments:
For Event #1 – amax=0.1, Mw=6.5, z=Depth [ft], rd=Shear stress reduction coefficient, presentation(z)=Term used to find rd, presentation(z)=Term used to find rd, Kpresentation= correction to cyclic resistance for overburden stress, Cpresentation=Term used to find Kpresentation, MSFmax=Upper bound on magnitude scaling, MSF, CRR7.5,1atm = standardized cyclic resistance corresponding to a moment magnitude earthquake of 7.5, with 15 uniform shear stress cycles, and one atmosphere of pressure, CRRM,presentation–v =Cyclic resistance ratio for the magnitude and effective stress of interest, CSRM,presentation–v=Cyclic stress ratio for the magnitude and effective stress of interest. Values collected every presentationz=0.164ft but reported herein every presentationz=0.984ft (except for first and last values).

See definition of variables on previous page.

Values collected every ∆z=0.164ft but reported herein every ∆z=0.984ft (except for first and last values).
0.163 1.00 0.01 0.00 1.10 0.07 1.12 1.04 0.09 0.11 0.07
1.148 1.00 0.00 0.00 1.10 0.08 1.13 1.05 0.10 0.12 0.07
2.133 1.00 -0.01 0.00 1.10 0.08 1.13 1.05 0.10 0.12 0.09
3.117 1.00 -0.02 0.00 1.10 0.07 1.12 1.04 0.10 0.11 0.11
4.101 0.99 -0.04 0.00 1.10 0.08 1.14 1.05 0.11 0.12 0.11
5.085 0.99 -0.05 0.01 1.10 0.08 1.13 1.05 0.10 0.12 0.12
6.070 0.98 -0.07 0.01 1.10 0.08 1.13 1.05 0.10 0.12 0.12
7.054 0.98 -0.09 0.01 1.10 0.08 1.14 1.05 0.10 0.12 0.13
8.038 0.98 -0.10 0.01 1.10 0.08 1.14 1.05 0.11 0.12 0.13
9.022 0.97 -0.12 0.01 1.10 0.08 1.13 1.05 0.10 0.12 0.13
10.007 0.97 -0.14 0.02 1.10 0.09 1.16 1.06 0.11 0.13 0.13
10.991 0.96 -0.16 0.02 1.10 0.10 1.21 1.08 0.12 0.15 0.13
11.975 0.96 -0.17 0.02 1.10 0.10 1.23 1.09 0.13 0.15 0.13
12.959 0.95 -0.19 0.02 1.10 0.10 1.22 1.08 0.13 0.15 0.13
13.944 0.95 -0.21 0.02 1.10 0.10 1.24 1.09 0.13 0.16 0.13
14.928 0.94 -0.23 0.03 1.10 0.13 1.39 1.15 0.17 0.22 0.13
15.912 0.94 -0.26 0.03 1.10 0.15 1.56 1.21 0.23 0.31 0.13
16.896 0.93 -0.28 0.03 1.10 0.15 1.56 1.21 0.24 0.31 0.13
17.881 0.92 -0.30 0.03 1.10 0.12 1.35 1.13 0.16 0.20 0.13
18.865 0.92 -0.32 0.04 1.09 0.12 1.33 1.12 0.15 0.19 0.13
19.849 0.91 -0.34 0.04 1.07 0.09 1.19 1.07 0.12 0.14 0.13
20.833 0.91 -0.37 0.04 1.07 0.10 1.21 1.08 0.12 0.14 0.13
21.818 0.90 -0.39 0.04 1.06 0.09 1.19 1.07 0.12 0.14 0.13
22.802 0.89 -0.42 0.05 1.06 0.10 1.21 1.08 0.12 0.14 0.13
23.786 0.89 -0.44 0.05 1.05 0.10 1.20 1.07 0.12 0.14 0.13
24.770 0.88 -0.47 0.05 1.05 0.10 1.20 1.07 0.12 0.14 0.13
25.755 0.88 -0.49 0.06 1.05 0.09 1.19 1.07 0.12 0.13 0.12
26.739 0.87 -0.52 0.06 1.04 0.09 1.20 1.07 0.12 0.13 0.12
27.723 0.86 -0.54 0.06 1.04 0.09 1.19 1.07 0.12 0.13 0.12
28.707 0.86 -0.57 0.06 1.04 0.09 1.18 1.07 0.12 0.13 0.12
29.692 0.85 -0.60 0.07 1.03 0.10 1.20 1.08 0.12 0.14 0.12
30.676 0.84 -0.62 0.07 1.03 0.10 1.24 1.09 0.13 0.15 0.12
31.660 0.84 -0.65 0.07 1.03 0.10 1.25 1.09 0.13 0.15 0.12
32.644 0.83 -0.68 0.08 1.03 0.11 1.29 1.11 0.14 0.16 0.12
33.629 0.82 -0.70 0.08 1.02 0.11 1.27 1.10 0.14 0.16 0.12
34.613 0.82 -0.73 0.08 1.02 0.12 1.32 1.12 0.15 0.18 0.12
35.597 0.81 -0.76 0.08 1.02 0.12 1.35 1.13 0.16 0.19 0.12
36.581 0.81 -0.79 0.09 1.02 0.15 1.60 1.23 0.25 0.31 0.12
37.566 0.80 -0.82 0.09 1.02 0.22 2.12 1.42 0.78 1.14 0.11
38.550 0.79 -0.85 0.09 1.01 0.20 2.00 1.38 0.59 0.82 0.11
39.561 0.79 -0.87 0.10 1.01 0.21 2.06 1.40 0.67 0.95 0.11
40.546 0.78 -0.90 0.10 1.00 0.25 2.20 1.45 1.57 2.28 0.11
41.530 0.77 -0.93 0.10 1.00 0.24 2.20 1.45 1.33 1.93 0.11
42.569 0.77 -0.96 0.11 0.99 0.25 2.20 1.45 1.56 2.24 0.11
43.553 0.76 -0.99 0.11 0.98 0.30 2.20 1.45 2.00 2.84 0.11
44.537 0.75 -1.02 0.11 0.97 0.30 2.20 1.45 2.00 2.82 0.11
45.549 0.75 -1.05 0.12 0.96 0.30 2.20 1.45 2.00 2.80 0.11
46.533 0.74 -1.08 0.12 0.96 0.28 2.20 1.45 2.00 2.78 0.10
47.517 0.73 -1.11 0.12 0.96 0.23 2.20 1.45 1.04 1.45 0.10
48.529 0.73 -1.14 0.13 0.95 0.25 2.20 1.45 1.48 2.05 0.10
49.513 0.72 -1.17 0.13 0.93 0.30 2.20 1.45 2.00 2.71 0.10
50.498 0.72 -1.19 0.13 0.93 0.30 2.20 1.45 2.00 2.69 0.10
51.482 0.71 -1.22 0.14 0.92 0.30 2.20 1.45 2.00 2.67 0.10
52.466 0.70 -1.25 0.14 0.91 0.30 2.20 1.45 2.00 2.66 0.10
53.450 0.70 -1.28 0.14 0.91 0.28 2.20 1.45 2.00 2.65 0.10
54.435 0.69 -1.31 0.14 0.93 0.22 2.18 1.44 0.90 1.20 0.10
55.474 0.69 -1.34 0.15 0.90 0.29 2.20 1.45 2.00 2.61 0.10
56.458 0.68 -1.36 0.15 0.89 0.30 2.20 1.45 2.00 2.59 0.09
57.442 0.67 -1.39 0.15 0.89 0.28 2.20 1.45 2.00 2.59 0.09
58.481 0.67 -1.42 0.16 0.88 0.30 2.20 1.45 2.00 2.55 0.09
59.465 0.66 -1.45 0.16 0.87 0.30 2.20 1.45 2.00 2.54 0.09
60.449 0.66 -1.47 0.16 0.87 0.30 2.20 1.45 2.00 2.52 0.09
61.488 0.65 -1.50 0.16 0.86 0.30 2.20 1.45 2.00 2.50 0.09
62.473 0.65 -1.53 0.17 0.86 0.30 2.20 1.45 2.00 2.49 0.09
63.457 0.64 -1.55 0.17 0.85 0.30 2.20 1.45 2.00 2.47 0.09
64.469 0.64 -1.58 0.17 0.85 0.30 2.20 1.45 2.00 2.45 0.09
65.518 0.63 -1.60 0.18 0.84 0.30 2.20 1.45 2.00 2.44 0.09
66.503 0.62 -1.63 0.18 0.83 0.30 2.20 1.45 2.00 2.42 0.09
67.626 0.62 -1.65 0.18 0.83 0.30 2.20 1.45 2.00 2.41 0.08
68.611 0.61 -1.68 0.18 0.82 0.30 2.20 1.45 2.00 2.39 0.08
69.595 0.61 -1.70 0.19 0.82 0.30 2.20 1.45 2.00 2.38 0.08
70.579 0.60 -1.72 0.19 0.81 0.30 2.20 1.45 2.00 2.37 0.08
71.563 0.60 -1.75 0.19 0.81 0.30 2.20 1.45 2.00 2.35 0.08
72.548 0.60 -1.77 0.19 0.85 0.23 2.20 1.45 1.07 1.32 0.08
73.710 0.59 -1.79 0.19 0.80 0.30 2.20 1.45 2.00 2.33 0.08
74.694 0.59 -1.81 0.20 0.80 0.30 2.20 1.45 2.00 2.31 0.08
75.678 0.58 -1.83 0.20 0.79 0.30 2.20 1.45 2.00 2.30 0.08
77.018 0.58 -1.86 0.20 0.79 0.30 2.20 1.45 2.00 2.28 0.08
78.002 0.57 -1.88 0.20 0.83 0.24 2.20 1.45 1.23 1.47 0.08
78.986 0.57 -1.90 0.20 0.78 0.30 2.20 1.45 2.00 2.26 0.08
79.970 0.57 -1.91 0.21 0.77 0.30 2.20 1.45 2.00 2.25 0.08
80.955 0.56 -1.93 0.21 0.77 0.30 2.20 1.45 2.00 2.23 0.08
81.939 0.56 -1.95 0.21 0.77 0.30 2.20 1.45 2.00 2.22 0.08
82.923 0.55 -1.96 0.21 0.76 0.30 2.20 1.45 2.00 2.21 0.08
83.661 0.55 -1.97 0.21 0.76 0.30 2.20 1.45 2.00 2.20 0.08

Table G7. Results from Yoshimine et al. (2006) and Idriss and Boulanger (2008) calculations.

z FSpresentation presentationlim presentationmax presentationv presentation presentation Comments:
For Event #1 – amax=0.1, Mw=6.5, z=Depth [ft], FSpresentation =Factor of Safety, presentationlim =Limiting shear strain, presentationmax=Maximum shear strain, presentationv=Volumetric strain, presentation = presentations = Incremental soil settlement [in], presentation = s1D = Cumulative soil settlement from bottom of soil profile top of soil profile [in].

Values collected every presentationz=0.164ft but reported herein every presentationz=0.984ft (except for first and last values).

See definition of variables on previous page.

Values collected every 0.164ft but reported herein every 0.984ft (except for first and last values).
0.163 0.94 0.95 0.00 0.00 0.00 2.46
1.148 0.94 0.73 0.00 0.00 0.00 2.46
2.133 0.94 0.75 0.00 0.00 0.00 2.46
3.117 0.94 0.90 0.02 0.02 0.03 2.43
4.101 0.94 0.67 0.01 0.01 0.02 2.32
5.085 0.94 0.77 0.00 0.00 0.00 2.30
6.070 0.94 0.73 0.00 0.00 0.00 2.30
7.054 0.94 0.70 0.00 0.00 0.00 2.30
8.038 0.94 0.66 0.00 0.00 0.00 2.30
9.022 0.94 0.73 0.00 0.00 0.00 2.30
10.007 0.94 0.58 0.07 0.04 0.08 2.15
10.991 0.87 0.40 0.02 0.01 0.01 1.84
11.975 0.85 0.37 0.01 0.01 0.01 1.77
12.959 0.86 0.38 0.01 0.01 0.01 1.70
13.944 0.82 0.34 0.01 0.01 0.01 1.63
14.928 0.60 0.19 0.00 0.00 0.00 1.58
15.912 0.37 0.12 0.00 0.00 0.00 1.57
16.896 0.37 0.12 0.00 0.00 0.00 1.57
17.881 0.66 0.22 0.01 0.00 0.00 1.57
18.865 0.69 0.24 0.01 0.00 0.01 1.54
19.849 0.90 0.45 0.02 0.01 0.02 1.47
20.833 0.87 0.40 0.02 0.01 0.01 1.20
21.818 0.90 0.44 0.02 0.01 0.02 1.09
22.802 0.87 0.40 0.02 0.01 0.01 0.98
23.786 0.89 0.43 0.02 0.01 0.02 0.89
24.770 0.89 0.43 0.02 0.01 0.02 0.80
25.755 0.90 0.45 0.02 0.01 0.02 0.70
26.739 0.89 0.44 0.02 0.01 0.02 0.61
27.723 0.91 0.47 0.02 0.01 0.02 0.51
28.707 0.91 0.48 0.02 0.01 0.02 0.40
29.692 0.89 0.42 0.02 0.01 0.01 0.28
30.676 0.83 0.34 0.01 0.00 0.01 0.21
31.660 0.81 0.33 0.01 0.00 0.01 0.15
32.644 0.75 0.28 0.01 0.00 0.01 0.11
33.629 0.78 0.30 0.01 0.00 0.01 0.07
34.613 0.69 0.24 0.01 0.00 0.00 0.03
35.597 0.65 0.22 0.01 0.00 0.00 0.01
36.581 0.33 0.11 0.00 0.00 0.00 0.00
37.566 -0.19 0.04 0.00 0.00 0.00 0.00
38.550 -0.08 0.05 0.00 0.00 0.00 0.00
39.561 -0.13 0.04 0.00 0.00 0.00 0.00
40.546 -0.40 0.02 0.00 0.00 0.00 0.00
41.530 -0.36 0.03 0.00 0.00 0.00 0.00
42.569 -0.40 0.02 0.00 0.00 0.00 0.00
43.553 -0.67 0.01 0.00 0.00 0.00 0.00
44.537 -0.83 0.01 0.00 0.00 0.00 0.00
45.549 -1.01 0.00 0.00 0.00 0.00 0.00
46.533 -0.53 0.02 0.00 0.00 0.00 0.00
47.517 -0.28 0.03 0.00 0.00 0.00 0.00
48.529 -0.39 0.03 0.00 0.00 0.00 0.00
49.513 -0.82 0.01 0.00 0.00 0.00 0.00
50.498 -1.01 0.00 0.00 0.00 0.00 0.00
51.482 -0.81 0.01 0.00 0.00 0.00 0.00
52.466 -0.67 0.01 0.00 0.00 0.00 0.00
53.450 -0.55 0.02 0.00 0.00 0.00 0.00
54.435 -0.23 0.04 0.00 0.00 0.00 0.00
55.474 -0.57 0.02 0.00 0.00 0.00 0.00
56.458 -0.62 0.01 0.00 0.00 0.00 0.00
57.442 -0.53 0.02 0.00 0.00 0.00 0.00
58.481 -0.83 0.01 0.00 0.00 0.00 0.00
59.465 -1.28 0.00 0.00 0.00 0.00 0.00
60.449 -1.28 0.00 0.00 0.00 0.00 0.00
61.488 -1.28 0.00 0.00 0.00 0.00 0.00
62.473 -1.28 0.00 0.00 0.00 0.00 0.00
63.457 -1.28 0.00 0.00 0.00 0.00 0.00
64.469 -1.28 0.00 0.00 0.00 0.00 0.00
65.518 -1.28 0.00 0.00 0.00 0.00 0.00
66.503 -1.28 0.00 0.00 0.00 0.00 0.00
67.626 -1.28 0.00 0.00 0.00 0.00 0.00
68.611 -1.24 0.00 0.00 0.00 0.00 0.00
69.595 -1.03 0.00 0.00 0.00 0.00 0.00
70.579 -0.89 0.01 0.00 0.00 0.00 0.00
71.563 -0.85 0.01 0.00 0.00 0.00 0.00
72.548 -0.29 0.03 0.00 0.00 0.00 0.00
73.710 -0.60 0.02 0.00 0.00 0.00 0.00
74.694 -1.28 0.00 0.00 0.00 0.00 0.00
75.678 -1.28 0.00 0.00 0.00 0.00 0.00
77.018 -1.25 0.00 0.00 0.00 0.00 0.00
78.002 -0.33 0.03 0.00 0.00 0.00 0.00
78.986 -1.28 0.00 0.00 0.00 0.00 0.00
79.970 -1.28 0.00 0.00 0.00 0.00 0.00
80.955 -1.28 0.00 0.00 0.00 0.00 0.00
81.939 -1.28 0.00 0.00 0.00 0.00 0.00
82.923 -1.28 0.00 0.00 0.00 0.00 0.00
83.661 -1.28 0.00 0.00 0.00 0.00 0.00

Table G8. Results from ALLCPT Pile Capacity Analysis and calculations (reported in imperial units).

z Qs Qb ∆Qs QwUTL R Min(Q,R) Comments:
z=Depth [ft], Qs=Summation of side resistance from ALLCPT pile capacity analysis [tons], Qb=End resistance from ALLCPT pile capacity analysis [tons], ∆Q=Incremental side resistance [tons], QwUTL=Load in pile with unfactored top load [tons], R=Resistance in pile [tons], Min(Q,R)=Load [tons] used to develop combination curve to identify the location of the neutral plane. Values calculated every ∆z=0.164ft but reported herein every ∆z=0.984ft (except for first and last values).

See definition of variables on previous page.
0.164 0.000 0.000 0.000 107.000 378.116 107.000
1.148 0.326 5.530 0.045 107.326 377.835 107.326
2.133 0.854 6.935 0.124 107.854 377.385 107.854
3.117 1.641 7.497 0.191 108.641 376.666 108.641
4.101 2.630 7.171 0.079 109.630 375.564 109.630
5.085 3.181 5.868 0.090 110.181 375.024 110.181
6.070 3.833 3.631 0.124 110.833 374.406 110.833
7.054 4.564 4.069 0.124 111.564 373.676 111.564
8.038 5.294 5.418 0.124 112.294 372.945 112.294
9.022 6.036 10.094 0.124 113.036 372.203 113.036
10.007 6.789 18.749 0.157 113.789 371.484 113.789
10.991 8.194 26.977 0.225 115.194 370.146 115.194
11.975 9.611 33.778 0.225 116.611 368.730 116.611
12.959 11.139 38.959 0.270 118.139 367.246 118.139
13.944 12.735 45.344 0.281 119.735 365.661 119.735
14.928 14.298 52.763 0.303 121.298 364.121 121.298
15.912 16.456 59.934 0.393 123.456 362.053 123.456
16.896 18.850 62.182 0.393 125.850 359.659 125.850
17.881 21.065 58.462 0.326 128.065 357.377 128.065
18.865 23.077 51.054 0.337 130.077 355.376 130.077
19.849 24.605 42.039 0.247 131.605 353.758 131.605
20.833 26.157 36.498 0.225 133.157 352.184 133.157
21.818 27.494 35.205 0.225 134.494 350.846 134.494
22.802 28.933 36.486 0.292 135.933 349.475 135.933
23.786 30.563 39.162 0.225 137.563 347.778 137.563
24.770 31.968 41.039 0.236 138.968 346.384 138.968
25.755 33.395 42.095 0.236 140.395 344.956 140.395
26.739 34.800 43.624 0.259 141.800 343.574 141.800
27.723 36.205 46.153 0.247 143.205 342.157 143.205
28.707 37.723 49.874 0.270 144.723 340.662 144.723
29.692 39.387 54.617 0.303 146.387 339.033 146.387
30.676 41.275 59.608 0.337 148.275 337.178 148.275
31.660 43.354 64.262 0.348 150.354 335.110 150.354
32.644 45.580 68.724 0.382 152.580 332.918 152.580
33.629 47.862 73.951 0.371 154.862 330.625 154.862
34.613 50.267 82.280 0.416 157.267 328.264 157.267
35.597 52.943 74.783 0.450 159.943 325.623 159.943
36.581 55.888 85.675 0.540 162.888 322.768 162.888
37.566 59.990 96.713 0.731 166.990 318.856 166.990
38.550 64.441 105.143 0.708 171.441 314.382 171.441
39.567 69.039 111.483 0.742 176.039 309.819 176.039
40.551 73.782 116.024 0.821 180.782 305.154 180.782
41.535 78.515 120.824 0.798 185.515 300.399 185.515
42.585 83.865 127.545 0.821 190.865 295.071 190.865
43.570 89.148 133.019 0.910 196.148 289.878 196.148
44.554 94.847 135.526 0.955 201.847 284.224 201.847
45.538 100.849 135.616 0.978 207.849 278.244 207.849
46.522 106.616 134.121 0.899 213.616 272.399 213.616
47.507 s 111.719 133.626 s 0.832 218.719 267.229 , 218.719
48.524 116.755 135.919 0.843 223.755 262.204 223.755
49.508 122.431 140.202 0.978 229.431 256.663 229.431
50.492 128.355 143.687 0.989 235.355 250.750 235.355
51.476 134.278 144.091 0.989 241.278 244.826 241.278
52.461 140.135 142.237 0.967 247.135 238.948 238.948
53.445 145.912 139.899 0.944 252.912 233.148 233.148
54.429 151.364 138.606 0.877 258.364 227.629 227.629
55.479 157.220 139.955 0.955 264.220 221.851 221.851
56.463 163.133 143.777 0.989 270.133 215.972 215.972
57.448 168.978 152.364 0.978 275.978 210.116 210.116
58.465 174.991 168.966 0.978 281.991 204.102 204.102
59.449 180.915 185.962 0.989 287.915 198.190 198.190
60.433 186.839 201.328 0.989 293.839 192.266 192.266
61.483 193.088 214.479 0.989 300.088 186.016 186.016
62.467 199.001 218.896 0.978 306.001 180.093 180.093
63.451 204.925 217.424 0.989 311.925 174.180 174.180
64.469 211.006 214.119 0.978 318.006 168.088 168.088
65.518 217.323 207.409 1.383 324.323 162.175 162.175
66.503 223.247 199.833 0.989 330.247 155.858 155.858
67.618 230.002 190.042 0.989 337.002 149.103 149.103
68.602 235.926 181.117 0.989 342.926 143.179 143.179
69.587 241.838 174.148 0.978 348.838 137.255 137.255
70.571 247.762 167.809 0.989 354.762 131.343 131.343
71.555 253.686 165.066 0.989 360.686 125.419 125.419
72.539 259.598 167.033 0.978 366.598 119.495 119.495
73.720 266.590 175.486 0.989 373.590 112.515 112.515
74.705 272.513 182.140 0.989 379.513 106.591 106.591
75.689 278.426 185.614 0.978 385.426 100.668 100.668
77.001 286.485 187.828 0.989 393.485 92.619 92.619
77.986 292.330 190.481 0.989 399.330 86.774 86.774
78.970 298.254 198.034 0.989 405.254 80.851 80.851
79.987 304.178 206.712 0.989 411.178 74.927 74.927
80.971 310.090 213.748 0.989 417.090 69.015 69.015
81.923 316.014 216.120 0.989 423.014 63.091 63.091
82.907 321.938 213.692 0.989 428.938 57.167 57.167
83.825 327.356 209.612 0.978 434.356 51.738 51.738
84.810 333.279 205.846 0.989 440.279 45.826 45.826
85.794 339.192 203.531 0.978 446.192 39.902 39.902
86.778 345.115 202.418 0.989 452.115 33.989 33.989
87.434 349.061 202.384 0.989 456.061 30.044 30.044

Table G9. Results from ALLCPT Pile Capacity Analysis and calculations (reported in imperial units).

z Qs Qb ∆Qs QwUTL R Min(Q,R) Comments:
z=Depth [ft], Qs=Summation of side resistance from ALLCPT pile capacity analysis [tons], Qb=End resistance from ALLCPT pile capacity analysis [tons], ∆Q=Incremental side resistance [tons], QwUTL=Load in pile with unfactored top load [tons], R=Resistance in pile [tons], Min(Q,R)=Load [tons] used to develop combination curve to identify the location of the neutral plane.

Values calculated every ∆z=0.164ft but reported herein every ∆z=0.984ft (except for first and last values).

See definition of variables on previous page.
0.164 0.000 0.000 0.000 107.000 378.116 107.000
1.148 0.326 5.530 0.045 107.326 377.835 107.326
2.133 0.854 6.935 0.124 107.854 377.385 107.854
3.117 1.641 7.497 0.191 108.641 376.666 108.641
4.101 2.630 7.171 0.079 109.630 375.564 109.630
5.085 3.181 5.868 0.090 110.181 375.024 110.181
6.070 3.833 3.631 0.124 110.833 374.406 110.833
7.054 4.564 4.069 0.124 111.564 373.676 111.564
8.038 5.294 5.418 0.124 112.294 372.945 112.294
9.022 6.036 10.094 0.124 113.036 372.203 113.036
10.007 6.789 18.749 0.157 113.789 371.484 113.789
10.991 8.194 26.977 0.225 115.194 370.146 115.194
11.975 9.611 33.778 0.225 116.611 368.730 116.611
12.959 11.139 38.959 0.270 118.139 367.246 118.139
13.944 12.735 45.344 0.281 119.735 365.661 119.735
14.928 14.298 52.763 0.303 121.298 364.121 121.298
15.912 16.456 59.934 0.393 123.456 362.053 123.456
16.896 18.850 62.182 0.393 125.850 359.659 125.850
17.881 21.065 58.462 0.326 128.065 357.377 128.065
18.865 23.077 51.054 0.337 130.077 355.376 130.077
19.849 24.605 42.039 0.247 131.605 353.758 131.605
20.833 26.157 36.498 0.225 133.157 352.184 133.157
21.818 27.494 35.205 0.225 134.494 350.846 134.494
22.802 28.933 36.486 0.292 135.933 349.475 135.933
23.786 30.563 39.162 0.225 137.563 347.778 137.563
24.770 31.968 41.039 0.236 138.968 346.384 138.968
25.755 33.395 42.095 0.236 140.395 344.956 140.395
26.739 34.800 43.624 0.259 141.800 343.574 141.800
27.723 36.205 46.153 0.247 143.205 342.157 143.205
28.707 37.723 49.874 0.270 144.723 340.662 144.723
29.692 39.387 54.617 0.303 146.387 339.033 146.387
30.676 41.275 59.608 0.337 148.275 337.178 148.275
31.660 43.354 64.262 0.348 150.354 335.110 150.354
32.644 45.580 68.724 0.382 152.580 332.918 152.580
33.629 47.862 73.951 0.371 154.862 330.625 154.862
34.613 50.267 82.280 0.416 157.267 328.264 157.267
35.597 52.943 74.783 0.450 159.943 325.623 159.943
36.581 55.888 85.675 0.540 162.888 322.768 162.888
37.566 59.990 96.713 0.731 166.990 318.856 166.990
38.550 64.441 105.143 0.708 171.441 314.382 171.441
39.567 69.039 111.483 0.742 176.039 309.819 176.039
40.551 73.782 116.024 0.821 180.782 305.154 180.782
41.535 78.515 120.824 0.798 185.515 300.399 185.515
42.585 83.865 127.545 0.821 190.865 295.071 190.865
43.570 89.148 133.019 0.910 196.148 289.878 196.148
44.554 94.847 135.526 0.955 201.847 284.224 201.847
45.538 100.849 135.616 0.978 207.849 278.244 207.849
46.522 106.616 134.121 0.899 213.616 272.399 213.616
47.507 111.719 133.626 0.832 218.719 267.229 218.719
48.524 116.755 135.919 0.843 223.755 262.204 223.755
49.508 122.431 140.202 0.978 229.431 256.663 229.431
50.492 128.355 143.687 0.989 235.355 250.750 235.355
51.476 134.278 144.091 0.989 241.278 244.826 241.278
52.461 140.135 142.237 0.967 247.135 238.948 238.948
53.445 145.912 139.899 0.944 252.912 233.148 233.148
54.429 151.364 138.606 0.877 258.364 227.629 227.629
55.479 157.220 139.955 0.955 264.220 221.851 221.851
56.463 163.133 143.777 0.989 270.133 215.972 215.972
57.448 168.978 152.364 0.978 275.978 210.116 210.116
58.465 174.991 168.966 0.978 281.991 204.102 204.102
59.449 180.915 185.962 0.989 287.915 198.190 198.190
60.433 186.839 201.328 0.989 293.839 192.266 192.266
61.483 193.088 214.479 0.989 300.088 186.016 186.016
62.467 199.001 218.896 0.978 306.001 180.093 180.093
63.451 204.925 217.424 0.989 311.925 174.180 174.180
64.469 211.006 214.119 0.978 318.006 168.088 168.088
65.518 217.323 207.409 1.383 324.323 162.175 162.175
66.503 223.247 199.833 0.989 330.247 155.858 155.858
67.618 230.002 190.042 0.989 337.002 149.103 149.103
68.602 235.926 181.117 0.989 342.926 143.179 143.179
69.587 241.838 174.148 0.978 348.838 137.255 137.255
70.571 247.762 167.809 0.989 354.762 131.343 131.343
71.555 253.686 165.066 0.989 360.686 125.419 125.419
72.539 259.598 167.033 0.978 366.598 119.495 119.495
73.720 266.590 175.486 0.989 373.590 112.515 112.515
74.705 272.513 182.140 0.989 379.513 106.591 106.591
75.689 278.426 185.614 0.978 385.426 100.668 100.668
77.001 286.485 187.828 0.989 393.485 92.619 92.619
77.986 292.330 190.481 0.989 399.330 86.774 86.774
78.970 298.254 198.034 0.989 405.254 80.851 80.851
79.987 304.178 206.712 0.989 411.178 74.927 74.927
80.971 310.090 213.748 0.989 417.090 69.015 69.015
81.923 316.014 216.120 0.989 423.014 63.091 63.091
82.907 321.938 213.692 0.989 428.938 57.167 57.167
83.825 327.356 209.612 0.978 434.356 51.738 51.738
84.810 333.279 205.846 0.989 440.279 45.826 45.826
85.794 339.192 203.531 0.978 446.192 39.902 39.902
87.434 349.061 202.384 0.989 456.061 30.044 30.044

Table G10. Results from ALLCPT/TZPILE and additional spreadsheet calculations.

z Min(Q,R) presentationp Comments:
0.5 106.95 0.238 z=Depth [ft], Min(Q,R)=Load used to develop combination curve to identify the location of the neutral plane, presentationp =pile settlement.
1.5 107.20 0.235
2.5 107.50 0.232
3.5 107.85 0.229
4.5 108.25 0.226
5.5 108.70 0.223
6.5 109.25 0.220
7.5 109.80 0.217
8.5 110.45 0.215
9.5 111.15 0.212
10.4 111.95 0.209
11.4 112.90 0.206
12.4 113.85 0.203
13.4 114.85 0.199
14.4 115.90 0.196
15.4 117.05 0.193
16.4 118.20 0.190
17.4 119.45 0.187
18.4 120.75 0.184
19.4 122.10 0.181
20.4 123.50 0.177
21.4 125.05 0.174
22.4 126.70 0.171
23.4 128.45 0.167
24.4 130.40 0.164
25.4 132.40 0.160
26.4 134.60 0.157
27.3 136.90 0.153
28.3 139.35 0.149
29.3 141.90 0.146
30.3 144.30 0.142
31.3 146.00 0.138
32.3 146.40 0.134
33.3 145.35 0.130
34.3 143.30 0.126
35.3 140.65 0.122
36.3 137.60 0.119
37.3 134.40 0.115
38.3 131.15 0.111
39.3 127.85 0.108
40.3 124.60 0.105
41.3 121.35 0.101
42.3 118.15 0.098
43.3 114.95 0.095
44.3 111.75 0.092
45.2 108.55 0.089
46.2 105.40 0.086
47.2 102.25 0.083
48.2 99.15 0.081
49.2 96.10 0.078
50.2 93.05 0.075
51.2 90.05 0.073
52.2 87.10 0.071
53.2 84.20 0.068
54.2 81.30 0.066 See definition of variables on previous page.
55.2 78.45 0.064
56.2 75.60 0.062
57.2 72.85 0.060
58.2 70.05 0.058
59.2 67.35 0.056
60.2 64.65 0.054
61.2 62.00 0.053
62.1 59.40 0.051
63.1 56.85 0.050
64.1 54.30 0.048
65.1 51.85 0.047
66.1 49.45 0.045
67.1 47.09 0.044
68.1 44.79 0.043
69.1 42.53 0.042
70.1 40.33 0.041
71.1 38.17 0.039
72.1 36.06 0.038
73.1 33.99 0.038
74.1 31.96 0.037
75.1 29.96 0.036
76.1 28.00 0.035
77.1 26.08 0.034
78.1 24.18 0.034
79.1 22.31 0.033
80.0 20.47 0.032
81.0 18.65 0.032
82.0 16.85 0.031
83.0 15.08 0.031
84.0 13.32 0.031
85.0 11.57 0.030
86.0 9.84 0.030
87.0 8.12 0.030

Table H1. Average CPT sounding record for the Blytheville, AR Test Site.

z fs qt u2 Comments:
z=Depth [ft],
fs=Sleeve friction [tsf],
qt=Tip resistance [tsf],
u2=Pore pressure [psi]

Values collected every presentationz=0.164ft but reported herein every presentationz=0.984ft (except for first and last values).

See definition of variables on previous page.
0.163 0.163 5.990 -0.080
1.148 0.343 6.644 -4.346
2.133 0.321 5.024 -3.507
3.117 0.232 30.509 -0.474
4.101 0.283 8.452 0.052
5.085 0.259 3.602 1.817
6.070 0.279 4.583 2.308
7.054 0.321 5.619 3.310
8.038 0.314 7.221 3.799
9.022 0.178 4.880 4.400
10.007 0.153 26.554 4.783
10.991 0.283 55.585 4.052
11.975 0.380 57.579 3.349
12.959 0.299 41.164 3.934
13.944 0.332 44.718 1.888
14.928 0.412 80.277 0.862
15.912 0.485 99.291 1.548
16.896 0.549 102.243 3.270
17.881 0.435 82.030 3.717
18.865 0.413 81.470 3.345
19.849 0.259 50.410 3.707
20.833 0.233 30.238 3.041
21.818 0.294 56.976 4.014
22.802 0.274 48.270 7.523
23.786 0.306 58.057 7.823
24.770 0.316 59.929 8.438
25.755 0.303 59.692 9.553
26.739 0.271 53.975 11.429
27.723 0.290 60.599 11.954
28.707 0.307 68.630 12.150
29.692 0.328 76.193 13.576
30.676 0.350 86.086 13.470
31.660 0.336 89.231 14.176
32.644 0.419 98.042 15.180
33.629 0.396 96.314 15.467
34.613 0.372 107.343 15.953
35.597 0.410 113.077 16.297
36.581 0.508 143.899 16.744
37.566 0.666 185.609 16.821
38.550 0.608 179.809 17.287
39.561 0.656 185.487 17.292
40.546 0.840 206.957 17.771
41.530 0.866 205.632 16.569
42.569 0.925 210.830 18.586
43.553 0.995 232.122 18.903
44.537 1.028 245.359 19.529
45.549 1.077 259.473 19.777
46.533 1.035 227.953 20.100
47.517 1.000 211.495 20.318
48.529 0.967 220.913 20.881
49.513 1.070 253.789 20.833
50.498 0.895 268.331 21.315
51.482 0.757 256.276 21.870
52.466 0.680 247.751 22.581
53.450 0.717 240.933 22.914
54.435 0.653 219.256 23.600
55.474 0.648 245.683 24.005
56.458 0.688 250.659 24.232
57.442 0.767 245.872 24.346
58.481 0.847 268.145 25.168
59.465 1.130 329.023 25.588
60.449 1.317 379.164 25.735
61.488 1.457 392.789 24.920
62.473 1.366 378.497 26.585
63.457 1.462 393.080 26.816
64.469 1.218 375.347 27.283
65.518 1.017 343.502 27.677
66.503 1.156 355.590 28.325
67.626 1.355 342.052 29.154
68.611 0.844 311.256 29.417
69.595 0.687 298.517 29.799
70.579 0.727 290.580 30.460
71.563 0.842 289.159 30.683
72.548 0.729 249.830 31.720
73.710 0.863 273.813 31.332
74.694 0.901 348.687 31.865
75.678 1.635 376.848 31.968
77.018 0.980 322.823 32.031
78.002 0.885 260.040 28.425
78.986 1.091 375.017 32.978
79.970 0.986 345.087 32.455
80.955 0.788 374.134 33.192
81.939 1.274 374.530 34.805
82.923 1.856 404.210 33.297
83.661 1.557 351.657 29.414

Table H2. Results from Yoshimine et at. (2006) and Idriss and Boulanger (2008) calculations.

Event 1 Event 2 Comments:
For Event #1 amax=0.1, Mw=6.5, For Event #2 amax=0.4, Mw=7.7, z=Depth [ft], δ = ∆s = Incremental soil settlement [in], Σδ = s1D = Cumulative soil settlement from bottom of soil profile top of soil profile [in].

Values collected every ∆z=0.164ft but reported herein every ∆z=0.984ft (except for first and last values).

See definition of variables on previous page.

Values collected every 0.164ft but reported herein every 0.984ft (except for first and last values).
z δ Σδ δ Σδ
0.163 0.00 2.46 0.00 12.23
1.148 0.00 2.46 0.20 12.23
2.133 0.00 2.46 0.21 12.03
3.117 0.03 2.43 0.57 11.82
4.101 0.02 2.32 0.09 11.25
5.085 0.00 2.30 0.00 11.16
6.070 0.00 2.30 0.00 11.16
7.054 0.00 2.30 0.00 11.16
8.038 0.00 2.30 0.00 11.16
9.022 0.00 2.30 0.17 11.16
10.007 0.08 2.15 0.49 10.99
10.991 0.01 1.84 0.40 10.50
11.975 0.01 1.77 0.41 10.10
12.959 0.01 1.70 0.40 9.69
13.944 0.01 1.63 0.38 9.29
14.928 0.00 1.58 0.29 8.91
15.912 0.00 1.57 0.26 8.62
16.896 0.00 1.57 0.29 8.36
17.881 0.00 1.57 0.32 8.07
18.865 0.01 1.54 0.41 7.75
19.849 0.02 1.47 0.50 7.34
20.833 0.01 1.20 0.45 6.84
21.818 0.02 1.09 0.45 6.39
22.802 0.01 0.98 0.44 5.94
23.786 0.02 0.89 0.44 5.50
24.770 0.02 0.80 0.44 5.06
25.755 0.02 0.70 0.44 4.62
26.739 0.02 0.61 0.45 4.18
27.723 0.02 0.51 0.46 3.73
28.707 0.02 0.40 0.46 3.27
29.692 0.01 0.28 0.43 2.81
30.676 0.01 0.21 0.39 2.38
31.660 0.01 0.15 0.38 1.99
32.644 0.01 0.11 0.37 1.61
33.629 0.01 0.07 0.36 1.24
34.613 0.00 0.03 0.32 0.88
35.597 0.00 0.01 0.31 0.56
36.581 0.00 0.00 0.12 0.25
37.566 0.00 0.00 0.02 0.13
38.550 0.00 0.00 0.04 0.11
39.561 0.00 0.00 0.01 0.07
40.546 0.00 0.00 0.01 0.06
41.530 0.00 0.00 0.00 0.05
42.569 0.00 0.00 0.00 0.05
43.553 0.00 0.00 0.00 0.05
44.537 0.00 0.00 0.00 0.05
45.549 0.00 0.00 0.00 0.05
46.533 0.00 0.00 0.00 0.05
47.517 0.00 0.00 0.03 0.05
48.529 0.00 0.00 0.00 0.02
49.513 0.00 0.00 0.00 0.02
50.498 0.00 0.00 0.00 0.02
51.482 0.00 0.00 0.00 0.02
52.466 0.00 0.00 0.00 0.02
53.450 0.00 0.00 0.00 0.02
54.435 0.00 0.00 0.00 0.02
55.474 0.00 0.00 0.00 0.02
56.458 0.00 0.00 0.00 0.02
57.442 0.00 0.00 0.00 0.02
58.481 0.00 0.00 0.00 0.02
59.465 0.00 0.00 0.00 0.02
60.449 0.00 0.00 0.00 0.02
61.488 0.00 0.00 0.00 0.02
62.473 0.00 0.00 0.00 0.02
63.457 0.00 0.00 0.00 0.02
64.469 0.00 0.00 0.00 0.02
65.518 0.00 0.00 0.00 0.02
66.503 0.00 0.00 0.00 0.02
67.626 0.00 0.00 0.00 0.02
68.611 0.00 0.00 0.00 0.02
69.595 0.00 0.00 0.00 0.02
70.579 0.00 0.00 0.00 0.02
71.563 0.00 0.00 0.00 0.02
72.548 0.00 0.00 0.00 0.02
73.710 0.00 0.00 0.00 0.02
74.694 0.00 0.00 0.00 0.02
75.678 0.00 0.00 0.00 0.02
77.018 0.00 0.00 0.02 0.02
78.002 0.00 0.00 0.00 0.00
78.986 0.00 0.00 0.00 0.00
79.970 0.00 0.00 0.00 0.00
80.955 0.00 0.00 0.00 0.00
81.939 0.00 0.00 0.00 0.00
82.923 0.00 0.00 0.00 0.00
83.661 0.00 0.00 0.00 0.02

Table H3. Results from ALLCPT Pile Capacity Analysis and calculations (reported in imperial units).

z Qs Qb ∆Qs QwUTL R Min(Q,R) δEC Σδ Comments:
z=Depth [ft], Qs=Summation of side resistance from ALLCPT pile capacity analysis [tons], Qb=End resistance from ALLCPT pile capacity analysis [tons], ∆Q=Incremental side resistance [tons], QwUTL=Load in pile with unfactored top load [tons], R=Resistance in pile [tons], Min(Q,R)=Load [tons] used to develop combination curve to identify the location of the neutral plane. δEC=incremental elastic compression in pile [in], Σδ=pile settlement [in],

Values calculated every ∆z=0.164ft but reported herein every ∆z=0.984ft (except for first and last values).

See definition of variables on previous page.

Values calculated every ∆z=0.164ft but reported herein every ∆z=0.984ft (except for first and last values).
0.164 0.000 0.000 0.000 107.000 451.652 107.000 0.00051 0.399
1.148 0.326 8.329 0.045 107.326 451.371 107.326 0.00051 0.396
2.133 0.832 8.363 0.112 107.832 450.933 107.832 0.00051 0.393
3.117 1.607 8.711 0.191 108.607 450.236 108.607 0.00052 0.390
4.101 2.574 8.284 0.067 109.574 449.146 109.574 0.00052 0.387
5.085 3.125 7.857 0.090 110.125 448.617 110.125 0.00053 0.384
6.070 3.766 5.204 0.124 110.766 448.011 110.766 0.00053 0.381
7.054 4.474 5.216 0.124 111.474 447.302 111.474 0.00053 0.378
8.038 5.204 8.273 0.124 112.204 446.572 112.204 0.00054 0.374
9.022 5.924 13.275 0.112 112.924 445.841 112.924 0.00054 0.371
10.007 6.666 22.672 0.157 113.666 445.144 113.666 0.00054 0.368
10.991 8.048 31.406 0.214 115.048 443.818 115.048 0.00055 0.365
11.975 9.442 40.735 0.225 116.442 442.435 116.442 0.00056 0.361
12.959 10.937 48.177 0.270 117.937 440.985 117.937 0.00056 0.358
13.944 12.511 56.124 0.281 119.511 439.423 119.511 0.00057 0.355
14.928 14.039 64.026 0.303 121.039 437.917 121.039 0.00058 0.351
15.912 16.153 70.702 0.382 123.153 435.882 123.153 0.00059 0.348
16.896 18.513 72.613 0.393 125.513 433.533 125.513 0.00060 0.344
17.881 20.682 69.443 0.326 127.682 431.296 127.682 0.00061 0.340
18.865 22.661 61.407 0.337 129.661 429.329 129.661 0.00062 0.337
19.849 24.156 52.853 0.247 131.156 427.744 131.156 0.00063 0.333
20.833 25.684 46.727 0.225 132.684 426.193 132.684 0.00063 0.329
21.818 27.000 44.512 0.225 134.000 424.878 134.000 0.00064 0.325
22.802 28.405 44.714 0.281 135.405 423.529 135.405 0.00065 0.322
23.786 30.001 47.716 0.214 137.001 421.865 137.001 0.00065 0.318
24.770 31.383 50.031 0.225 138.383 420.494 138.383 0.00066 0.314
25.755 32.788 51.965 0.236 139.788 419.100 139.788 0.00067 0.310
26.739 34.160 54.089 0.247 141.160 417.740 141.160 0.00067 0.306
27.723 35.542 57.506 0.236 142.542 416.346 142.542 0.00068 0.302
28.707 37.026 61.879 0.259 144.026 414.885 144.026 0.00069 0.297
29.692 38.667 67.162 0.292 145.667 413.278 145.667 0.00070 0.293
30.676 40.522 72.962 0.337 147.522 411.468 147.522 0.00070 0.289
31.660 42.568 78.739 0.348 149.568 409.433 149.568 0.00071 0.285
32.644 44.748 84.719 0.371 151.748 407.275 151.748 0.00072 0.281
33.629 46.996 92.554 0.371 153.996 405.027 153.996 0.00074 0.276
34.613 49.346 103.176 0.405 156.346 402.712 156.346 0.00075 0.272
35.597 51.976 92.914 0.438 158.976 400.115 158.976 0.00076 0.267
36.581 54.876 105.177 0.540 161.876 397.316 161.876 0.00077 0.263
37.566 58.889 117.485 0.719 165.889 393.483 165.889 0.00079 0.258
38.550 63.272 127.669 0.708 170.272 389.088 170.272 0.00081 0.253
39.567 67.780 136.088 0.719 174.780 384.592 174.780 0.00083 0.248
40.551 72.433 143.383 0.809 179.433 380.028 179.433 0.00086 0.243
41.535 77.087 149.622 0.787 184.087 375.352 184.087 0.00088 0.238
42.585 82.336 155.883 0.798 189.336 370.114 189.336 0.00090 0.232
43.570 87.518 161.244 0.888 194.518 365.022 194.518 0.00093 0.227
44.554 93.116 164.234 0.933 200.116 359.470 200.116 0.00096 0.221
45.538 99.017 165.493 0.967 206.017 353.602 206.017 0.00098 0.215
46.522 104.682 165.482 0.899 211.682 347.869 211.682 0.00101 0.209
47.507 109.684 165.920 0.821 216.684 342.789 216.684 0.00103 0.203
48.524 114.619 168.157 0.821 221.619 337.854 221.619 0.00106 0.197
49.508 120.205 171.878 0.967 227.205 332.414 227.205 0.00108 0.191
50.492 126.017 174.463 0.967 233.017 326.602 233.017 0.00111 0.184
51.476 131.828 175.272 0.967 238.828 320.791 238.828 0.00114 0.177
52.461 137.583 174.710 0.955 244.583 315.025 244.583 0.00117 0.170
53.445 143.259 172.878 0.944 250.259 309.337 250.259 0.00119 0.163
54.429 148.599 171.473 0.854 255.599 303.908 255.599 0.00122 0.156
55.479 154.354 173.676 0.933 261.354 298.232 261.354 0.00125 0.148
56.463 160.165 180.488 0.978 267.165 292.465 267.165 0.00128 0.141
57.448 165.898 191.874 0.955 272.898 286.710 272.898 0.00130 0.133
58.465 171.810 208.342 0.978 278.810 280.820 278.810 0.00133 0.125
59.449 177.622 227.484 0.967 284.622 274.998 274.998 0.00131 0.117
60.433 183.433 244.907 0.967 290.433 269.186 269.186 0.00129 0.110
61.483 189.570 257.833 0.978 296.570 263.060 263.060 0.00126 0.102
62.467 195.381 264.353 0.967 302.381 257.238 257.238 0.00123 0.094
63.451 201.193 265.263 0.967 308.193 251.426 251.426 0.00120 0.087
64.469 207.161 261.161 0.967 314.161 245.458 245.458 0.00117 0.080
65.518 213.366 254.484 1.360 320.366 239.646 239.646 0.00114 0.073
66.503 219.177 245.413 0.967 326.177 233.442 233.442 0.00111 0.066
67.618 225.809 233.689 0.967 332.809 226.810 226.810 0.00108 0.060
68.602 231.621 223.112 0.967 338.621 220.998 220.998 0.00106 0.053
69.587 237.432 214.513 0.967 344.432 215.187 215.187 0.00103 0.047
71.063 246.155 206.465 0.967 353.155 206.465 206.465 0.00099 0.038

Table I1. Design soil properties for the Alaska gravel design example.

Material Depth z γ′ φ′ N1,60 σz Vs Es ν Ko Ir qn fs
Gravel 0 - 10 5 120 36 25 1200 735 428634 0.2 0.4 194 120021 161
Gravel 10 - 21 15.5 67 38 50 1937 935 106269 0.3 0.3 271 226386 256
Sand 21 - 30 25.5 55 33 11 2432 709 275908 0.2 0.4 72 140667 328
Gravel 30 - 36 33 65 38 47 2822 973 129277 0.3 0.3 226 300946 373
Silt 36 - 50 43 65 30 10 3732 725 307209 0.1 0.5 61 194674 500
Sand 50 - 65 57.5 63 38 41 4677 993 142671 0.3 0.3 151 405671 619
Gravel 65 - 96 80.5 60 38 29 6537 953 116535 0.3 0.3 88 431575 865
Gravel 96 - 124 110 60 35 16 8217 867 737490 0.2 0.4 51 409767 110
Gravel 12 - 136 130 52 33 10 8841 799 495616 0.2 0.4 35 367109 119
Gravel 13 - 200 168 75 40 60 1364 118 341389 0.3 0.3 113 102991 177
z=Midpoint depth of layer [ft], γ′=Effective unit weight [pcf], φ′=Friction angle [o], N1,60=Corrected SPT blow count [bpf], σz′=Effective vertical stress [psf], Vs=Shear wave velocity [ft/sec], Es=Young’s modulus of the soil, ν=Poisson’s ratio of the soil, K=Lateral earth pressure coefficient, Ir=Rigidity index, qn′=Nominal unit end bearing capacity [psf], fs=Nominal unit side resistance [psf]

Table I3. Results from calculations to find corrected shear wave velocity.

Elev z Soil Type γ′ σvo σvo Vs Vs1 FC Comments:
For Event #1 – amax=0.3, Mw=6.2, Elev=Elevation above sea level [t], z=Depth [ft], γ′=Effective unit weight [pcf], σvo=Vertical total stress [psf], σvo′=Vertical effective stress [psf], Vs=Shear wave velocity [fps], Vs1=Overburden corrected shear wave velocity [fps], FC=Fines Content.

See definition of variables on previous page.

See definition of variables on previous page.
1960 0 Gravel 120 0 0 735 - 0
1959 1 Gravel 120 120 120 735 1506 0
1958 2 Gravel 120 240 240 735 1267 0
1957 3 Gravel 120 360 360 735 1144 0
1956 4 Gravel 120 480 480 735 1065 0
1955 5 Gravel 120 600 600 735 1007 0
1954 6 Gravel 120 720 720 735 962 0
1953 7 Gravel 120 840 840 735 926 0
1952 8 Gravel 120 960 960 735 896 0
1951 9 Gravel 120 1080 1080 735 870 0
1950 10 Gravel 67 1209 1147 935 1090 31
1949 11 Gravel 67 1339 1214 935 1074 31
1948 12 Gravel 67 1468 1281 935 1060 31
1947 13 Gravel 67 1598 1348 935 1047 31
1946 14 Gravel 67 1727 1415 935 1034 31
1945 15 Gravel 67 1856 1482 935 1022 31
1944 16 Gravel 67 1986 1549 935 1011 31
1943 17 Gravel 67 2115 1616 935 1000 31
1942 18 Gravel 67 2245 1683 935 990 31
1941 19 Gravel 67 2374 1750 935 980 31
1940 20 Gravel 67 2503 1817 935 971 31
1939 21 Sand 55 2621 1872 709 731 6
1938 22 Sand 55 2738 1927 709 726 6
1937 23 Sand 55 2856 1982 709 721 6
1936 24 Sand 55 2973 2037 709 716 6
1935 25 Sand 55 3090 2092 709 711 6
1934 26 Sand 55 3208 2147 709 706 6
1933 27 Sand 55 3325 2202 709 702 6
1932 28 Sand 55 3443 2257 709 698 6
1931 29 Sand 55 3560 2312 709 693 6
1930 30 Gravel 65 3687 2377 973 945 6
1929 31 Gravel 65 3815 2442 973 939 6
1928 32 Gravel 65 3942 2507 973 933 6
1927 33 Gravel 65 4070 2572 973 927 6
1926 34 Gravel 65 4197 2637 973 921 6
1925 35 Gravel 65 4324 2702 973 915 6
1924 36 Silt 65 4452 2767 725 678 92
1923 37 Silt 65 4579 2832 725 674 92
1922 38 Silt 65 4707 2897 725 670 92
1921 39 Silt 65 4834 2962 725 667 92
1920 40 Silt 65 4961 3027 725 663 92
1919 41 Silt 65 5089 3092 725 659 92
1918 42 Silt 65 5216 3157 725 656 92
1917 43 Silt 65 5344 3222 725 653 92
1916 44 Silt 65 5471 3287 725 649 92
1915 45 Silt 65 5598 3352 725 646 92
1914 46 Silt 65 5726 3417 725 643 92
1913 47 Silt 65 5853 3482 725 640 92
1912 48 Silt 65 5981 3547 725 637 92
1911 49 Silt 65 6108 3612 725 634 92
1910 50 Sand 63 6233 3675 993 865 4
1909 51 Sand 63 6359 3738 993 861 4
1908 52 Sand 63 6484 3801 993 858 4
1907 53 Sand 63 6610 3864 993 854 4
1906 54 Sand 63 6735 3927 993 851 4
1905 55 Sand 63 6860 3990 993 847 4
1904 56 Sand 63 6986 4053 993 844 4
1903 57 Sand 63 7111 4116 993 841 4
1902 58 Sand 63 7237 4179 993 838 4
1901 59 Sand 63 7362 4242 993 835 4
1900 60 Sand 63 7487 4305 993 831 4
1899 61 Sand 63 7613 4368 993 828 4
1898 62 Sand 63 7738 4431 993 825 4
1897 63 Sand 63 7864 4494 993 823 4
1896 64 Sand 63 7989 4557 993 820 4
1895 65 Gravel 60 8111 4617 953 784 0
1894 66 Gravel 60 8234 4677 953 782 0
1893 67 Gravel 60 8356 4737 953 779 0
1892 68 Gravel 60 8479 4797 953 777 0
1891 69 Gravel 60 8601 4857 953 774 0
1890 70 Gravel 60 8723 4917 953 772 0
1889 71 Gravel 60 8846 4977 953 770 0
1888 72 Gravel 60 8968 5037 953 767 0
1887 73 Gravel 60 9091 5097 953 765 0
1886 74 Gravel 60 9213 5157 953 763 0
1885 75 Gravel 60 9335 5217 953 761 0
1884 76 Gravel 60 9458 5277 953 758 0
1883 77 Gravel 60 9580 5337 953 756 0
1882 78 Gravel 60 9703 5397 953 754 0
1881 79 Gravel 60 9825 5457 953 752 0
1880 80 Gravel 60 9947 5517 953 750 0
1879 81 Gravel 60 10070 5577 953 748 0
1878 82 Gravel 60 10192 5637 953 746 0
1877 83 Gravel 60 10315 5697 953 744 0
1876 84 Gravel 60 10437 5757 953 742 0
1875 85 Gravel 60 10559 5817 953 740 0
1874 86 Gravel 60 10682 5877 953 738 0
1873 87 Gravel 60 10804 5937 953 736 0
1872 88 Gravel 60 10927 5997 953 734 0
1871 89 Gravel 60 11049 6057 953 733 0
1870 90 Gravel 60 11171 6117 953 731 0
1869 91 Gravel 60 11294 6177 953 729 0
1868 92 Gravel 60 11416 6237 953 727 0
1867 93 Gravel 60 11539 6297 953 726 0
1866 94 Gravel 60 11661 6357 953 724 0
1865 95 Gravel 60 11783 6417 953 722 0
1864 96 Gravel 60 11906 6477 867 655 0
1863 97 Gravel 60 12028 6537 867 654 0
1862 98 Gravel 60 12151 6597 867 652 0
1861 99 Gravel 60 12273 6657 867 651 0
1860 100 Gravel 60 12395 6717 867 650 0
1859 101 Gravel 60 12518 6777 867 648 0
1858 102 Gravel 60 12640 6837 867 647 0
1857 103 Gravel 60 12763 6897 867 645 0
1856 104 Gravel 60 12885 6957 867 644 0
1855 105 Gravel 60 13007 7017 867 642 0
1854 106 Gravel 60 13130 7077 867 641 0
1853 107 Gravel 60 13252 7137 867 640 0
1852 108 Gravel 60 13375 7197 867 638 0
1851 109 Gravel 60 13497 7257 867 637 0
1850 110 Gravel 60 13619 7317 867 636 0
1849 111 Gravel 60 13742 7377 867 634 0
1848 112 Gravel 60 13864 7437 867 633 0
1847 113 Gravel 60 13987 7497 867 632 0
1846 114 Gravel 60 14109 7557 867 631 0
1845 115 Gravel 60 14231 7617 867 629 0
1844 116 Gravel 60 14354 7677 867 628 0
1843 117 Gravel 60 14476 7737 867 627 0
1842 118 Gravel 60 14599 7797 867 626 0
1841 119 Gravel 60 14721 7857 867 625 0
1840 120 Gravel 60 14843 7917 867 623 0
1839 121 Gravel 60 14966 7977 867 622 0
1838 122 Gravel 60 15088 8037 867 621 0
1837 123 Gravel 60 15211 8097 867 620 0
1836 124 Gravel 52 15325 8149 799 570 0
1835 125 Gravel 52 15439 8201 799 569 0
1834 126 Gravel 52 15554 8253 799 569 0
1833 127 Gravel 52 15668 8305 799 568 0
1832 128 Gravel 52 15783 8357 799 567 0
1831 129 Gravel 52 15897 8409 799 566 0
1830 130 Gravel 52 16011 8461 799 565 0
1829 131 Gravel 52 16126 8513 799 564 0
1828 132 Gravel 52 16240 8565 799 563 0
1827 133 Gravel 52 16355 8617 799 562 0
1826 134 Gravel 52 16469 8669 799 562 0
1825 135 Gravel 52 16583 8721 799 561 0
1824 136 Gravel 75 16721 8796 1187 831 0
1823 137 Gravel 75 16858 8871 1187 830 0
1822 138 Gravel 75 16996 8946 1187 828 0
1821 139 Gravel 75 17133 9021 1187 826 0
1820 140 Gravel 75 17270 9096 1187 824 0
1819 141 Gravel 75 17408 9171 1187 823 0
1818 142 Gravel 75 17545 9246 1187 821 0
1817 143 Gravel 75 17683 9321 1187 819 0
1816 144 Gravel 75 17820 9396 1187 818 0
1815 145 Gravel 75 17957 9471 1187 816 0
1814 146 Gravel 75 18095 9546 1187 814 0
1813 147 Gravel 75 18232 9621 1187 813 0
1812 148 Gravel 75 18370 9696 1187 811 0
1811 149 Gravel 75 18507 9771 1187 810 0
1810 150 Gravel 75 18644 9846 1187 808 0

Table I4. Results from calculations to find factor of safety against liquefaction (Rollins et al., 2022).

Elev z α(z) β(z) rd CRR7.5 CRR CSR FSLiq Comments:
For Event #1 – amax=0.3, Mw=6.2, MSF=Magnitude scaling factor=1.5, PL=probability of liquefaction triggering=0.15, Elev=Elevation above sea level [t], z=Depth [ft], rd=Shear stress reduction coefficient, α(z)=Term used to find rd, β(z)=Term used to find rd, CRR7.5 = standardized cyclic resistance corresponding to a moment magnitude earthquake of 7.5, with 15 uniform shear stress cycles, and one atmosphere of pressure, CRR=Cyclic resistance ratio for the magnitude and effective stress of interest, CSR=Cyclic stress ratio for the magnitude and effective stress of interest, FSLiq=Factor of safety against liquefaction.

See definition of variables on previous page.

See definition of variables on previous page.
1960 0 0.02 0.00 1.00 - -
1959 1 0.00 0.00 1.00 1.00 1.50 0.20 2.00
1958 2 -0.01 0.00 1.00 1.00 1.50 0.20 2.00
1957 3 -0.02 0.00 1.00 1.00 1.50 0.19 2.00
1956 4 -0.04 0.00 0.99 1.00 1.50 0.19 2.00
1955 5 -0.05 0.01 0.99 0.92 1.38 0.19 2.00
1954 6 -0.07 0.01 0.98 0.69 1.03 0.19 2.00
1953 7 -0.08 0.01 0.98 0.55 0.83 0.19 2.00
1952 8 -0.10 0.01 0.97 0.47 0.70 0.19 2.00
1951 9 -0.12 0.01 0.97 0.41 0.61 0.19 2.00
1950 10 -0.14 0.02 0.96 1.00 1.50 0.20 2.00
1949 11 -0.16 0.02 0.96 1.00 1.50 0.21 2.00
1948 12 -0.17 0.02 0.95 1.00 1.50 0.21 2.00
1947 13 -0.19 0.02 0.94 1.00 1.50 0.22 2.00
1946 14 -0.22 0.02 0.94 1.00 1.50 0.22 2.00
1945 15 -0.24 0.03 0.93 1.00 1.50 0.23 2.00
1944 16 -0.26 0.03 0.93 0.94 1.41 0.23 2.00
1943 17 -0.28 0.03 0.92 0.87 1.32 0.23 2.00
1942 18 -0.30 0.03 0.91 0.82 1.23 0.24 2.00
1941 19 -0.32 0.04 0.91 0.77 1.16 0.24 2.00
1940 20 -0.35 0.04 0.90 0.73 1.09 0.24 2.00
1939 21 -0.37 0.04 0.89 0.23 0.34 0.24 1.39
1938 22 -0.40 0.04 0.89 0.22 0.33 0.25 1.36
1937 23 -0.42 0.05 0.88 0.22 0.33 0.25 1.32
1936 24 -0.45 0.05 0.87 0.21 0.32 0.25 1.30
1935 25 -0.47 0.05 0.87 0.21 0.32 0.25 1.27
1934 26 -0.50 0.06 0.86 0.21 0.31 0.25 1.25
1933 27 -0.52 0.06 0.85 0.20 0.31 0.25 1.23
1932 28 -0.55 0.06 0.85 0.20 0.30 0.25 1.21
1931 29 -0.58 0.06 0.84 0.20 0.30 0.25 1.19
1930 30 -0.60 0.07 0.83 0.62 0.93 0.25 2.00
1929 31 -0.63 0.07 0.82 0.60 0.90 0.25 2.00
1928 32 -0.66 0.07 0.82 0.57 0.86 0.25 2.00
1927 33 -0.69 0.08 0.81 0.56 0.84 0.25 2.00
1926 34 -0.72 0.08 0.80 0.54 0.81 0.25 2.00
1925 35 -0.74 0.08 0.80 0.52 0.78 0.25 2.00
1924 36 -0.77 0.09 0.79 0.19 0.29 0.25 1.15
1923 37 -0.80 0.09 0.78 0.19 0.28 0.25 1.14
1922 38 -0.83 0.09 0.77 0.19 0.28 0.25 1.14
1921 39 -0.86 0.10 0.77 0.18 0.28 0.24 1.13
1920 40 -0.89 0.10 0.76 0.18 0.27 0.24 1.12
1919 41 -0.92 0.10 0.75 0.18 0.27 0.24 1.12
1918 42 -0.95 0.11 0.75 0.18 0.27 0.24 1.11
1917 43 -0.98 0.11 0.74 0.18 0.26 0.24 1.11
1916 44 -1.00 0.11 0.73 0.17 0.26 0.24 1.10
1915 45 -1.03 0.11 0.72 0.17 0.26 0.24 1.10
1914 46 -1.06 0.12 0.72 0.17 0.26 0.23 1.10
1913 47 -1.09 0.12 0.71 0.17 0.26 0.23 1.10
1912 48 -1.12 0.12 0.70 0.17 0.25 0.23 1.09
1911 49 -1.15 0.13 0.70 0.17 0.25 0.23 1.09
1910 50 -1.18 0.13 0.69 0.40 0.60 0.23 2.00
1909 51 -1.21 0.13 0.68 0.39 0.59 0.23 2.00
1908 52 -1.24 0.14 0.68 0.39 0.58 0.23 2.00
1907 53 -1.27 0.14 0.67 0.38 0.57 0.22 2.00
1906 54 -1.29 0.14 0.67 0.37 0.56 0.22 2.00
1905 55 -1.32 0.15 0.66 0.37 0.55 0.22 2.00
1904 56 -1.35 0.15 0.65 0.36 0.54 0.22 2.00
1903 57 -1.38 0.15 0.65 0.36 0.53 0.22 2.00
1902 58 -1.41 0.15 0.64 0.35 0.53 0.22 2.00
1901 59 -1.43 0.16 0.63 0.34 0.52 0.21 2.00
1900 60 -1.46 0.16 0.63 0.34 0.51 0.21 2.00
1899 61 -1.49 0.16 0.62 0.34 0.50 0.21 2.00
1898 62 -1.51 0.17 0.62 0.33 0.50 0.21 2.00
1897 63 -1.54 0.17 0.61 0.33 0.49 0.21 2.00
1896 64 -1.56 0.17 0.61 0.32 0.48 0.21 2.00
1895 65 -1.59 0.17 0.60 0.28 0.42 0.21 2.00
1894 66 -1.61 0.18 0.59 0.27 0.41 0.20 2.00
1893 67 -1.64 0.18 0.59 0.27 0.41 0.20 2.00
1892 68 -1.66 0.18 0.58 0.27 0.40 0.20 2.00
1891 69 -1.69 0.18 0.58 0.27 0.40 0.20 2.00
1890 70 -1.71 0.19 0.57 0.26 0.40 0.20 1.99
1889 71 -1.73 0.19 0.57 0.26 0.39 0.20 1.99
1888 72 -1.76 0.19 0.57 0.26 0.39 0.20 1.98
1887 73 -1.78 0.19 0.56 0.26 0.39 0.19 1.98
1886 74 -1.80 0.20 0.56 0.25 0.38 0.19 1.97
1885 75 -1.82 0.20 0.55 0.25 0.38 0.19 1.97
1884 76 -1.84 0.20 0.55 0.25 0.38 0.19 1.97
1883 77 -1.86 0.20 0.54 0.25 0.37 0.19 1.96
1882 78 -1.88 0.20 0.54 0.25 0.37 0.19 1.96
1881 79 -1.90 0.20 0.53 0.24 0.37 0.19 1.95
1880 80 -1.91 0.21 0.53 0.24 0.36 0.19 1.95
1879 81 -1.93 0.21 0.53 0.24 0.36 0.19 1.95
1878 82 -1.95 0.21 0.52 0.24 0.36 0.18 1.94
1877 83 -1.96 0.21 0.52 0.24 0.36 0.18 1.94
1876 84 -1.98 0.21 0.52 0.24 0.35 0.18 1.94
1875 85 -1.99 0.21 0.51 0.23 0.35 0.18 1.93
1874 86 -2.01 0.22 0.51 0.23 0.35 0.18 1.93
1873 87 -2.02 0.22 0.51 0.23 0.35 0.18 1.92
1872 88 -2.03 0.22 0.50 0.23 0.34 0.18 1.92
1871 89 -2.05 0.22 0.50 0.23 0.34 0.18 1.92
1870 90 -2.06 0.22 0.50 0.23 0.34 0.18 1.91
1869 91 -2.07 0.22 0.50 0.22 0.34 0.18 1.91
1868 92 -2.08 0.22 0.49 0.22 0.34 0.18 1.91
1867 93 -2.09 0.22 0.49 0.22 0.33 0.18 1.90
1866 94 -2.09 0.22 0.49 0.22 0.33 0.17 1.90
1865 95 -2.10 0.22 0.49 0.22 0.33 0.17 1.89
1864 96 -2.11 0.22 0.48 0.18 0.27 0.17 1.54
1863 97 -2.12 0.22 0.48 0.18 0.27 0.17 1.54
1862 98 -2.12 0.22 0.48 0.18 0.26 0.17 1.53
1861 99 -2.13 0.22 0.48 0.18 0.26 0.17 1.53
1860 100 -2.13 0.22 0.48 0.17 0.26 0.17 1.53
1859 101 -2.13 0.22 0.48 0.17 0.26 0.17 1.53
1858 102 -2.14 0.22 0.47 0.17 0.26 0.17 1.52
1857 103 -2.14 0.22 0.47 0.17 0.26 0.17 1.52
1856 104 -2.14 0.22 0.47 0.17 0.26 0.17 1.52
1855 105 -2.14 0.22 0.47 0.17 0.26 0.17 1.51
1854 106 -2.14 0.22 0.47 0.17 0.26 0.17 1.51
1853 107 -2.14 0.22 0.47 0.17 0.26 0.17 1.51
1852 108 -2.13 0.22 0.47 0.17 0.25 0.17 1.50
1851 109 -2.13 0.22 0.47 0.17 0.25 0.17 1.50
1850 110 -2.13 0.22 0.47 0.17 0.25 0.17 1.49
1849 111 -2.12 0.22 0.47 0.17 0.25 0.17 1.49
1848 112 -2.12 0.22 0.47 0.17 0.25 0.17 1.48
1847 113 -2.11 0.22 0.47 0.17 0.25 0.17 1.48
1846 114 -2.11 0.22 0.47 0.17 0.25 0.17 1.47
1845 115 -2.10 0.21 0.47 0.17 0.25 0.17 1.47
1844 116 -2.09 0.21 0.47 0.16 0.25 0.17 1.46
1843 117 -2.08 0.21 0.47 0.16 0.25 0.17 1.45
1842 118 -2.07 0.21 0.47 0.16 0.25 0.17 1.45
1841 119 -2.06 0.21 0.47 0.16 0.25 0.17 1.44
1840 120 -2.05 0.21 0.47 0.16 0.24 0.17 1.43
1839 121 -2.04 0.21 0.47 0.16 0.24 0.17 1.43
1838 122 -2.03 0.20 0.47 0.16 0.24 0.17 1.42
1837 123 -2.01 0.20 0.47 0.16 0.24 0.17 1.41
1836 124 -2.00 0.20 0.47 0.14 0.22 0.17 1.25
1835 125 -1.99 0.20 0.47 0.14 0.22 0.17 1.24
1834 126 -1.97 0.20 0.47 0.14 0.21 0.17 1.24
1833 127 -1.95 0.19 0.47 0.14 0.21 0.17 1.23
1832 128 -1.94 0.19 0.48 0.14 0.21 0.18 1.22
1831 129 -1.92 0.19 0.48 0.14 0.21 0.18 1.21
1830 130 -1.90 0.19 0.48 0.14 0.21 0.18 1.21
1829 131 -1.89 0.19 0.48 0.14 0.21 0.18 1.20
1828 132 -1.87 0.18 0.48 0.14 0.21 0.18 1.19
1827 133 -1.85 0.18 0.48 0.14 0.21 0.18 1.18
1826 134 -1.83 0.18 0.49 0.14 0.21 0.18 1.17
1825 135 -1.81 0.18 0.49 0.14 0.21 0.18 1.17
1824 136 -1.79 0.17 0.49 0.34 0.51 0.18 2.00
1823 137 -1.76 0.17 0.49 0.34 0.51 0.18 2.00
1822 138 -1.74 0.17 0.50 0.33 0.50 0.18 2.00
1821 139 -1.72 0.17 0.50 0.33 0.50 0.18 2.00
1820 140 -1.70 0.16 0.50 0.33 0.50 0.19 2.00
1819 141 -1.67 0.16 0.51 0.33 0.49 0.19 2.00
1818 142 -1.65 0.16 0.51 0.32 0.49 0.19 2.00
1817 143 -1.63 0.15 0.51 0.32 0.48 0.19 2.00
1816 144 -1.60 0.15 0.51 0.32 0.48 0.19 2.00
1815 145 -1.58 0.15 0.52 0.32 0.48 0.19 2.00
1814 146 -1.55 0.15 0.52 0.31 0.47 0.19 2.00
1813 147 -1.52 0.14 0.53 0.31 0.47 0.19 2.00
1812 148 -1.50 0.14 0.53 0.31 0.47 0.20 2.00
1811 149 -1.47 0.14 0.53 0.31 0.46 0.20 2.00
1810 150 -1.44 0.13 0.54 0.31 0.46 0.20 2.00

Table I5. Results from calculations to find factor of safety against liquefaction (Kayen et al., 2013).

Elev z rd CSR CRR7.5 CRR FSLiq Comments:
For Event #1 – amax=0.3, Mw=6.2, DFW=Duration weighting factor=1.29, PL=probability of liquefaction triggering=0.15, Elev=Elevation above sea level [t], z=Depth [ft], rd=Shear stress reduction coefficient, CSR=Cyclic stress ratio for the magnitude and effective stress of interest, CRR7.5 = standardized cyclic resistance corresponding to a moment magnitude earthquake of 7.5, with 15 uniform shear stress cycles, and one atmosphere of pressure, CRR=Cyclic resistance ratio for the magnitude and effective stress of interest, FSLiq=Factor of safety against liquefaction.

See definition of variables on previous page.

See definition of variables on previous page.
1960 0
1959 1 1.00 0.19 1.00 1.29 2.00
1958 2 1.00 0.19 1.00 1.29 2.00
1957 3 1.00 0.19 1.00 1.29 2.00
1956 4 1.00 0.19 1.00 1.29 2.00
1955 5 1.00 0.19 1.00 1.29 2.00
1954 6 1.00 0.19 1.00 1.29 2.00
1953 7 1.00 0.19 1.00 1.29 2.00
1952 8 1.00 0.19 1.00 1.29 2.00
1951 9 1.00 0.19 1.00 1.29 2.00
1950 10 1.00 0.20 1.00 1.29 2.00
1949 11 1.00 0.21 1.00 1.29 2.00
1948 12 0.99 0.22 1.00 1.29 2.00
1947 13 0.99 0.23 1.00 1.29 2.00
1946 14 0.99 0.24 1.00 1.29 2.00
1945 15 0.99 0.24 1.00 1.29 2.00
1944 16 0.99 0.25 1.00 1.29 2.00
1943 17 0.99 0.25 1.00 1.29 2.00
1942 18 0.99 0.26 1.00 1.29 2.00
1941 19 0.99 0.26 1.00 1.29 2.00
1940 20 0.99 0.26 1.00 1.29 2.00
1939 21 0.98 0.27 0.49 0.63 2.00
1938 22 0.98 0.27 0.47 0.61 2.00
1937 23 0.98 0.28 0.45 0.58 2.00
1936 24 0.98 0.28 0.44 0.56 2.00
1935 25 0.98 0.28 0.42 0.54 1.94
1934 26 0.97 0.28 0.41 0.53 1.86
1933 27 0.97 0.29 0.39 0.51 1.78
1932 28 0.97 0.29 0.38 0.49 1.72
1931 29 0.96 0.29 0.37 0.48 1.66
1930 30 0.96 0.29 1.00 1.29 2.00
1929 31 0.96 0.29 1.00 1.29 2.00
1928 32 0.95 0.29 1.00 1.29 2.00
1927 33 0.95 0.29 1.00 1.29 2.00
1926 34 0.94 0.29 1.00 1.29 2.00
1925 35 0.94 0.29 1.00 1.29 2.00
1924 36 0.93 0.29 0.38 0.49 1.67
1923 37 0.93 0.29 0.37 0.47 1.63
1922 38 0.92 0.29 0.36 0.46 1.59
1921 39 0.91 0.29 0.35 0.45 1.55
1920 40 0.91 0.29 0.34 0.44 1.52
1919 41 0.90 0.29 0.33 0.43 1.49
1918 42 0.89 0.29 0.33 0.42 1.47
1917 43 0.89 0.29 0.32 0.41 1.44
1916 44 0.88 0.29 0.31 0.40 1.42
1915 45 0.87 0.28 0.31 0.40 1.40
1914 46 0.86 0.28 0.30 0.39 1.38
1913 47 0.86 0.28 0.30 0.38 1.36
1912 48 0.85 0.28 0.29 0.38 1.34
1911 49 0.84 0.28 0.29 0.37 1.33
1910 50 0.84 0.28 1.00 1.29 2.00
1909 51 0.83 0.27 1.00 1.29 2.00
1908 52 0.82 0.27 1.00 1.29 2.00
1907 53 0.81 0.27 1.00 1.29 2.00
1906 54 0.81 0.27 1.00 1.29 2.00
1905 55 0.80 0.27 1.00 1.29 2.00
1904 56 0.80 0.27 1.00 1.29 2.00
1903 57 0.79 0.27 1.00 1.29 2.00
1902 58 0.78 0.26 1.00 1.29 2.00
1901 59 0.78 0.26 1.00 1.29 2.00
1900 60 0.77 0.26 1.00 1.29 2.00
1899 61 0.77 0.26 1.00 1.29 2.00
1898 62 0.76 0.26 1.00 1.29 2.00
1897 63 0.76 0.26 1.00 1.29 2.00
1896 64 0.76 0.26 1.00 1.29 2.00
1895 65 0.75 0.26 0.75 0.96 2.00
1894 66 0.75 0.26 0.73 0.94 2.00
1893 67 0.75 0.26 0.71 0.92 2.00
1892 68 0.74 0.26 0.70 0.90 2.00
1891 69 0.74 0.26 0.68 0.89 2.00
1890 70 0.74 0.26 0.67 0.87 2.00
1889 71 0.74 0.26 0.66 0.85 2.00
1888 72 0.74 0.26 0.64 0.83 2.00
1887 73 0.73 0.26 0.63 0.82 2.00
1886 74 0.73 0.26 0.62 0.80 2.00
1885 75 0.73 0.25 0.61 0.79 2.00
1884 76 0.73 0.25 0.60 0.77 2.00
1883 77 0.73 0.25 0.59 0.76 2.00
1882 78 0.73 0.25 0.58 0.75 2.00
1881 79 0.73 0.25 0.57 0.74 2.00
1880 80 0.73 0.25 0.56 0.72 2.00
1879 81 0.72 0.25 0.55 0.71 2.00
1878 82 0.72 0.26 0.54 0.70 2.00
1877 83 0.72 0.26 0.53 0.69 2.00
1876 84 0.72 0.26 0.52 0.68 2.00
1875 85 0.72 0.26 0.52 0.67 2.00
1874 86 0.72 0.26 0.51 0.66 2.00
1873 87 0.72 0.26 0.50 0.65 2.00
1872 88 0.72 0.26 0.49 0.64 2.00
1871 89 0.72 0.26 0.49 0.63 2.00
1870 90 0.72 0.26 0.48 0.62 2.00
1869 91 0.72 0.26 0.47 0.61 2.00
1868 92 0.72 0.26 0.47 0.61 2.00
1867 93 0.72 0.26 0.46 0.60 2.00
1866 94 0.72 0.26 0.46 0.59 2.00
1865 95 0.72 0.26 0.45 0.58 2.00
1864 96 0.72 0.26 0.28 0.37 1.43
1863 97 0.72 0.26 0.28 0.36 1.41
1862 98 0.72 0.26 0.28 0.36 1.40
1861 99 0.72 0.26 0.28 0.36 1.38
1860 100 0.72 0.26 0.27 0.35 1.37
1859 101 0.72 0.26 0.27 0.35 1.36
1858 102 0.72 0.26 0.27 0.35 1.34
1857 103 0.72 0.26 0.27 0.34 1.33
1856 104 0.72 0.26 0.26 0.34 1.32
1855 105 0.72 0.26 0.26 0.34 1.31
1854 106 0.72 0.26 0.26 0.34 1.29
1853 107 0.72 0.26 0.26 0.33 1.28
1852 108 0.72 0.26 0.26 0.33 1.27
1851 109 0.72 0.26 0.25 0.33 1.26
1850 110 0.72 0.26 0.25 0.33 1.25
1849 111 0.72 0.26 0.25 0.32 1.24
1848 112 0.72 0.26 0.25 0.32 1.23
1847 113 0.72 0.26 0.25 0.32 1.22
1846 114 0.72 0.26 0.24 0.32 1.21
1845 115 0.72 0.26 0.24 0.31 1.20
1844 116 0.72 0.26 0.24 0.31 1.19
1843 117 0.72 0.26 0.24 0.31 1.18
1842 118 0.72 0.26 0.24 0.31 1.17
1841 119 0.72 0.26 0.24 0.30 1.16
1840 120 0.72 0.26 0.23 0.30 1.15
1839 121 0.72 0.26 0.23 0.30 1.14
1838 122 0.72 0.26 0.23 0.30 1.14
1837 123 0.72 0.26 0.23 0.30 1.13
1836 124 0.72 0.26 0.18 0.23 0.87
1835 125 0.72 0.26 0.18 0.23 0.86
1834 126 0.72 0.26 0.17 0.23 0.86
1833 127 0.72 0.26 0.17 0.22 0.85
1832 128 0.72 0.26 0.17 0.22 0.85
1831 129 0.72 0.26 0.17 0.22 0.84
1830 130 0.72 0.26 0.17 0.22 0.84
1829 131 0.72 0.26 0.17 0.22 0.83
1828 132 0.72 0.26 0.17 0.22 0.83
1827 133 0.72 0.27 0.17 0.22 0.83
1826 134 0.72 0.27 0.17 0.22 0.82
1825 135 0.72 0.27 0.17 0.22 0.82
1824 136 0.72 0.27 1.00 1.29 2.00
1823 137 0.72 0.27 1.00 1.29 2.00
1822 138 0.72 0.27 1.00 1.29 2.00
1821 139 0.72 0.27 1.00 1.29 2.00
1820 140 0.72 0.27 1.00 1.29 2.00
1819 141 0.72 0.27 1.00 1.29 2.00
1818 142 0.72 0.27 1.00 1.29 2.00
1817 143 0.72 0.27 1.00 1.29 2.00
1816 144 0.72 0.26 1.00 1.29 2.00
1815 145 0.72 0.26 0.99 1.28 2.00
1814 146 0.72 0.26 0.98 1.26 2.00
1813 147 0.72 0.26 0.96 1.25 2.00
1812 148 0.72 0.26 0.95 1.23 2.00
1811 149 0.72 0.26 0.93 1.21 2.00
1810 150 0.72 0.26 0.92 1.19 2.00

Table I6. Results from calculations to find soil settlement (Boulanger and Idriss, 2008).

Elev z Dr γlim γmax εv δ Σδ Comments: For Event #1 – amax=0.3, Mw=6.2, z=Depth [ft], FSα =Factor of Safety, γlim =Limiting shear strain, γmax =Maximum shear strain, εv =Volumetric strain, δ = ∆s = Incremental soil settlement [in], Σδ = s1D = Cumulative soil settlement from bottom of soil profile to top of soil profile [in].

Applicable for gravel and sand (merged FS based on Rollins et al. 2022 for gravel or Kayen et al. 2013 for sand and silt).

See definition of variables on previous page.

Applicable for gravel and sand (merged FS based on Rollins et al. 2022 for gravel or Kayen et al. 2013 for sand and silt).

See definition of variables on previous page.

Applicable for gravel and sand (merged FS based on Rollins et al. 2022 for gravel or Kayen et al. 2013 for sand and silt).
1960 0
1959 1 67 0.488 15 0.000 0.00 0.00 2.85
1958 2 67 0.488 15 0.000 0.00 0.00 2.85
1957 3 67 0.488 15 0.000 0.00 0.00 2.85
1956 4 67 0.488 15 0.000 0.00 0.00 2.85
1955 5 67 0.488 15 0.000 0.00 0.00 2.85
1954 6 67 0.488 15 0.000 0.00 0.00 2.85
1953 7 67 0.488 15 0.000 0.00 0.00 2.85
1952 8 67 0.488 15 0.000 0.00 0.00 2.85
1951 9 67 0.488 15 0.000 0.00 0.00 2.85
1950 10 99 -1.196 0 0.000 0.00 0.00 2.85
1949 11 99 -1.196 0 0.000 0.00 0.00 2.85
1948 12 99 -1.196 0 0.000 0.00 0.00 2.85
1947 13 99 -1.196 0 0.000 0.00 0.00 2.85
1946 14 99 -1.196 0 0.000 0.00 0.00 2.85
1945 15 99 -1.196 0 0.000 0.00 0.00 2.85
1944 16 99 -1.196 0 0.000 0.00 0.00 2.85
1943 17 99 -1.196 0 0.000 0.00 0.00 2.85
1942 18 99 -1.196 0 0.000 0.00 0.00 2.85
1941 19 99 -1.196 0 0.000 0.00 0.00 2.85
1940 20 99 -1.196 0 0.000 0.00 0.00 2.85
1939 21 46 0.924 49 0.000 0.00 0.00 2.85
1938 22 46 0.924 49 0.000 0.00 0.00 2.85
1937 23 46 0.924 49 0.000 0.00 0.00 2.85
1936 24 46 0.924 49 0.000 0.00 0.00 2.85
1935 25 46 0.924 49 0.017 0.01 0.00 2.85
1934 26 46 0.924 49 0.041 0.02 0.00 2.85
1933 27 46 0.924 49 0.067 0.03 0.00 2.85
1932 28 46 0.924 49 0.094 0.04 0.01 2.84
1931 29 46 0.924 49 0.124 0.06 0.01 2.84
1930 30 94 -0.852 1 0.000 0.00 0.00 2.83
1929 31 94 -0.852 1 0.000 0.00 0.00 2.83
1928 32 94 -0.852 1 0.000 0.00 0.00 2.83
1927 33 94 -0.852 1 0.000 0.00 0.00 2.83
1926 34 94 -0.852 1 0.000 0.00 0.00 2.83
1925 35 94 -0.852 1 0.000 0.00 0.00 2.83
1924 36 42 0.948 58 0.085 0.04 0.01 2.83
1923 37 42 0.948 58 0.101 0.05 0.01 2.82
1922 38 42 0.948 58 0.118 0.06 0.01 2.82
1921 39 42 0.948 58 0.135 0.07 0.01 2.81
1920 40 42 0.948 58 0.153 0.08 0.01 2.80
1919 41 42 0.948 58 0.171 0.09 0.01 2.79
1918 42 42 0.948 58 0.189 0.10 0.01 2.78
1917 43 42 0.948 58 0.208 0.11 0.01 2.77
1916 44 42 0.948 58 0.227 0.12 0.01 2.76
1915 45 42 0.948 58 0.246 0.13 0.02 2.74
1914 46 42 0.948 58 0.265 0.14 0.02 2.73
1913 47 42 0.948 58 0.284 0.15 0.02 2.71
1912 48 42 0.948 58 0.304 0.16 0.02 2.69
1911 49 42 0.948 58 0.323 0.17 0.02 2.67
1910 50 84 -0.254 3 0.000 0.00 0.00 2.65
1909 51 84 -0.254 3 0.000 0.00 0.00 2.65
1908 52 84 -0.254 3 0.000 0.00 0.00 2.65
1907 53 84 -0.254 3 0.000 0.00 0.00 2.65
1906 54 84 -0.254 3 0.000 0.00 0.00 2.65
1905 55 84 -0.254 3 0.000 0.00 0.00 2.65
1904 56 84 -0.254 3 0.000 0.00 0.00 2.65
1903 57 84 -0.254 3 0.000 0.00 0.00 2.65
1902 58 84 -0.254 3 0.000 0.00 0.00 2.65
1901 59 84 -0.254 3 0.000 0.00 0.00 2.65
1900 60 84 -0.254 3 0.000 0.00 0.00 2.65
1899 61 84 -0.254 3 0.000 0.00 0.00 2.65
1898 62 84 -0.254 3 0.000 0.00 0.00 2.65
1897 63 84 -0.254 3 0.000 0.00 0.00 2.65
1896 64 84 -0.254 3 0.000 0.00 0.00 2.65
1895 65 68 0.454 14 0.000 0.00 0.00 2.65
1894 66 68 0.454 14 0.000 0.00 0.00 2.65
1893 67 68 0.454 14 0.000 0.00 0.00 2.65
1892 68 68 0.454 14 0.000 0.00 0.00 2.65
1891 69 68 0.454 14 0.002 0.00 0.00 2.65
1890 70 68 0.454 14 0.008 0.00 0.00 2.65
1889 71 68 0.454 14 0.015 0.00 0.00 2.65
1888 72 68 0.454 14 0.021 0.01 0.00 2.65
1887 73 68 0.454 14 0.027 0.01 0.00 2.65
1886 74 68 0.454 14 0.033 0.01 0.00 2.65
1885 75 68 0.454 14 0.038 0.01 0.00 2.65
1884 76 68 0.454 14 0.044 0.01 0.00 2.65
1883 77 68 0.454 14 0.049 0.01 0.00 2.65
1882 78 68 0.454 14 0.054 0.01 0.00 2.64
1881 79 68 0.454 14 0.059 0.02 0.00 2.64
1880 80 68 0.454 14 0.064 0.02 0.00 2.64
1879 81 68 0.454 14 0.068 0.02 0.00 2.64
1878 82 68 0.454 14 0.073 0.02 0.00 2.64
1877 83 68 0.454 14 0.078 0.02 0.00 2.63
1876 84 68 0.454 14 0.083 0.02 0.00 2.63
1875 85 68 0.454 14 0.088 0.02 0.00 2.63
1874 86 68 0.454 14 0.093 0.03 0.00 2.63
1873 87 68 0.454 14 0.098 0.03 0.00 2.62
1872 88 68 0.454 14 0.103 0.03 0.00 2.62
1871 89 68 0.454 14 0.108 0.03 0.00 2.62
1870 90 68 0.454 14 0.113 0.03 0.00 2.61
1869 91 68 0.454 14 0.119 0.03 0.00 2.61
1868 92 68 0.454 14 0.125 0.03 0.00 2.61
1867 93 68 0.454 14 0.131 0.04 0.00 2.60
1866 94 68 0.454 14 0.137 0.04 0.00 2.60
1865 95 68 0.454 14 0.143 0.04 0.00 2.59
1864 96 48 0.906 44 0.242 0.11 0.01 2.59
1863 97 48 0.906 44 0.244 0.11 0.01 2.57
1862 98 48 0.906 44 0.245 0.11 0.01 2.56
1861 99 48 0.906 44 0.247 0.11 0.01 2.55
1860 100 48 0.906 44 0.250 0.11 0.01 2.53
1859 101 48 0.906 44 0.252 0.11 0.01 2.52
1858 102 48 0.906 44 0.255 0.12 0.01 2.51
1857 103 48 0.906 44 0.258 0.12 0.01 2.49
1856 104 48 0.906 44 0.261 0.12 0.01 2.48
1855 105 48 0.906 44 0.264 0.12 0.01 2.47
1854 106 48 0.906 44 0.268 0.12 0.01 2.45
1853 107 48 0.906 44 0.272 0.12 0.01 2.44
1852 108 48 0.906 44 0.276 0.12 0.01 2.42
1851 109 48 0.906 44 0.280 0.13 0.02 2.41
1850 110 48 0.906 44 0.285 0.13 0.02 2.39
1849 111 48 0.906 44 0.291 0.13 0.02 2.38
1848 112 48 0.906 44 0.296 0.13 0.02 2.36
1847 113 48 0.906 44 0.302 0.14 0.02 2.34
1846 114 48 0.906 44 0.308 0.14 0.02 2.33
1845 115 48 0.906 44 0.315 0.14 0.02 2.31
1844 116 48 0.906 44 0.322 0.15 0.02 2.29
1843 117 48 0.906 44 0.330 0.15 0.02 2.28
1842 118 48 0.906 44 0.338 0.15 0.02 2.26
1841 119 48 0.906 44 0.346 0.16 0.02 2.24
1840 120 48 0.906 44 0.356 0.16 0.02 2.22
1839 121 48 0.906 44 0.365 0.17 0.02 2.20
1838 122 48 0.906 44 0.376 0.170 0.020 2.18
1837 123 48 0.906 44 0.386 0.17 0.02 2.16
1836 124 37 -0.952 72 2.325 1.38 0.17 2.14
1835 125 37 -0.952 72 2.354 1.40 0.17 1.98
1834 126 37 -0.952 72 2.383 1.42 0.17 1.81
1833 127 37 -0.952 72 2.413 1.44 0.17 1.64
1832 128 37 -0.952 72 2.445 1.45 0.17 1.47
1831 129 37 -0.952 72 2.477 1.47 0.18 1.29
1830 130 37 -0.952 72 2.511 1.49 0.18 1.11
1829 131 37 -0.952 72 2.545 1.51 0.18 0.93
1828 132 37 -0.952 72 2.581 1.54 0.18 0.75
1827 133 37 -0.952 72 2.618 1.56 0.19 0.57
1826 134 37 -0.952 72 2.655 1.58 0.19 0.38
1825 135 37 -0.952 72 2.694 1.60 0.19 0.19
1824 136 86 -0.364 3 0.000 0.00 0.00 0.00
1823 137 86 -0.364 3 0.000 0.00 0.00 0.00
1822 138 86 -0.364 3 0.000 0.00 0.00 0.00
1821 139 86 -0.364 3 0.000 0.00 0.00 0.00
1820 140 86 -0.364 3 0.000 0.00 0.00 0.00
1819 141 86 -0.364 3 0.000 0.00 0.00 0.00
1818 142 86 -0.364 3 0.000 0.00 0.00 0.00
1817 143 86 -0.364 3 0.000 0.00 0.00 0.00
1816 144 86 -0.364 3 0.000 0.00 0.00 0.00
1815 145 86 -0.364 3 0.000 0.00 0.00 0.00
1814 146 86 -0.364 3 0.000 0.00 0.00 0.00
1813 147 86 -0.364 3 0.000 0.00 0.00 0.00
1812 148 86 -0.364 3 0.000 0.00 0.00 0.00
1811 149 86 -0.364 3 0.000 0.00 0.00 0.00
1810 150 86 -0.364 3 0.000 0.00 0.00 0.00

Table I8. PileAXL output for the Tok River Bridge Abutment 1 piles (converted to imperial units).

Depth ULS fs ULS Qs ULS Qb ∆Q Q R min(Q,R) δpile Comments:
ULS fs = Ultimate unit side shear (PileAXL output), ULS Qs = Ultimate total side resistance (PileAXL output), ULS Qb = Ultimate bearing resistance (PileAXL output), ∆Q =incremental side resistance (Excel calculation), Q = cumulative load in the pile (Excel calculation), R= resistance from soil surrounding pile; resistance values are calculated at the top of each sublayer (Excel calculation), min(Q,R) = minimum of Q and R for each depth (Excel calculation).

See definition of variables on previous page.
[ft] [tsf] [tons] [tons] [tons] [tons] [tons] [tons] [in]
0.00 0.0000 0.0000 0.0000 0.0000 149.5000 411.6266 149.5000 1.540
2.00 0.0357 0.1684 10.6007 0.1684 149.6684 411.6266 149.6684 1.531
4.00 0.0715 0.6736 21.2014 0.5052 150.1736 411.4583 150.1736 1.523
6.00 0.0835 1.4040 31.8022 0.7304 150.9040 410.9531 150.9040 1.514
8.00 0.0835 2.1912 42.4029 0.7872 151.6912 410.2227 151.6912 1.505
10.00 0.0835 2.9784 53.0036 0.7872 152.4784 409.4355 152.4784 1.496
12.00 0.1253 3.9624 58.8319 0.9840 153.4624 408.6482 153.4624 1.487
14.00 0.1253 5.1433 64.6412 1.1808 154.6433 407.6642 154.6433 1.478
16.00 0.1253 6.3241 70.4505 1.1808 155.8241 406.4834 155.8241 1.469
18.00 0.1253 7.5049 76.2598 1.1808 157.0049 405.3025 157.0049 1.460
20.00 0.1253 8.6858 82.0691 1.1808 158.1858 404.1217 158.1858 1.451
22.00 0.1671 10.0634 69.8799 1.3776 159.5634 402.9409 159.5634 1.441
24.00 0.1671 11.6379 73.6816 1.5744 161.1379 401.5632 161.1379 1.432
26.00 0.1671 13.2123 77.4832 1.5744 162.7123 399.9888 162.7123 1.423
28.00 0.1671 14.7867 81.2849 1.5745 164.2867 398.4143 164.2867 1.413
30.00 0.1880 16.4596 106.3620 1.6728 165.9596 396.8399 165.9596 1.403
32.00 0.1880 18.2308 111.9858 1.7713 167.7308 395.1671 167.7308 1.394
34.00 0.1880 20.0021 117.6095 1.7713 169.5021 393.3958 169.5021 1.384
36.00 0.1880 21.7733 123.2332 1.7712 171.2733 391.6245 171.2733 1.374
37.99 0.2506 23.8398 103.0855 2.0665 173.3398 389.8533 173.3398 1.364
39.99 0.2506 26.2015 107.5845 2.3617 175.7015 387.7868 175.7015 1.353
41.99 0.2506 28.5631 112.0835 2.3617 178.0631 385.4252 178.0631 1.343
43.99 0.2506 30.9248 116.5825 2.3617 180.4248 383.0635 180.4248 1.333
45.99 0.2506 33.2865 121.0815 2.3617 182.7865 380.7018 182.7865 1.322
47.99 0.2506 35.6481 125.5804 2.3617 185.1481 378.3402 185.1481 1.311
49.99 0.3133 38.3050 162.5888 2.6569 187.8050 375.9785 187.8050 1.300
51.99 0.3133 41.2571 168.0438 2.9521 190.7571 373.3216 190.7571 1.289
53.99 0.3133 44.2092 173.4988 2.9521 193.7092 370.3695 193.7092 1.278
55.99 0.3133 47.1613 178.9538 2.9521 196.6613 367.4174 196.6613 1.266
57.99 0.3133 50.1134 184.4088 2.9521 199.6134 364.4654 199.6134 1.255
59.99 0.3133 53.0654 189.8638 2.9521 202.5654 361.5133 202.5654 1.243
61.99 0.3133 56.0175 195.3188 2.9521 205.5175 358.5612 205.5175 1.231
63.99 0.3133 58.9696 200.7738 2.9521 208.4696 355.6091 208.4696 1.219
65.99 0.4282 62.4629 206.0956 3.4933 211.9629 352.6570 211.9629 1.207
67.99 0.4282 66.4974 211.2919 4.0345 215.9974 349.1637 215.9974 1.194
69.99 0.4282 70.5319 216.4882 4.0345 220.0319 345.1292 220.0319 1.181
71.99 0.4282 74.5665 221.4368 4.0345 224.0665 341.0947 224.0665 1.168
73.99 0.4282 78.6010 221.4368 4.0345 228.1010 337.0602 228.1010 1.155
75.99 0.4282 82.6355 221.4368 4.0345 232.1355 333.0257 232.1355 1.141
77.99 0.4282 86.6700 221.4368 4.0345 236.1700 328.9911 236.1700 1.128
79.99 0.4282 90.7045 221.4368 4.0345 240.2045 324.9566 240.2045 1.114
81.99 0.4282 94.7390 221.4368 4.0345 244.2390 320.9221 244.2390 1.099
83.99 0.4282 98.7736 221.4368 4.0345 248.2736 316.8876 248.2736 1.085
85.99 0.4282 102.8081 221.4368 4.0345 252.3081 312.8531 252.3081 1.070
87.99 0.4282 106.8426 221.4368 4.0345 256.3426 308.8186 256.3426 1.055
89.99 0.4282 110.8771 221.4368 4.0345 260.3771 304.7840 260.3771 1.040
91.99 0.4282 114.9116 221.4368 4.0345 264.4116 300.7495 264.4116 1.025
93.99 0.4282 118.9461 221.4368 4.0345 268.4461 296.7150 268.4461 1.009
95.99 0.4282 122.9807 221.4368 4.0345 272.4807 292.6805 272.4807 0.993
97.99 0.5535 127.6056 221.4368 4.6249 277.1056 288.6460 277.1056 0.977
99.99 0.5535 132.8209 221.4368 5.2153 282.3209 284.0210 282.3209 0.961
101.99 0.5535 138.0363 221.4368 5.2153 287.5363 278.8057 278.8057 0.945
103.99 0.5535 143.2516 221.4368 5.2153 292.7516 273.5903 273.5903 0.929
105.99 0.5535 148.4670 221.4368 5.2153 297.9670 268.3750 268.3750 0.913
107.99 0.5535 153.6823 221.4368 5.2153 303.1823 263.1596 263.1596 0.898
109.98 0.5535 158.8977 221.4368 5.2154 308.3977 257.9443 257.9443 0.883
111.98 0.5535 164.1130 221.4368 5.2153 313.6130 252.7289 252.7289 0.868
113.98 0.5535 169.3284 221.4368 5.2153 318.8284 247.5136 247.5136 0.854
115.98 0.5535 174.5437 221.4368 5.2153 324.0437 242.2982 242.2982 0.840
117.98 0.5535 179.7591 221.4368 5.2153 329.2591 237.0829 237.0829 0.826
119.98 0.5535 184.9744 221.4368 5.2153 334.4744 231.8675 231.8675 0.812
121.98 0.5535 190.1898 221.4368 5.2153 339.6898 226.6522 226.6522 0.799
123.98 0.5535 195.4051 221.4368 5.2153 344.9051 221.4368 221.4368 0.786

Table I11. Results from TZPILE, as obtained by using PileAXL input and additional spreadsheet post-processing calculations.

z Min(Q,R) δp Comments:
z=Depth [ft], Min(Q,R)=Load used to develop combination curve to identify the location of the neutral plane, δp =pile settlement.
1 294.1 2.887
3 294.0 2.878
5 293.8 2.870
7 293.7 2.861
9 293.6 2.852
11 293.6 2.843
13 293.8 2.834
15 294.1 2.825
17 294.7 2.816
19 295.5 2.808
21 296.5 2.799
23 297.7 2.790
25 299.2 2.781
27 300.9 2.772
29 302.9 2.763
31 305.0 2.753
33 307.2 2.744
35 309.6 2.735
37 312.2 2.726
39 315.0 2.716
41 317.9 2.707
43 320.7 2.697
45 323.3 2.687
47 325.5 2.678
49 326.5 2.668
51 326.5 2.658
53 326.4 2.648
55 327.0 2.638
57 328.4 2.628
59 330.5 2.618
61 333.2 2.608
63 336.7 2.598
65 340.7 2.588
67 345.3 2.578
69 350.4 2.567
71 355.9 2.557
73 361.8 2.546
75 368.1 2.535
77 374.6 2.524
79 381.3 2.512
81 388.3 2.501
83 395.5 2.489
85 402.8 2.477
87 410.3 2.465
89 418.0 2.452
91 425.7 2.440
93 433.6 2.427
95 441.6 2.414
97 449.6 2.400
99 457.6 2.386
101 465.4 2.373
103 473.0 2.358
105 480.4 2.344
107 487.4 2.329
109 494.0 2.315
111 500.1 2.300
113 505.4 2.285
115 509.8 2.269
117 512.9 2.254
119 514.1 2.238
121 513.2 2.223
123 509.9 2.208
125 502.5 2.192
127 491.9 2.177
129 481.2 2.163
131 470.4 2.148
133 459.4 2.134
135 448.4 2.121

Table J2. Parameters used to find soil settlement.

Layer Depth z σzo α1,L α1,L IL α1,R α1,R IR Σ(I) ∆σ σ′zf δinc Σδ
0 - 1 0.5 28.8 0.025 1.446 0.500 0.014 1.551 0.500 1.000 3479 3508 0.069 0.069
1 - 2 1.5 86.4 0.067 1.212 0.493 0.043 1.510 0.500 0.993 3455 3542 0.054 0.123
2 - 3 2.5 144 0.095 1.012 0.473 0.071 1.470 0.500 0.973 3387 3531 0.046 0.169
3 - 4 3.5 201.6 0.108 0.852 0.443 0.099 1.430 0.500 0.943 3282 3484 0.041 0.210
4 - 5 4.5 259.2 0.111 0.727 0.408 0.126 1.390 0.500 0.908 3161 3420 0.037 0.247
5 - 6 5.5 316.8 0.109 0.629 0.374 0.153 1.351 0.500 0.873 3039 3356 0.034 0.281
6 - 7 6.5 374.4 0.104 0.552 0.341 0.179 1.313 0.499 0.840 2925 3299 0.031 0.313
7 - 8 7.5 432 0.098 0.490 0.312 0.205 1.275 0.499 0.811 2822 3254 0.029 0.342
8 - 9 8.5 489.6 0.092 0.440 0.286 0.230 1.239 0.498 0.785 2731 3220 0.027 0.369
9 - 10 9.5 547.2 0.086 0.399 0.264 0.253 1.203 0.498 0.762 2650 3197 0.025 0.395
10 - 11 10.5 604.8 0.080 0.364 0.244 0.276 1.168 0.497 0.741 2579 3184 0.024 0.419
11 - 12 11.5 662.4 0.075 0.335 0.227 0.298 1.134 0.496 0.723 2516 3178 0.023 0.441
12 - 13 12.5 720 0.071 0.310 0.211 0.319 1.101 0.495 0.707 2459 3179 0.021 0.463
13 - 14 13.5 777.6 0.067 0.288 0.198 0.339 1.070 0.494 0.692 2409 3186 0.020 0.483
14 - 15 14.5 835.2 0.063 0.269 0.186 0.358 1.039 0.493 0.679 2363 3198 0.019 0.502
15 - 16 15.5 892.8 0.059 0.253 0.175 0.376 1.009 0.492 0.667 2321 3214 0.018 0.521
16 - 17 16.5 950.4 0.056 0.238 0.165 0.393 0.981 0.490 0.656 2282 3233 0.018 0.538
17 - 18 17.5 1008 0.054 0.225 0.157 0.409 0.953 0.489 0.646 2247 3255 0.017 0.555
18 - 19 18.5 1065.6 0.051 0.213 0.149 0.424 0.927 0.487 0.636 2214 3280 0.016 0.572
19 - 20 19.5 1123.2 0.049 0.202 0.142 0.438 0.902 0.485 0.627 2183 3306 0.016 0.587
20 - 21 20.5 1180.8 0.047 0.193 0.135 0.451 0.877 0.484 0.619 2154 3335 0.015 0.602
21 - 22 21.5 1238.4 0.045 0.184 0.129 0.463 0.854 0.482 0.611 2126 3365 0.014 0.617
22 - 23 22.5 1296 0.043 0.176 0.124 0.474 0.831 0.480 0.604 2100 3396 0.014 0.630
23 - 24 23.5 1353.6 0.041 0.169 0.119 0.484 0.809 0.477 0.596 2075 3429 0.013 0.644
24 - 25 24.5 1411.2 0.039 0.162 0.114 0.494 0.788 0.475 0.589 2051 3463 0.013 0.657
25 - 26 25.5 1468.8 0.038 0.156 0.110 0.503 0.768 0.473 0.583 2028 3497 0.013 0.669
26 - 27 26.5 1526.4 0.037 0.150 0.106 0.511 0.749 0.470 0.576 2006 3532 0.012 0.681
27 - 28 27.5 1594 0.035 0.144 0.102 0.519 0.731 0.468 0.570 1985 3579 0.012 0.693
28 - 29 28.5 1661.6 0.034 0.139 0.099 0.526 0.713 0.465 0.564 1964 3625 0.011 0.704
29 - 30 29.5 1729.2 0.033 0.135 0.096 0.532 0.696 0.463 0.558 1943 3673 0.011 0.715
30 - 31 30.5 1796.8 0.032 0.130 0.093 0.538 0.680 0.460 0.553 1924 3720 0.011 0.726
31 - 32 31.5 1864.4 0.031 0.126 0.090 0.543 0.664 0.458 0.547 1904 3769 0.010 0.736
32 - 33 32.5 1932 0.030 0.122 0.087 0.547 0.649 0.455 0.542 1885 3817 0.010 0.746
33 - 34 33.5 1999.6 0.029 0.119 0.084 0.551 0.634 0.452 0.537 1867 3867 0.010 0.755
34 - 35 34.5 2067.2 0.028 0.115 0.082 0.555 0.620 0.449 0.531 1849 3916 0.009 0.764
35 - 36 35.5 2134.8 0.028 0.112 0.080 0.558 0.607 0.446 0.526 1831 3966 0.009 0.773
36 - 37 36.5 2202.4 0.027 0.109 0.078 0.561 0.594 0.444 0.521 1814 4016 0.009 0.782
37 - 38 37.5 2270 0.026 0.106 0.076 0.563 0.582 0.441 0.516 1797 4067 0.008 0.790
38 - 39 38.5 2337.6 0.026 0.104 0.074 0.565 0.569 0.438 0.511 1780 4117 0.008 0.799
39 - 40 39.5 2405.2 0.025 0.101 0.072 0.567 0.558 0.435 0.507 1763 4169 0.008 0.807
40 - 41 40.5 2472.8 0.024 0.098 0.070 0.568 0.547 0.432 0.502 1747 4220 0.008 0.814
41 - 42 41.5 2540.4 0.024 0.096 0.068 0.569 0.536 0.429 0.497 1731 4271 0.007 0.822
42 - 43 42.5 2608 0.023 0.094 0.067 0.570 0.526 0.426 0.493 1715 4323 0.007 0.829
43 - 44 43.5 2675.6 0.023 0.092 0.065 0.571 0.516 0.423 0.488 1700 4375 0.007 0.836
44 - 45 44.5 2743.2 0.022 0.090 0.064 0.571 0.506 0.420 0.484 1684 4427 0.007 0.843
45 - 46 45.5 2810.8 0.022 0.088 0.063 0.571 0.496 0.417 0.480 1669 4480 0.007 0.850
46 - 47 46.5 2878.4 0.021 0.086 0.061 0.571 0.487 0.414 0.475 1654 4533 0.007 0.856
47 - 48 47.5 2946 0.021 0.084 0.060 0.570 0.479 0.411 0.471 1639 4585 0.006 0.863
48 - 49 48.5 3018.6 0.020 0.082 0.059 0.570 0.470 0.408 0.467 1625 4643 0.006 0.869
49 - 50 49.5 3091.2 0.020 0.081 0.058 0.569 0.462 0.405 0.463 1610 4702 0.006 0.875
50 - 51 50.5 3163.8 0.020 0.079 0.056 0.568 0.454 0.402 0.459 1596 4760 0.006 0.881
51 - 52 51.5 3236.4 0.019 0.078 0.055 0.567 0.446 0.399 0.455 1582 4819 0.006 0.887
z=layer midpoint depth [ft], σ′z=vertical effective stress [psf], α1 and α2=angles [radians] from Figure J3, I=influence factor, ∆σ=change in stress [psf], δinc=settlement of sublayer [ft], Σδ (from bottom to top) = soil settlement profile (Figure J2). L=left, R=right, o=initial, f=final.

Table J3. PileAXL output for the MN 74551 bridge abutment piles.

Depth ULS fs ULS Qs ULS Qb ∆Q Q R min(Q,R) δpile Comments:
ULS fs = Ultimate unit side resistance (PileAXL output), ULS Qs = Ultimate total side resistance (PileAXL output), ULS Qb = Ultimate bearing resistance (PileAXL output), ∆Q = discretized side resistance (Excel calculation), Q = cumulative load in the pile (Excel calculation), R= resistance from soil surrounding pile; resistance values are calculated at the top of each sublayer (Excel calculation), min(Q,R) = minimum of Q and R for each depth (Excel calculation), δpile=Settlement of pile obtained by determining the pile head movement from the PileAXL load-settlement curve (Figure J13) and the elastic compression.
[ft] [tsf] [tons] [tons] [tons] [tons] [tons] [tons] [in]
0.000 0.700 0.000 0.361 0.328 0.000 89.099 0.000 0.441
1.000 1.067 0.328 0.361 0.434 0.328 88.770 0.328 0.441
2.000 1.269 0.762 0.361 0.497 0.762 88.336 0.762 0.441
3.000 1.404 1.259 0.361 0.541 1.259 87.840 1.259 0.441
4.000 1.509 1.801 0.361 0.577 1.801 87.298 1.801 0.441
5.000 1.596 2.378 0.361 0.607 2.378 86.721 2.378 0.441
6.000 1.670 2.984 0.361 0.633 2.984 86.114 2.984 0.441
7.000 1.736 3.617 0.361 0.656 3.617 85.481 3.617 0.441
8.000 1.795 4.273 0.361 0.677 4.273 84.825 4.273 0.441
9.000 1.848 4.950 0.361 0.696 4.950 84.149 4.950 0.441
10.000 1.898 5.646 0.361 0.714 5.646 83.452 5.646 0.441
11.000 1.943 6.360 0.361 0.730 6.360 82.739 6.360 0.441
12.000 1.986 7.090 0.361 0.746 7.090 82.009 7.090 0.441
13.000 2.026 7.836 0.361 0.764 7.836 81.263 7.836 0.441
14.000 2.083 8.600 0.361 0.788 8.600 80.499 8.600 0.441
15.000 2.157 9.387 0.361 0.815 9.387 79.711 9.387 0.441
16.000 2.227 10.202 0.361 0.840 10.202 78.897 10.202 0.441
17.000 2.296 11.042 0.361 0.866 11.042 78.056 11.042 0.441
18.000 2.362 11.908 0.361 0.890 11.908 77.191 11.908 0.441
19.000 2.427 12.798 0.361 0.914 12.798 76.301 12.798 0.441
20.001 2.490 13.712 0.361 0.937 13.712 75.387 13.712 0.441
21.001 2.552 14.649 0.361 0.959 14.649 74.450 14.649 0.441
22.001 2.612 15.608 0.361 0.982 15.608 73.491 15.608 0.441
23.001 2.671 16.590 0.361 1.003 16.590 72.509 16.590 0.440
24.001 2.728 17.593 0.361 1.024 17.593 71.506 17.593 0.440
25.001 2.784 18.617 0.361 1.045 18.617 70.482 18.617 0.440
26.001 2.839 19.662 0.361 1.074 19.662 69.437 19.662 0.440
27.001 2.941 20.736 0.371 1.120 20.736 68.363 20.736 0.440
28.001 3.085 21.855 0.392 1.173 21.855 67.244 21.855 0.440
29.001 3.229 23.028 0.412 1.227 23.028 66.070 23.028 0.440
30.001 3.373 24.255 0.432 1.280 24.255 64.844 24.255 0.440
31.001 3.516 25.535 0.452 1.333 25.535 63.564 25.535 0.439
32.001 3.660 26.869 0.473 1.387 26.869 62.230 26.869 0.439
33.001 3.804 28.255 0.493 1.440 28.255 60.843 28.255 0.439
34.001 3.947 29.696 0.513 1.494 29.696 59.403 29.696 0.439
35.001 4.091 31.189 0.533 1.547 31.189 57.910 31.189 0.439
36.001 4.235 32.736 0.554 1.600 32.736 56.362 32.736 0.439
37.001 4.378 34.337 0.574 1.654 34.337 54.762 34.337 0.438
38.001 4.522 35.990 0.594 1.707 35.990 53.108 35.990 0.438
39.001 4.665 37.698 0.614 1.760 37.698 51.401 37.698 0.438
40.001 4.809 39.458 0.635 1.814 39.458 49.641 39.458 0.438
41.001 4.952 41.272 0.655 1.867 41.272 47.827 41.272 0.438
42.001 5.096 43.139 0.675 1.920 43.139 45.960 43.139 0.437
43.001 5.239 45.059 0.695 1.974 45.059 44.040 44.040 0.437
44.001 5.383 47.033 0.716 2.027 47.033 42.066 42.066 0.437
45.001 5.526 49.060 0.736 2.080 49.060 40.039 40.039 0.437
46.001 5.669 51.140 0.756 2.134 51.140 37.959 37.959 0.436
47.001 5.813 53.273 0.776 2.338 53.273 35.825 35.825 0.436
48.001 6.772 55.612 1.029 2.837 55.612 33.487 33.487 0.436
49.001 8.494 58.448 1.525 3.539 58.448 30.651 30.651 0.436
50.001 10.555 61.988 2.021 4.287 61.988 27.111 27.111 0.436
51.001 12.516 66.274 2.518 10.174 66.274 22.824 22.824 0.435
52.001 42.239 76.448 12.650 76.448 12.650 12.650 0.435
Page 269
Suggested Citation:"Appendix J: Design Example 8 - Embankment Over Clay Over Rock Using PileAXL." National Academies of Sciences, Engineering, and Medicine. 2024. Pile Design for Downdrag: Examples and Supporting Materials. Washington, DC: The National Academies Press. doi: 10.17226/27864.
×
Page 269
Page 270
Suggested Citation:"Appendix J: Design Example 8 - Embankment Over Clay Over Rock Using PileAXL." National Academies of Sciences, Engineering, and Medicine. 2024. Pile Design for Downdrag: Examples and Supporting Materials. Washington, DC: The National Academies Press. doi: 10.17226/27864.
×
Page 270
Page 271
Suggested Citation:"Appendix J: Design Example 8 - Embankment Over Clay Over Rock Using PileAXL." National Academies of Sciences, Engineering, and Medicine. 2024. Pile Design for Downdrag: Examples and Supporting Materials. Washington, DC: The National Academies Press. doi: 10.17226/27864.
×
Page 271
Page 272
Suggested Citation:"Appendix J: Design Example 8 - Embankment Over Clay Over Rock Using PileAXL." National Academies of Sciences, Engineering, and Medicine. 2024. Pile Design for Downdrag: Examples and Supporting Materials. Washington, DC: The National Academies Press. doi: 10.17226/27864.
×
Page 272
Page 273
Suggested Citation:"Appendix J: Design Example 8 - Embankment Over Clay Over Rock Using PileAXL." National Academies of Sciences, Engineering, and Medicine. 2024. Pile Design for Downdrag: Examples and Supporting Materials. Washington, DC: The National Academies Press. doi: 10.17226/27864.
×
Page 273
Page 274
Suggested Citation:"Appendix J: Design Example 8 - Embankment Over Clay Over Rock Using PileAXL." National Academies of Sciences, Engineering, and Medicine. 2024. Pile Design for Downdrag: Examples and Supporting Materials. Washington, DC: The National Academies Press. doi: 10.17226/27864.
×
Page 274
Page 275
Suggested Citation:"Appendix J: Design Example 8 - Embankment Over Clay Over Rock Using PileAXL." National Academies of Sciences, Engineering, and Medicine. 2024. Pile Design for Downdrag: Examples and Supporting Materials. Washington, DC: The National Academies Press. doi: 10.17226/27864.
×
Page 275
Page 276
Suggested Citation:"Appendix J: Design Example 8 - Embankment Over Clay Over Rock Using PileAXL." National Academies of Sciences, Engineering, and Medicine. 2024. Pile Design for Downdrag: Examples and Supporting Materials. Washington, DC: The National Academies Press. doi: 10.17226/27864.
×
Page 276
Page 277
Suggested Citation:"Appendix J: Design Example 8 - Embankment Over Clay Over Rock Using PileAXL." National Academies of Sciences, Engineering, and Medicine. 2024. Pile Design for Downdrag: Examples and Supporting Materials. Washington, DC: The National Academies Press. doi: 10.17226/27864.
×
Page 277
Page 278
Suggested Citation:"Appendix J: Design Example 8 - Embankment Over Clay Over Rock Using PileAXL." National Academies of Sciences, Engineering, and Medicine. 2024. Pile Design for Downdrag: Examples and Supporting Materials. Washington, DC: The National Academies Press. doi: 10.17226/27864.
×
Page 278
Page 279
Suggested Citation:"Appendix J: Design Example 8 - Embankment Over Clay Over Rock Using PileAXL." National Academies of Sciences, Engineering, and Medicine. 2024. Pile Design for Downdrag: Examples and Supporting Materials. Washington, DC: The National Academies Press. doi: 10.17226/27864.
×
Page 279
Page 280
Suggested Citation:"Appendix J: Design Example 8 - Embankment Over Clay Over Rock Using PileAXL." National Academies of Sciences, Engineering, and Medicine. 2024. Pile Design for Downdrag: Examples and Supporting Materials. Washington, DC: The National Academies Press. doi: 10.17226/27864.
×
Page 280
Page 281
Suggested Citation:"Appendix J: Design Example 8 - Embankment Over Clay Over Rock Using PileAXL." National Academies of Sciences, Engineering, and Medicine. 2024. Pile Design for Downdrag: Examples and Supporting Materials. Washington, DC: The National Academies Press. doi: 10.17226/27864.
×
Page 281
Page 282
Suggested Citation:"Appendix J: Design Example 8 - Embankment Over Clay Over Rock Using PileAXL." National Academies of Sciences, Engineering, and Medicine. 2024. Pile Design for Downdrag: Examples and Supporting Materials. Washington, DC: The National Academies Press. doi: 10.17226/27864.
×
Page 282
Pile Design for Downdrag: Examples and Supporting Materials Get This Book
×
 Pile Design for Downdrag: Examples and Supporting Materials
MyNAP members save 10% online.
Login or Register to save!
Download Free PDF

NCHRP Web-Only Document 398: Pile Design for Downdrag: Examples and Supporting Materials from TRB's National Cooperative Highway Research Program, provides appendices to NCHRP Research Report 1112: Design of Piles for Downdrag.

READ FREE ONLINE

  1. ×

    Welcome to OpenBook!

    You're looking at OpenBook, NAP.edu's online reading room since 1999. Based on feedback from you, our users, we've made some improvements that make it easier than ever to read thousands of publications on our website.

    Do you want to take a quick tour of the OpenBook's features?

    No Thanks Take a Tour »
  2. ×

    Show this book's table of contents, where you can jump to any chapter by name.

    « Back Next »
  3. ×

    ...or use these buttons to go back to the previous chapter or skip to the next one.

    « Back Next »
  4. ×

    Jump up to the previous page or down to the next one. Also, you can type in a page number and press Enter to go directly to that page in the book.

    « Back Next »
  5. ×

    Switch between the Original Pages, where you can read the report as it appeared in print, and Text Pages for the web version, where you can highlight and search the text.

    « Back Next »
  6. ×

    To search the entire text of this book, type in your search term here and press Enter.

    « Back Next »
  7. ×

    Share a link to this book page on your preferred social network or via email.

    « Back Next »
  8. ×

    View our suggested citation for this chapter.

    « Back Next »
  9. ×

    Ready to take your reading offline? Click here to buy this book in print or download it as a free PDF, if available.

    « Back Next »
Stay Connected!